Dr. Basil Notes- Nac Osce Vip

February 8, 2017 | Author: amroamasha | Category: N/A
Share Embed Donate


Short Description

nac osce...

Description

OSCE guide Third edition

Table of contents Table of contents ............................................................................................................................. 2 History taking – Medicine ............................................................................................................... 6 General review:............................................................................................................................ 7 Notes .......................................................................................................................................... 12 Chest pain – ACUTE ................................................................................................................. 14 Chest pain – CHRONIC ............................................................................................................ 16 Headache.................................................................................................................................... 18 Blood results / Macrocytic Anemia / B12 Deficiency................................................................. 21 Difficulty swallowing ................................................................................................................ 23 Elevated liver enzymes .............................................................................................................. 24 Drinking / Alcohol ..................................................................................................................... 26 Fever / Tired .............................................................................................................................. 28 Diarrhea – ACUTE .................................................................................................................... 29 Diarrhea – CHRONIC ............................................................................................................... 30 ASTHMA .................................................................................................................................. 32 COPD management ................................................................................................................... 35 Ankle swelling – Bilateral ......................................................................................................... 36 Ankle swelling – Unilateral ....................................................................................................... 37 Congestive heart failure – CHF ................................................................................................. 38 Heart racing ............................................................................................................................... 40 Fall ............................................................................................................................................. 42 Peripheral vascular disease: ....................................................................................................... 44 Urinary symptoms: .................................................................................................................... 45 Anuria ........................................................................................................................................ 46 Hematuria .................................................................................................................................. 47 Renal stones ............................................................................................................................... 48 Incontinence............................................................................................................................... 50 Lump – Neck Swelling .............................................................................................................. 51 Lump – Breast............................................................................................................................ 52 Dizziness.................................................................................................................................... 55 INR – Counselling ..................................................................................................................... 57 Patient is receiving blood – counsel for adverse reactions ........................................................ 59 Counselling – Ventilator............................................................................................................ 61 Ethical questions ........................................................................................................................ 62 HIV post-test counselling .......................................................................................................... 63 Lung Nodule .............................................................................................................................. 64 High Creatinine.......................................................................................................................... 66 Impotence / Erectile Dysfunction .............................................................................................. 67 Rheumatology – History Taking................................................................................................ 68 Multiple Sclerosis ...................................................................................................................... 69 Obesity....................................................................................................................................... 70 Epilepsy Counselling ................................................................................................................. 71 Medical note .............................................................................................................................. 73 Pre-diabetes – Counselling ........................................................................................................ 74 Emergency Medicine..................................................................................................................... 75 Emergency Room ...................................................................................................................... 76 Trauma ....................................................................................................................................... 77

OSCE-guide-III.doc

Page 2 of 255

Unconscious Patient – Neuro..................................................................................................... 81 Unconscious Patient – Diabetic ................................................................................................. 82 Unconscious Seizing Patient – DT / Epilepsy / Brain tumour / …............................................ 84 Heart Attack – Chest Pain (MI or Heart Block) ........................................................................ 86 Case 1: Chest pain with initial normal ECG.............................................................................. 87 Case 2: Chest pain with STEMI ................................................................................................ 89 Case 3: Chest pain – v fibrillation / v tachy............................................................................... 90 Case 4: Chest pain – v fibrillation – intoxicated patient ............................................................ 90 Heart Block................................................................................................................................ 91 Headache.................................................................................................................................... 92 Acute Abdominal Pain............................................................................................................... 93 Upper GIT bleeding ................................................................................................................... 98 Lower GIT bleeding................................................................................................................... 99 ECG ......................................................................................................................................... 100 Phone calls ............................................................................................................................... 103 Physical Examination .................................................................................................................. 106 Medical Physical Exam ........................................................................................................... 107 Abdominal examination:.......................................................................................................... 109 Liver Examination ................................................................................................................... 111 Nasal bleeding  Hematological Examination....................................................................... 112 Chest Examination................................................................................................................... 114 Pneumonia ............................................................................................................................... 115 Cardiac Examination – Essential HTN case ............................................................................ 117 Secondary Hypertension .......................................................................................................... 119 Hypertension............................................................................................................................ 120 SOB – shortness of breathe...................................................................................................... 121 DVT ......................................................................................................................................... 122 Peripheral Arterial Disease Examination................................................................................. 125 Diabetic Foot ........................................................................................................................... 126 Neurological Examination ....................................................................................................... 127 Cranial Nerves Examination.................................................................................................... 129 Tremors.................................................................................................................................... 131 Thyroid Exam .......................................................................................................................... 132 Dermatomes ............................................................................................................................. 133 Neck Examination.................................................................................................................... 134 Carpal Tunnel Syndrome ......................................................................................................... 135 Hand Laceration / Wrist Laceration......................................................................................... 136 Back Pain ................................................................................................................................. 137 Acute Back Pain....................................................................................................................... 138 Chronic Back Pain ................................................................................................................... 139 Back Joint Examination ........................................................................................................... 140 Ankle Twist ............................................................................................................................. 141 Shoulder Joint .......................................................................................................................... 142 Elbow....................................................................................................................................... 143 Hip Joint................................................................................................................................... 144 Knee Joint ................................................................................................................................ 145 Obstetrics and Gynecology.......................................................................................................... 146 History taking – OB-GYN ....................................................................................................... 147 OB/GYN cases......................................................................................................................... 147 MGOS history questions:......................................................................................................... 148 History of pregnant lady – third trimester................................................................................ 149

OSCE-guide-III.doc

Page 3 of 255

Vaginal Discharge.................................................................................................................... 150 Vaginal Bleeding – Non-Pregnant / Not-Known Pregnant...................................................... 151 Vaginal Bleeding – Pregnant / Ante-Partum Hemorrhage....................................................... 152 Abnormal Uterine Bleeding (AUB)......................................................................................... 153 Amenorrhea ............................................................................................................................. 155 Infertility .................................................................................................................................. 156 Counselling – pre-eclampsia.................................................................................................... 157 Caesarean Section – Counselling – wants to have CS............................................................. 159 Caesarean Section – Counselling – does not want to have CS ................................................ 161 OCPs / Contraception Counselling .......................................................................................... 163 HRT counselling...................................................................................................................... 166 Needle Stick Counselling – HIV ............................................................................................. 168 Counselling – PAP smear ........................................................................................................ 170 Antenatal Counselling.............................................................................................................. 172 Endometriosis .......................................................................................................................... 174 Woman wanting an abortion.................................................................................................... 175 Osteoporosis – Counselling / OR / Short Case ........................................................................ 176 Pediatrics ..................................................................................................................................... 177 History taking – Pediatrics....................................................................................................... 178 Jaundice ................................................................................................................................... 182 IUGR........................................................................................................................................ 184 Crying Baby............................................................................................................................. 185 Chronic Cough – Asthma ........................................................................................................ 186 Anemia..................................................................................................................................... 188 Vomiting .................................................................................................................................. 189 Diarrhea ................................................................................................................................... 191 Mother worried about her child weight ................................................................................... 193 Fever ........................................................................................................................................ 195 Runny Nose / Flu / URTI......................................................................................................... 196 Rash ......................................................................................................................................... 197 Delayed Speech........................................................................................................................ 198 Seizing child counselling ......................................................................................................... 199 ADHD counselling .................................................................................................................. 200 Vaccination counselling........................................................................................................... 202 Child with DM counselling...................................................................................................... 204 Bed wetting counselling / Nocturnal Enuresis......................................................................... 205 Breast feeding counselling....................................................................................................... 207 Psychiatry .................................................................................................................................... 209 Mental status exam – the psychiatry interview........................................................................ 210 DSM-IV-TR............................................................................................................................. 213 History taking – Psychiatry...................................................................................................... 214 Psychosis.................................................................................................................................. 216 Schizotypal personality disorder.............................................................................................. 218 Panic attack.............................................................................................................................. 219 Tiredness OR weight loss ........................................................................................................ 221 Sleep / fatigue notes................................................................................................................. 222 Insomnia .................................................................................................................................. 223 Domestic Violence – Spouse Abuse ........................................................................................ 224 Child Abuse ............................................................................................................................. 226 Domestic abuser....................................................................................................................... 228 Depression ............................................................................................................................... 230

OSCE-guide-III.doc

Page 4 of 255

Depression management / counselling..................................................................................... 230 Dysthymia................................................................................................................................ 231 Premenstrual Dysphoric Disorder (PMDD)............................................................................. 232 Abdominal Pain / Headache .................................................................................................... 233 Somatoform disorders DD ....................................................................................................... 234 Drug seeker .............................................................................................................................. 236 Lithium discontinuity............................................................................................................... 238 Manic patient ........................................................................................................................... 240 Suicidal attempt ....................................................................................................................... 241 Eating disorder......................................................................................................................... 243 Mini-mental status exam:......................................................................................................... 245 Dementia.................................................................................................................................. 246 Delirium................................................................................................................................... 248 Case 1: Dad has not been himself / not sleeping well.............................................................. 248 Case 2: DT ............................................................................................................................... 249 Smoking Cessation – counselling ............................................................................................ 250 Refusal to treatment – counselling........................................................................................... 251 Truth telling ............................................................................................................................. 252 Organ Donation........................................................................................................................ 253 OCD......................................................................................................................................... 254 NOTES .................................................................................................................................... 255

OSCE-guide-III.doc

Page 5 of 255

History taking – Medicine

History taking – Medicine

OSCE-guide-III.doc

Page 6 of 255

History taking – Medicine

General review:      

Introduction: Chief complaint History of present illness Past medical history Family history Social history Introduction

Chief complaint

Analysis of CC

HPI

Impact Red flags

TIME: Os Cf D Character: PQRST ↑ ↓  Chronic diarrhea: dehydration  Anemia: fatigue  Cancer: metastasis Constitutional symptoms Risk factors

Differential diagnosis Allergy Medications PMH: diseases (DM, HTN, heart attack, stroke, cancer) LMP / Last tetanus shot Events: hospitalization / surgery

PMH

A M P L E

FH

Any long term disease Any specific disease

SH

How do you support yourself financially? With whom do you live? SAD

OSCE-guide-III.doc

Page 7 of 255

History taking – Medicine

Introduction: - Knock the door - Go to the examiner  give stickers  use alcohol rub (disinfective) - Stand at the edge of the chair - Good evening Mr …, I am Dr …, I am the physician in charge today, o < 18 years: use first name o > 18 years: use Mr / Mrs / Ms - I understand that you are here because of …, Examples: - History Taking: Good evening Mr …, I am Dr …, I am the physician on duty now, and I understand that you are here today because of …. In the next few minutes I will ask you some questions about your cc, to figure out a working plan that can help you. If you have any concerns or questions, please fell free to stop me and let me know. -

Physical examination: Good evening Mr …, I am Dr …; I am the physician in charge now. I understand that you are here today because of …. In the next few minutes I will do a physical exam on your (e.g. shoulder), during which I will ask you to do some movements that may cause some discomfort and may be some pain, if you feel either, please do not hesitate to stop me. And if you have any concerns, please let me know. And I will be telling the findings to the examiner while we proceed.

1- Chief complaint [A] If the CC not known - How can I help you today? 1. Start to ask based on the age: MALE FEMALE > 65 yrs  Do you take meds on regular basis? Do you have a list of it? Or the bottles? Do you take sleeping pills?  Do you have difficulties with sleeping?  Do you have difficulties with your balance1, any falls?  Do you have difficulties with urination (incontinence / retention)?  Do you have changes in your vision / hearing?  Do you have changes in your mood / memory? 50s  Do you have problem drinking  use CAGE  Depression2  identify through social history  ED / Impotence 30s  Psychiatric problems  SAD  social history Teen / 20s  Premature ejaculation  Abortion  STDs  STDs  Eating disorders 2. Special conditions:  Fatigue  Insomnia  Headache  Abdominal pain  Vaginal bleeding 1 2

Domestic abuse

Normal pressure hydrocephalus: ataxia / incontinence / dementia Common triad association: alcohol / depression / suicide

OSCE-guide-III.doc

Page 8 of 255

History taking – Medicine

    

Chest pain SOB Heart racing Dizziness Numbness

[B] If the CC is known CLEAR  Any pain: headache, chest pain  Cough  Fever

Clarify: 1. Start first open-ended questions: - What do you mean? - Can you tell me more about this 2. Active listening: - Do not interrupt - Do not duplicate 3. Body language: nod your head

Panic attack

NOT clear  Vague symptoms: dizziness, tired  When the patient uses medical terms: abortion, jaundice, palpitation  When the patient uses the words: change / difficulty Clarify: - Use closed ended questions

2- History of present illness TIME (Os Cf D): - Onset: o How did it start (sudden / gradual)? o Setting: what were you doing? - Course: from that time till now, is your cc all the time or is it on and off (continuous vs. intermittent)? o Continuous:  From the beginning  Does it ↑ or ↓ or the same?  Frequency: is it your first time? o Intermittent:  Frequency: how often did you have it in the last (…)?  Are these attacks similar or different? • ↑ in duration (longer) or ↑ in severity (more severe) or ↑ in frequency (more often)?  What brings it? On doing certain thing, occurring at rest, awakening you from sleep? - Duration: o Usually given in the question o If > 24 hours  empathy: were you able to sleep Character: + PAIN: PQRST always ask “from the beginning?” - Position: where did it start? Can you point with one finger on it? - Quality: how does it feel like? Squeezing, tightness, sharp, stabbing, burning? - Radiation: does it shoot anywhere?

OSCE-guide-III.doc

Page 9 of 255

History taking – Medicine

-

-

-

Severity: on a scale of 10, 1 being the mildest pain you have ever had and 10 is the most severe, how much do you rate this one? o If bad pain  empathy: this must be difficult Timing: o Does it change with time; is it more in morning or towards the end of the day? o Any variation? Triggers: o What brings your headache? o Is it related to: stress / lack of sleep / over sleep / flashing lights / smells? o If female: is it related to your periods? Are you taking any meds or OCPs? o Any diet triggers?

+ Fluids (e.g. bleeding, diarrhea, vomitus): - COCA±B/D: colour, odour, contents/consistency, amount ± blood/discharge What ↑ or ↓: - What increases or decreases your cc? - Examples: noise / quiet places / movements / resting / coughing / leaning forward / lying down / [D] ASSOCIATED SYMPTOMS: - In addition to your cc, did you notice any other symptoms? - Now, I am going to ask you more questions to see if you have any other symptoms beside your cc. + By systems: e.g. chest pain 1- Same system 2- Near-by systems 3- Constitutional symptoms [RED FLAGS] 4- Risk factors Now, I am going to ask you more questions to see if you have any medical conditions that may explain / cause / predispose your cc. 5- Causes and consequences 6- Review of systems: + By differential diagnosis: e.g. headache + By causes and consequences: e.g. Macrocytic anemia – Vit B12 deficiency 3- Past medical history Because it is the first time I see you, I need to ask you some questions about your past medical history.  Allergy / Medications: a. Allergy b. Medications (OTC, Rx meds, supplements, herbs)  Past history of diseases for which you see doctor on regular basis (DM, high blood pressure, heart attack, stroke, cancer)?  LMP for females  Events: any history of hospitalization / procedures? 4- Family history Because it is the first time I see you, I need to ask you some questions about your family medical history, and by this I mean your parents and siblings.

OSCE-guide-III.doc

Page 10 of 255

History taking – Medicine

1- How do you describe their general health? 2- Any long term disease in the family? DM, HTN, heart attack, stroke? 3- Any specific disease runs in the family? 5- Social history 1- How do you support yourself financially? 2- With whom do you live? a. Alone  are you in any relationship? Are you sexually active? b. With a family  how is the relation with …? Is she/he supportive? 3- SAD: a. Do you smoke? b. Do you drink alcohol? i. How much? ii. For how long? c. Have you ever tried recreational drugs?

OSCE-guide-III.doc

Page 11 of 255

History taking – Medicine

Notes EMPATHY: If during history taking you noticed the patient is in pain  empathy: I can see you are in pain, please bear with me for few minutes and I will give you a pain medication as soon as I can - In the short cases (5 minutes)  use at least 1-2 empathy statement - In the long cases (10 minutes)  use at least 3 empathy statements -

Patient says “I’m not ok / I’m not so good”  I am sorry to hear that Patient says “I fell down”  Oh, did you hurt yourself / “No” – I am glad to hear that After suicide  It looks like you have gone through difficult times, can you tell me more about these difficulties you are facing Patient is regaining consciousness in the ER  Mr … you have had … and you are in the hospital now, you are ok now, I am Dr … and we are here to make sure you’re ok

I have a concern! Whenever the patient says: “I have a concern”: STOP the interview! - Can you tell me your concerns! - Why are you concerned? Worried / occupied patient! Whenever the patient shows non-verbal clues of being worried / occupied: - I can see that you are worried / occupied! Would you like to tell me more about your worries or concerns? -

Question types: Types of questions you can use: open-ended, closed-ended, choices Types of questions you can NOT use: leading questions, stacking questions

Time usage technique: 1/2/3 If you are stuck during the history taking; and could not find questions to ask; - First time: Summarize - Second time: Ask about constitutional symptoms / Review of systems - Third time: PMH / FH / SH Medications: When you ask about the medications and even if the patient says NO, in certain diseases, you need to confirm special medications, by saying, what about … - Diarrhea  what about antibiotics - Asthma  aspirin / β–blockers (HTN, heart failure, thyroid disease, social phobias) - Migraine  what about OCPs? (any birth control pills) - Bleeding  what about aspirin / blood thinners - Torticollis  do you take anti-psychotics? Do you see psychiatrist/ did he give you any medications MSD (mood / suicide / drinking): whenever you find one, screen for the others When the patient comes with a chronic long duration complaint, ask him: and what happened recently that made you decide to seek medical advice now? Whenever the patient has something affecting his life / social issue: Refer to social worker / services Do NOT criticize other doctors or the patient

OSCE-guide-III.doc

Page 12 of 255

History taking – Medicine

For breaking bad news [SPIKES] approach: o Setting o Perception of the patient: what do you know about … o Invitation: how much details you would like to know about … o Knowledge: give knowledge in understandable pieces, and make sure the patient understands this info. o Empathy /+/ Expectations: what are your expectations from today’s visit  You will have tremendous support, you are not working alone  We will try to make you as comfortable as possible  Oncology group AND palliative group: Referral: do you have any doctor you are comfortable with? o Summarize /+/ Strategy Counselling: 1. Inform the patient a. The medical condition is called “…” b. Explain the pathophysiology c. Consequences / complications of the condition! May happen again, may affect ability to do certain things, … d. Investigations that might be needed to conclude the condition OR to look for complications 2. Preventive measures: e.g. modify the poly-pharmacy … 3. Treatment: life style / medications (side effects / alternatives / consequences of not receiving treatment) 4. Offer more info: brochures / web sites / support groups 5. Break every 30-60 seconds (check & recheck that your patient understands); ask the patient: does that make sense? Is this acceptable? Reasonable? Is it clear? 6. General tips for the counselling sessions: - Make it interactive not lecturing - At the beginning ask whether your patient has a specific concern - Do not mislead your patient; if you are not sure about any thing, say that this is a very good question and you are going to check the answer for him. -

-

Whenever you hear “car accident”: I am sorry to hear that! Was anyone hurt? I am sorry for that Were you driving or a passenger? If you do not know the answer to a question: This is a good question / point, I will check it for you and we will discuss it next visit. It is better to refer you to the specialist; there are too many points regarding this issue that it will be better to discuss it with the specialist. A good statement to use in different counselling situations: always in medicine, we balance the benefits and the side effects. GIT symptoms: Nausea / Vomiting - Heart burn / acidic taste in mouth Abdominal pain - Distension / bloating / gas Change in bowel movements: constipation / diarrhea Blood in stools / vomiting blood LIVER: yellowish discoloration / itching / dark urine / pale stools

OSCE-guide-III.doc

Page 13 of 255

History taking – Medicine

Chest pain – ACUTE Introduction: Good evening Mr …, I am Dr …, I am the physician on duty now, and I understand that you are here because you have chest pain for the last … minutes. In the next few minutes I will ask you some questions about your chest pain, to figure out a working plan that can help you. If you have any concerns or questions, please fell free to stop me and let me know Analysis of  OsCfD: Onset / setting: what were you doing? CC  PQRST: ─ Position: where did it start? Can you point with one finger on it? ─ Quality: how does it feel like? Squeezing, tightness ─ Radiation: does it shoot anywhere? Your jaw, your shoulders, your back?  What ↑ or ↓: ─ Breathing / position ─ How did you come to the clinic today? Ambulance  did they give you aspirin / nitrates? Did it help you? Impact  Atherosclerosis: ─ Hx of stroke? Symptoms of stroke (weakness / numbness / change in vision / difficulty finding words)? ─ Any sexual dysfunction? ─ Do you feel abdominal pain after eating? ─ Do you feel cold extremities ─ Do you feel cold feet? Pain after walking?  CHF: ─ SOB? How many pillows do you use? Do you wake up gasping for air? ─ Any swelling in your LL? How high does it go? Is it related to position? ─ Eye puffiness? Pain on the liver? Red flags Constitutional  Fever / night sweats / chills  How about your appetite? Any weight changes? symptoms  Any lumps or bumps in your body? Risk factors CAD Pericarditis PE Differential Same system  Nausea / vomiting diagnosis  Sweating / feeling tired  SOB  if yes, analyze (OsCfD)  Do you feel your heart racing?  Did you feel dizzy / light headedness / LOC? Are you tired?  Did you notice swelling in your ankles? Legs? Calf muscles? Near by  CHEST: systems ─ Any cough or phlegm? Chest tightness? Wheezes? ─ Recent fever / flu like symptoms? Muscles/ joint ache?  GIT: ─ Difficulty swallowing (esophageal spasm) ─ Heart burn / acidic taste in your mouth? ─ Any hx of PUD? Reflux? GERD?  Chest wall: any trauma, any blisters / skin rash on your skin  DVT: any pain / swelling / redness in your legs / calves? Any recent long travel?

OSCE-guide-III.doc

Page 14 of 255

History taking – Medicine

Risk Factors: ─ CAD (Coronary Artery Disease): MAJOR: o High blood pressure o High blood sugar o High cholesterol: have you got your cholesterol measured before? o Family hx of heart attack at age < 50 yrs o SAD: Smoking / Cocaine MINOR: o Look for obesity o Do you exercise o How about your diet, do you eat a lot of fast food? o Are you under stress? ─ Pericarditis: o Recent flu like symptoms o Medications (Isoniazide / Rifampicin) o Hx of surgery o Hx of heart attack o Hx of kidney disease / puffy face / frothy urine o Hx of TB o Hx of autoimmune disease ─ Pulmonary Embolism: o Recent long flight o History of malignancy o Family history of blood clots o Female: pregnancy / OCPs / HRT Chest Pain Acute Chronic Minutes – hours Hours – days Intermittent Continuous Cardiac: Cardiac: Cardiac: - CAD - Pericarditis - Unstable angina - Aortic dissection - Unstable angina Non-cardiac: Non-cardiac - Cancer - Tension - Pneumonia - Herpes zoster pneumothorax - Pleurisy - Trauma Panic attack Pulmonary embolism Panic attack GIT: GIT: - GERD - GERD - PUD - PUD - Esophageal spasm - Esophageal spasm Questions:  Investigations: ECG / Cardiac enzymes

OSCE-guide-III.doc

Page 15 of 255

History taking – Medicine

Chest pain – CHRONIC Chest pain for 6 weeks UNSTABLE ANGINA /+/ GERD – GIT CAUSES OF CHEST PAIN Intro Analysis of CC

… But first I would like to ask you, how do you feel now? OsCfD: Onset / setting: what were you doing? PQRST: ─ Position: where did it start? Can you point with one finger on it? ─ Quality: how does it feel like? Squeezing, tightness ─ Radiation: does it shoot anywhere? Your jaw, your shoulders, your back?  What ↑ or ↓: ─ Breathing / position ─ Is it related to activity? How many blocks were you able to walk? And now? ─ How about rest? And during night? ─ When was the last attack  Triggers Angina GERD  

Exertion  Golf (leaning forward) Stress (emotional)  Coffee / dairy products Cold air  Smoking / Alcohol Heavy meals  Heavy / late meals Sexual activity  Pregnancy (progestin) Impact Effect  Atherosclerosis  Chronic cough  CHF  Change in the voice Red flags Constitutional  Fever / night sweats / chills symptoms  How about your appetite? Any weight changes?  Any lumps or bumps in your body? Risk factors CAD Differential Same system  Nausea / vomiting diagnosis  Sweating / feeling tired  SOB  if yes, analyze (OsCfD)  Do you feel your heart racing?  Did you feel dizzy / light headedness / LOC? Are you tired?  Did you notice swelling in your ankles? Legs? Calf muscles? Near by  CHEST: systems ─ Any cough or phlegm? ─ Chest tightness? Wheezes? ─ Recent fever / flu like symptoms? Muscles/ joint ache?  GIT: ─ Difficulty swallowing (esophageal spasm) ─ Heart burn / acidic taste in your mouth? ─ Any hx of PUD? Reflux? GERD?  Chest wall: any trauma, any blisters / skin rash on your skin  DVT: any pain / swelling / redness in your legs / calves? Any recent long travel? PMH FH SH     

OSCE-guide-III.doc

Page 16 of 255

History taking – Medicine

Counselling: Concern: The patient has a concern; is this IHD? Is his heart endangered? ─ This is quite a reasonable concern? What made you think about that? ─ Especially you have many risk factors that may predispose to heart attack. Right now the physical exam is ok; it is less likely your condition is due to heart problem. But we still need to check your heart more, we will do some lab works and an electrical tracing for your heart (ECG), then if we find that we still need, we may send you to have a stress ECG, in which, we trace your heart while you are exercising. Then we know for sure the condition of your heart. ─ However, we would like to take measures to try to decrease your risk of developing heart attack, e.g. exercise / diet / smoking / cholesterol. ─ On the other hand, the most likely diagnosis of what you have is a medical condition called “GERD”. GERD stands for Gastro-Esophageal Reflux Disease, any idea about that? Do you know anything about GERD? ─ Explain with a drawing: the esophagus (food pipe) / lower esophageal sphincter / physiologic mechanism to keep it competent / in GERD  weak sphincter  acid refluxes / irritates the esophagus / impact (short term and long term) ─ Treatment: o Avoid triggers o Life style modifications:  Raise the head of the bed  Smaller meals  Do not eat late  ↓ smoking o Medications: proton pump inhibitors (PPIs), e.g. pantoprazole o Side effects of PPIs:  In general, proton pump inhibitors are well tolerated, and the incidence of short-term adverse effects is relatively uncommon  Common adverse effects include: headache (in 5.5% of users in clinical trials), nausea, diarrhea, abdominal pain, fatigue, and dizziness. Long-term use is associated with hypomagnesemia  Because the body uses gastric acid to release B12 from food particles, decreased vitamin B12 absorption may occur with long-term use of protonpump inhibitors and may lead to Vitamin B12 deficiency  Infrequent adverse effects include rash, itch, flatulence, constipation, and anxiety

OSCE-guide-III.doc

Page 17 of 255

History taking – Medicine

Headache HPI: ─ OsCfD: gradual onset / all the time / increasing / for few days ─ PQRST: temporal area / vague deep pain / severe o Severe:  empathy: this must be difficult, were you able to sleep o Triggers:  What brings your headache?  Is it related to: stress / lack of sleep / flashing lights / smells / diet?  If female: is it related to your periods? Are you taking any OCPs? ─ What ↑ or ↓? Lying down / coughing / resting in quiet room / … Associated symptoms / differential diagnosis: 1- Infection: ─ Fever / night sweats / chills / constitutional symptoms ─ Neck pain / stiffness ─ Recent flu like symptoms / skin rash ─ Bothered by light ─ Nausea / vomiting 2- Subdural hematoma: ─ Trauma / fall ─ SAD (Smoking, Alcohol, Drugs) 3- Subarachnoid hemorrhage: ─ Very acute /+/ Very severe headache / the worst headache ─ History of aneurysm or polycystic kidney disease ─ Visual changes (pupil changes) ─ Your heart is beating slow 4- Neurological screening: If while you are doing the neurological screening, you suspect particular cause, e.g. temporal arteritis  go to TA block then return to complete the neurological screening. ─ Cranial nerves: o Any change in smelling perception? o Any difficulty in vision / vision loss? o Any difficulty in hearing / buzzing sounds? o Difficulty finding words? Aphasia? o Difficulty swallowing? ─ Brain: o Any dizziness / light headedness / LOC? o Any tremors / jerky movements / hx of seizures? ─ Personality and cognition: o Any memory / mood / concentration problems? o Did anybody tell you that you there is a change in your personality recently? ─ UL/LL: o Any weakness / numbness / tingling in your arms / legs o Any difficulty in your balance / any falls? ─ Spine: o Any difficulty with urination / need to strain to pass urine? o Any change in bowel movements?

OSCE-guide-III.doc

Page 18 of 255

History taking – Medicine

5- Temporal arteritis: ─ Age > 55 years ─ When you touch this part of your head, is it painful? Can you comb your hair? ─ Do you feel cord-like structure? ─ Do you have any visual disturbances / impairment? ─ When you are chewing, is it painful, cramps in your jaws? ─ Any weakness / numbness in your shoulders / hips? ─ Is there any cough? Mild fever? 6- HTN: ─ Were you diagnosed before with high blood pressure? ─ Do you know your blood pressure? Have you had it checked before? ─ Salty food? Family history of HTN / heart disease? ─ Any history of repeated headaches? 7- Extra-cranial causes of headache: ─ Eyes: any hx of glaucoma, red eye, pain in your eyes? Do you usually wear eyeglasses? Do you see well? Any vision problems? When was last time you saw your optometrist? ─ E – do you have any pain / discharge in your ears? ─ N – nasal discharge / sinusitis / hx of facial pain? ─ T – any teeth pain / difficulty swallowing? 8- Medications: ─ Do you take any nitrates? ─ Do use too much of advil (or other NSAIDs)? For how long? ─ Were you used to take large amounts of coffee and then you stopped abruptly? ─ OCPs? Temporal Arteritis: Investigations: ─ TA biopsy ─ Doppler ─ ESR ─ CT head

Treatment: If suspect GCA (Giant Call Arteritis), immediately start high dose prednisone; 1 mg/kg OD (to prevent blindness) then maintain dose daily (in divided doses), then taper prednisone dose after symptoms resolve.

Polymyalgia Rheumatica: ─ Constitutional symptoms + Fatigue ─ Age > 50 yrs ─ ESR > 50 mm/hr

Treatment: Corticosteroids; 15 mg/day (for long periods of time). Taper after ESR decreases < 50 mm/hr and stop if ESR normalizes (< 20 mm/hr)

OSCE-guide-III.doc

Page 19 of 255

History taking – Medicine

PRIMARY HEADACHE Intermittent / episodic Headache Tension Duration Days Quality Pressing / tightening / bilateral Place Associated symptoms Aggravating factors Others

Band around the head

Stress Physical injury

Treatment - Acetaminophen - NSAIDs - Physiotherapy / ms massage / heat compresses (neck)

OSCE-guide-III.doc

Migraine Hours Mostly unilateral / pulsating / interferes with daily activities Mostly unilateral Photophobia / phonophobia Physical activity / motion Light / sound  Family history  Types: + Classical: with aura + Non-classical: no aura Acute phase: - Acetaminophen - NSAIDs (ibuprofen)

Cluster Minutes Comes in series / severe pain / hyperaesthesia Around the eyes / nose Red eyes / lacrimation / rhinorrhea / sweating Smoking / alcohol Smell / exercise

- Oxygen - NSAIDs - Triptans / ergotamine

Prophylactic: - Remove precipitant - Ca channel blockers - Triptans (somatriptan)

Page 20 of 255

History taking – Medicine

Blood results / Macrocytic Anemia / B12 Deficiency Cases: ─ ─

Middle age man received blood report showing Macrocytic anemia Elderly (65 years old) man presenting with ataxia, dizziness, macrocytic anemia. Findings: poor diet. Most likely diagnosis: pernicious anemia

Investigations: ─ CBC / Differential / Peripheral blood film ─ B12 level in the blood / Folic acid level in the blood Introduction Good morning Mr …, I am Dr …, I understand that you are here today (OR we called you to come) to get the results of your blood tests (OR x-ray) that you have done few days ago, I have it and I am going to discuss it with you. However, because it is the first time that I see you, I need first to ask you some questions to help me get better understanding and interpretation of these results. Is it OK with you? + If patient anxious about results  tell her/him then continue history 1- First let me ask you few questions about the lab test itself (this applies to any blood work, xray, HIV testing, biopsy, jaundice, anemia): ─ Why have you done this test? ─ Is it the first time to have it? ─ Who ordered this test for you? Why? ─ When did you have it? 2- Give the information: ─ If it is bad news  SPIKES ─ If abnormal blood results or x-ray: explain the results to patient, to decrease the patient anxiety. The blood tests you had show that you have special kind of anemia that we call “Macrocytic anemia” in which the size of the red blood cells (which are a component of your blood) is larger than usual, there is different causes for this. o If the patient panicked? Is it serious doctor?  Do you have any concerns?  There are different causes that may lead to this result, some of them are serious, could be, we need to do more investigations. 3- I would like to ask you some questions to see how did this (anemia) affect you: CONSEQUENCES of anemia: ─ Anemia symptoms: o Did anyone comment that you are pale, recently? o Did you notice any ↓ in your activity level? o Heart racing / SOB / chest pain with exercise? o Any dizziness / light headedness / fainting? ─ Neuro symptoms: o Any tingling / numbness / in your feet? o Difficulty in your balance / any falls? o Any difficulty concentrating / memory problems?

OSCE-guide-III.doc

Page 21 of 255

History taking – Medicine

4- I would like also to ask more questions to find out what might be the cause: CAUSES of Vit B12 deficiency: ─ Diet intake: Are you vegetarian? For how long? Do you take supplements? ─ Gastric causes: o Did you have any surgeries in your stomach? When? o History of long standing PUD? Any heaviness / fullness after meals / indigestion? (Lack of acidity) o Were you ever yourself or any member of your family diagnosed with what is called “autoimmune disease”; by this I mean a condition called “pernicious anemia”, or rheumatoid disease / lupus? ─ Terminal ileum: o Did you have any bowel surgery before? o Were you diagnosed with “Crohn’s disease” before? Any repeated attacks of diarrhea? Any foul smelling bulky stools? ─ Pancreatic and liver failure: o Any hx of liver / pancreatic disease? o Yellowish discoloration / itching / dark urine / pale stools? ─ Alcohol: o Do you drink alcohol? How much? For how long? ─ Meds: Do you take medications on regular basis? What kind? o Have you ever been diagnosed with epilepsy? Do you take anti-epileptics? o Do you see a psychiatrist? Do you take a mood stabilizer? o Any hx of chemotherapy? Have you ever taken a drug called “methotrexate”? ─ Hematological causes: o Any recent bleeding (nose / gum / coughing / vomiting blood)? Any bruises / dark urine / tarry stools? o Any fever / night sweats / chills? Change in appetite / weight loss? Lumps and bumps in your body (for LNs)? Bony pains? Any repeated infections? ─ Parasites: o Have you ever consumed raw fish (chronic intestinal infestation by the fish tapeworm: Diphyllobothrium)? 5- PMH 6- FH 7- SH

OSCE-guide-III.doc

Page 22 of 255

History taking – Medicine

Difficulty swallowing What do you mean by difficulty swallowing? ─ Do you feel difficulty initiating the swallowing? ─ Do you feel pain when you swallow? ─ Do you feel food is stuck? Can you point where it is usually stuck? Dysphagia (esophageal)

Progressive All the time and ↑

Progressive, solids then fluids

Mechanical Cancer OR stricture

 

Intermittent On and Off

Achalasia: respiratory symptoms

Fluids first Then solids

Fluids and solids

Solids only (Large bolus)

Achalasia Scleroderma

Esophageal spasm

Esophageal webs and rings

Scleroderma: reflux / tight skin of fingers / change hand color when exposed to cold (Reynaud’s disease)

Mechanical Dysphagia: Analysis  OsCfD: gradual, ↑ progressively, to solids then fluids / PQRST / What ↑ / ↓ of CC  Associated symptoms: ─ The same system: o Nausea / vomiting / undigested food o Change in bowel movements o Change in the size of the abdomen / abd pain / blood in stools o Liver: yellowish discoloration / itching / dark urine/ pale stools ─ Near-by systems: o Any chest pain / tightness o Any cough / change in your voice / neck swelling (thyroid lump) Impact Weight loss Red flags  Constitutional symptoms: fever/ night sweats/ chills / change in appetite / loss of weight / lumps & bumps  Risk factors: ─ GERD / PUD: o Hx of heart burn o Were you ever diagnosed with a condition called GERD / PUD o Have you ever checked with a camera or a light (endoscope) inserted into your stomach ─ Smoking / Alcohol ─ Family history: esophageal cancer ─ Radiation to chest ─ Have you ever swallowed any chemical? Barium swallow: string sign /or/ apple core sign / graded narrowing of intra-esophageal diameter extending from T5-T8 level  most likely diagnosis: esophageal cancer Investigations: endoscopy and biopsy / chest x-ray and CT / liver function tests / abdominal US

OSCE-guide-III.doc

Page 23 of 255

History taking – Medicine

Elevated liver enzymes Introduction HPI: 1- First let me ask you few questions about the lab test itself (this applies to any blood work, xray, HIV testing, biopsy, jaundice, anemia): ─ Why have you done this test? ─ Is it the first time to have it? ─ Who ordered this test for you? Why? ─ When did you have it? 2- Give the information: ─ Liver enzymes: AST / ALT ≥ 2 ─ Explanation: there is increase in one of the markers used to assess / check the liver functions, it may indicate that there is an injury to your liver, I am happy you came here today to discuss it so that we can figure this out. 3- I would like to ask you some questions to see how did this affect you: CONSEQUENCES of liver injury: ─ Acute phase: o Any yellowish discoloration / itching / dark urine/ pale stools o Recently, have you noticed any fever / flu-like symptoms / muscle/joint aches o Constitutional symptoms: sweats / chills / appetite / weight / lumps ─ Chronic manifestations: o Did you notice any increase in the size of your abdomen? Puffiness in your face? Swelling in your legs/ ankles? o Bruises in your body? o Vomited blood? Blood in stools? o WITH ALCOHOL: did you notice changes in memory and concentration? Any weakness / numbness? Balance and falls? 4- I would like to ask you more questions to find what might be the cause: CAUSES of liver injury: Now, I would like to ask you some questions to see if you were exposed to liver disease without being aware of that, some of these questions may be personal, but it is important to ask (start from least offensive to most offensive) Including the alcohol, during which  Drinking assessment 5- PMH: ─ Were you ever diagnosed with liver disease before ─ Were you ever checked for liver disease before ─ Were you ever vaccinated for liver disease before 6- FH: suicide / depression / drinking / liver cancer 7- SH

OSCE-guide-III.doc

Page 24 of 255

History taking – Medicine

Now, I would like to ask you some questions to see if you were exposed to liver disease without being aware of that, some of these questions may be personal, but it is important to ask (start from least offensive to most offensive) Oral ─ ─

Any recent travel outside Canada? Did you eat any raw shell fish? Did you eat in new place that you are not used to?

Surgical ─ Any history of surgeries / hospitalization? ─ Any history of blood transfusion? ─ Any history of blood donation? Social ─ ─

How do you support yourself financially? If hazardous occupation? Did you get exposed to blood products / body fluids?

Risky behaviour ─ Any tattooing or piercing? ─ SAD? o Do you smoke? o Drink alcohol? How about the past?  Drinking assessment o Have you ever tried recreational drugs? Any injected drug use? When was the last time? ─ With whom do you live? For how long have you been together? ─ Before being with your current partner, did you have other partners? ─ When did you start to be sexually active? How many sexual partners did you have from that time till now? ─ What is your preference, men, women or both? ─ What type of sexual activity do you practice? Did you practice safe sex all the time? And by that I mean using condoms! ─ Any history of sexually transmitted infections? And screening for STIs? ─ Have you ever had sex with sex worker? ─ Within the last 12 months, have you had any other sexual partners? When do you need to take extensive sexual history? Risky behaviour!  Liver enzymes / Jaundice  Fever / Tired  LNs  Discharge  PAP results  HIV test results

Liver investigations          

OSCE-guide-III.doc

AST / ALT / GGT / Alkaline phosphatase LDH Bilirubin INR / PTT Albumin Glucose Serum ferritin / TIBC / serum ceruloplasmin Viral serology: Hep A/B/C antibodies and Hep B Ag Abdominal US Liver biopsy

Page 25 of 255

History taking – Medicine

Drinking / Alcohol Drinking assessment

Use / abuse

MOAPS

Drinking hx How much? CAGE

Mood Organic Anxiety Psychosis Self-care / suicide

Impact Medical Social Liver Home Memory / conc. Work B12 Heart

Legal

Use / Abuse: ─ Do you drink alcohol? How about the past? ─ What do you drink? o For how long? o How often? ─ How much? o 2 bottles of wine a day? 12 beers a day? Have you drunk more than 6 drinks in one setting? Have you ever exceeded the amount you intended to drink? o Do you drink alone or with other people? o Did you ever drink to the extent of black out? o What do you feel if you do not drink? Any shaking / heart racing / sweating? Have you ever had seizures before? Were you hospitalized? Did you have delirium tremens? o Do you avoid going to places where you do not have access to alcohol? ─ CAGE: o Did you ever think that you need to cut down on your drinks? o Do you get annoyed by other people criticizing your drinking? o Do you feel guilty for your drinking habits? o Early morning drink? Problem drinking: 2 of CAGE list for males OR 1 for females MOAPS: ─ Mood: How is your mood? Interest? If ok  do not proceed o If not ok  MI PASS ECG Anxiety: o Are you the person who worries too much? o Do you have excessive fears or worries? Psychosis: o Do you hear voices or see things that others do not? o Do you think that someone else would like to hurt you? Self care / suicide o Any chance you might harm yourself or somebody else? o







OSCE-guide-III.doc

Page 26 of 255

History taking – Medicine

Impact: I am going to ask some questions to check what effects does alcohol have on your life? ─ The medical is already done in the consequences of liver injury ─ Social: o With whom do you live? For how long? How is the relation? Is there any problems? Is it related to your drinking habits? o How do you support yourself financially? Where do you work? For how long?  How is the relation with your coworkers / manager?  Do you miss working days because of your drinking habits?  Do you need to drink at work? ─ Legal: o Did you have any legal issues related to your drinking? o Fights? Arguments? Were you arrested before because of drinking? o Were you charged before for DUI (driving under influence)?

Alcoholic beverages: ─ Beer: o Alcohol percent around 5% o Pitcher (60 oz) = 3 pints (pint = 20 ounces) o Ounce (oz) = around 28-30 ml ─ Wine / Champaign: o Alcohol percent around 12% o Bottle: 750 ml o Glass: 150 ml ─ Hard liquor (whisky / gen): o Alcohol percent around 40% o 1 glass (shot) = 1 ½ oz (50 ml) Ethical challenges (patient asks) Doctor is there any possibility that these test results are wrong? Can we repeat it to make sure?  Whenever we do blood work, we take a lot of measures to make sure it is accurate and usually if it is positive, the labs double check it before they send it. Chances of having a mistake are very low.  Based on your history, symptoms and examination, we might not only repeat the test, we might need to do more tests. It is early to tell now, let us proceed with history and examination, and at the end I will be able to tell you. Do you drink yourself, doctor?  Why do you ask?  Whether I drink or no does not matter, it is better to discuss your case now. By the way, are you Canadian graduate, doctor?  I fulfill all the requirements to practice medicine in Canada!  I passed all the exams, follow Canadian guidelines, and practice under supervision!

OSCE-guide-III.doc

Page 27 of 255

History taking – Medicine

Fever / Tired Intro CC

… But first I would like to ask you, how do you feel now?  Fever  Do you have other concerns? Analysis OsCfD  Did you measure it? How often? How? What is highest?  And medications? Did it help?  Any flu / illness / sickness  Any diurnal variation? Any special pattern? Is it more every 3rd or 4th day? (malaria) Impact Are you able to function? Red flags Constitutional symptoms Differential  CNS: headache / neck pain / stiffness / nausea / vomiting / vision changes / diagnosis bothered by light / weakness / numbness  ENT: ─ Ears: pain / discharge Extensive review of ─ Nose: runny nose / sinusitis (facial pain) ─ Throat: sore throat / teeth pain / difficulty swallowing systems  Cardiac: chest pain / heart racing (pericarditis)  Lung (pneumonia, PE (DVT), TB, cancer): cough / blood / phlegm / wheezes / chest tightness / contact with TB pt 3  GIT (except the liver ): abd pain / distension / change in bowel movements / blood in stools  Urinary: burning / frequency / flank pain / blood in urine  Do you have any discharge? Ulcers? Blisters? Warts?  MSK: joint pain / swelling / ulcers in your body / mouth / skin rash / red eye  Autoimmune: fm hx / dx before with autoimmune dis  The LIVER: ─ Local: yellow / itching / dark urine / pale stools ─ Dx before with liver dis? Screened? Vaccinated? ─ Transition to risky behaviour PMH Cancer / Autoimmune disease FH Cancer / Autoimmune disease SH Does your partner have any fever? Discharge? Skin rash?

 

3

During the last 6 weeks, did you seek medical attention? What made you choose to come today? Compared to before, any special changes?

The liver will be put at the end as a transition to ask about risky behaviour (see liver enzymes case)

OSCE-guide-III.doc

Page 28 of 255

History taking – Medicine

Diarrhea – ACUTE Analysis of CC

Impact Red flags

Differential diagnosis

OsCfD COCA ± B/Mucous ↑↓

How many times? What bout during night? ─ Yes  organic ─ No  irritable bowel syndrome (IBS) – day only ─ How does if affect your sleep?  Consistency: watery / loose / formed / bulky. Any floating fat droplets / difficult to flush / undigested food  Did you notice blood? When did it start? ─ Before you have your bowel move? ─ Mixed (higher source of bleeding)? ─ On the surface? AS  Pain  OCD / PQRST ─ If pain improves after bowel movement: IBS  Vomiting  Alteration with constipation  Acute dehydration: thirsty / dizziness / light headedness / LOC / weak  Constitutional symptoms – for infection / cancer  Flu like symptoms  Any body around you have the same diarrhea?  Other causes: ─ Hyperthyroidisms: heat intolerance ─ Stress? What do you do for life? Any stress? Does the diarrhea ↑ with stress? How about your mood? ─ Infectious: travel / camping / with whom do you live? Any other person at home with diarrhea? ─ HIV – if risk factors ─ Diet: Celiac disease / a lot of dairy products / lactose intolerance / lots of juice / sugars ─ Medications: antibiotics / stool softeners  Rheumatic diseases: red eyes / mouth ulcers / skin changes/ rash / nail changes / hx of psoriasis / joint pain / swelling / back pain / stiffness (especially in morning) / discharge / renal stones  

PMH FH SH Bloody diarrhea DD: ─ GE (gastroenteritis) ─ IBD (inflammatory bowel disease) ─ Bleeding peptic ulcer Investigations for clostridium difficile  CBC / differential / lytes and chemistry  Stool culture for parasites  Stool assay for clostridium toxin  Endoscopy  Blood grouping and cross matching

OSCE-guide-III.doc

Treatment for clostridium difficile  Stop the antibiotic  Metronidazole (500 mg tid x 7 days)  If metronidazole is not effective or severe case  vancomycin (125 mg qid x 14 d)

Page 29 of 255

History taking – Medicine

Diarrhea – CHRONIC The same as acute diarrhea, except the impact and red flags Introduction CC Analysis of CC

Impact Red flags

Differential diagnosis

OsCfD COCA ± B/Mucous ↑↓

How many times? What bout during night? ─ Yes  organic ─ No  irritable bowel syndrome (IBS) – day only ─ How does if affect your sleep?  Consistency: watery / loose / formed / bulky. Any floating fat droplets / difficult to flush / undigested food  Did you notice blood? When did it start? ─ Before you have your bowel move? ─ Mixed (higher source of bleeding)? ─ On the surface?  Pain  OCD / PQRST AS ─ If pain improves after bowel movement: IBS  Vomiting  Alteration with constipation  Acute dehydration: thirsty / dizziness / light headedness / LOC / weak  Chronic  weight loss  Constitutional symptoms – for infection / cancer  For cancer: Age / family hx of Ca colon / change in the calibre of stools / what kind of diet  Rheumatic diseases: red eyes / mouth ulcers / skin changes/ rash / nail changes / hx of psoriasis / joint pain / swelling / back pain / stiffness (especially in morning) / discharge / renal stones  Other causes: ─ Hyperthyroidisms: heat intolerance ─ Stress? What do you do for life? Any stress? Does the diarrhea ↑ with stress? How about your mood? ─ Infectious: travel / camping / with whom do you live? Any other person at home with diarrhea? ─ HIV – if risk factors ─ Diet: Celiac disease / a lot of dairy products / lactose intolerance / lots of juice / sugars ─ Medications: antibiotics / stool softeners  

PMH FH SH Rheumatic diseases: IBS / ankylosing spondylitis / psoriasis / reactive arthritis

OSCE-guide-III.doc

Page 30 of 255

History taking – Medicine

Counselling: ─ Explanation: o From what you have told me, the most likely explanation for your diarrhea is the medical condition known as “Irritable Bowel Syndrome”, it is like “unhappy colon” o What do you know about IBS? o We do not know the exact mechanism behind this disease, and it is a common condition, a lot of people have it, this is a long term disease, but it is treatable. o What I need to do is to do physical exam, and do some blood works and stool analysis to rule out other causes, how do you think about that? o Is it serious condition doctor?  It is not serious, as it does not affect life expectancy, and around 80% of patients improve over time ─ Management: o Psychotherapy:  Establish good relationship with the patient  CBT (cognitive behavioural therapy)  If mood is low  depression counselling, it might be a mood problem o Life style modifications:  Stress management and relief ─ Relaxation techniques such as meditation ─ Physical activities such as yoga or tai chi ─ Regular exercise such as swimming, walking or running  Diet modification: lactose-free diet or a diet restricting fructose is sometimes recommended  If drinks too much alcohol  advise to decrease alcohol o Medications  Abdominal pain: ─ Hyoscyamine (antispasmodic): 0.125 to 0.25 mg PO or SL q4h or PRN /OR/ extended-release tablets: 0.375 to 0.75 mg orally every 12 hours (do not exceed 1.5mg in 24 hours) ─ Amitriptyline (10 mg qhs)  Diarrhea: ─ Imodium up to 8 tab / day ─ Lomotil  Constipation: ─ ↑ fibre content in diet ─ Metamucil (psyllium): bulk-producing laxative and fibre supplement  SSRIs o Alternative medicine: ─ Probiotics ─ Herbal remedies, e.g. peppermint oil: ─ Offer more information: o I will give you some brochures and web sites in case you want to read more about that Associated diseases: - IBS - Fibromyalgia - Chronic fatigue syndrome - Interstitial cystitis

OSCE-guide-III.doc

Page 31 of 255

History taking – Medicine

ASTHMA Mr … comes to your office as post-ER visit follow-up, he had asthmatic attack three days ago. He went to ER; he was treated and discharged with advice to see his family physician. Introduction EVENT

Asthma history

Triggers

Infection Medications

Outdoor

Indoor

Stress PMH and FH

How do you feel now?  O S Cf D  Which medication was used? How many times did you need to puff?  Symptoms: SOB / Tightness / Wheezes / Sweating / heart racing / LOC / did you turn blue? Were you able to talk?  Did you call 911 or someone called for you? Did they give you meds? What were these meds?  Were you admitted to hospital? ER? Did they need to put a tube? What were the discharge meds?  When were you diagnosed? How? Type of buffers?  Were you controlled? How many times do you puff (excluding exercise)? Are you using spacer?  Recently, did you notice a need to ↑ the doses?  Any attacks during the night?  Do you use peak flow meter?  Did you have PFTs (pulmonary function tests) done?  How many times did you have to go to ER?  Recent chest infection? Flu-like symptoms? Fever / chills?  How do you use puffers? Stored properly? Not expired?  Did you start new medication? β-blockers? Aspirin? Any recent ↑ in dose of these medications?  Exercise  Cold air  Pollens (is it seasonal?)  Dust: construction / smug (smoke/ fog/ exhaust)  Do you smoke? Anybody around you?  Do you have pets? People around you?  Fabrics related: carpets floor? Any change in linen? Pillows? Blankets? Mattress? Curtains?  Relation to any type of food?  Perfumes  Do you live in a house (basement  mold)?  Any construction renovation? Exposure to chemicals?  Any new stressful situations?  Skin allergies  Other allergies

Asthma Management 1- Confirm diagnosis: ─ Symptoms: o Cough (dry / more at night / more with exercise / induced by allergens) o Wheezes (noisy breathing) o Chest tightness ─ Examination: wheezes ─ Diagnosis: o Chest x-ray: R/O pneumonia / infection / cancer o Pulmonary Function Tests (PFTs):  FEV1/FVC < 80% of expected  obstructive lung disease  Give bronchodilators, repeat PFTs after 20 min, if ↑ > 12%  Asthma

OSCE-guide-III.doc

Page 32 of 255

History taking – Medicine

2- Management: ─ Environment control: avoidance of irritant and allergic triggers (e.g. avoid smoking / change β-blocker for treatment of HTN) ─ Patient education: the allergic nature of the disease and triggering factors ─ Written action plan: see the diagram below (next page) 3- Medications: Type Mild intermittent

Symptoms < 2 times / week

Mild persistent

> 2 times / week but < 1 time / day

Moderate

Daily

Severe

Continuous / Uncontrolled

Treatment Short acting β2-agonist: 1-2 puffs (PRN and before exercise) Short acting β2-agonist (Ventolin 100 mcg – 12 puffs qid) LABA (Serevent 50 mcg – 1 puff bid) Add LABA or LTRA (Singulair 10 mg PO qhs)

Notes Does not need daily medication Low dose ICS (Flovent 125 mcg – 1 puff bid) Moderate dose ICS (Flovent 250 mcg – 1 puff bid) High dose ICS (Flovent 250 mcg – 2 puffs bid)

Oral prednisone

LTRAs are second-line monotherapy for mild asthma 6-11 yrs: ICS should be ↑ to moderate dose > 12 yrs: LABA should be considered first Omalizumab (anti IgE) may be considered in patients > 12 yrs

ICS : Inhaled Corticosteroids; 1 puff = 100 mcg LABA : Long-acting beta2-adrenoceptor agonist LTRA : Leukotriene receptor antagonist

Puffers and LTRA tablets: Medication Ventolin Serevent Flovent

Color Blue Blue Orange

Active ingredient Salbutamol Salmeterol Fluticasone propionate

Advair

Purple

Pulmicort

Brown

Fluticasone Salmeterol Budesonide

Symbicort

Red

Atrovent Spiriva Singulair

Budesonide Formoterol Green Ipratropium bromide Spiriva Tiotropium bromide Handihaler Tablets Montelukast

/ puff 100 mcg 50 mcg 125 mcg 250 mcg 500 mcg 250 mcg 50 mcg 100 mcg 200 mcg 400 mcg 400 mcg 12 mcg 20 mcg 18 mcg 4 mg 10 mg

Class Short acting β2-agonist LABA ICS

Notes

ICS LABA ICS

Also available as diskus

ICS LABA Anti-cholinergic Long-acting, 24 hrs, anticholinergic bronchodilator Leukotriene receptor antagonist (LTRA)

Also available as diskus

For Asthma and COPD For COPD

4- Emergency treatment of Asthma: ─ Oxygen, aim for SaO2 > 94% ─ Bronchodilators: o Beta 2 agonists: salbutamol 100 mcg 4 puffs q 15-20 min x 3 AND o Ipratropium bromide 4 puffs q 15-20 min x 3 ─ Corticosteroids: hydrocortisone 200 mg IV or prednisone 40-60 mg PO ─ Rehydration: aggressive IV fluids to liquefy bronchial secretions

OSCE-guide-III.doc

Page 33 of 255

History taking – Medicine

Canadian Thoracic Society – Asthma Management Continuum – 2010

Asthma action plan:

OSCE-guide-III.doc

Page 34 of 255

History taking – Medicine

COPD management Prolong survival

Smoking cessation Vaccination: influenza virus, pneumococcus (Pneumovax) Home oxygen: to prevent cor pulmonale and decrease mortality if used > 15 hrs/day (indications: PaO2 < 55 mmHg; or < 60 mmHg with cor pulmonale or polycythemia) Ventolin (q6h PRN) + Atrovent (1-2 puffs q6h)  LABA (Serevent 50 mcg/dose) ± Atrovent /or/  LACA (Spiriva): 18 mcg qAM + must stop Atrovent   

Bronchodilators Short acting Long acting Respiratory rehabilitation Corticosteroids Inhaled

Combination of ICS + LABA Advair: Fluticasone / Salmeterol (250mcg/50mcg) od or bid  Symbicort: Budesonide / Formoterol (400mcg/12mcg) 2 puffs bid Short course of oral corticosteroids: 50mg/d x 5 days 

Oral

COPDE (COPD exacerbation):  Definition: episode of increased dyspnea, coughing, increase in sputum volume or purulence  Etiology: viral URTI, bacteria, air pollution, CHF, PE, Ml must be considered  Management: ─ Assess ABC, consider assisted ventilation if decreasing LOC or poor ABGs ─ Supplemental O2 (controlled FiO2: target 88-92% SaO2 for CO2 retainers ─ Bronchodilators by nebulizer o Short acting beta2-agonists used concurrently with anti-cholinergics o Salbutamol and Ipratropium bromide via nebulizers x 3 back-to-back ─ Systemic corticosteroids: IV solumedrol or oral prednisone (50mg/d x 5 days) ─ Antibiotics: often used to treat precipitating infection: o Indications (2 out of 3): increased SOB, increased sputum amount, or increased sputum purulence (change in colour, e.g. greenish) o Type of antibiotics: see below ─ Post exacerbation: rehab with general conditioning to improve exercise tolerance  ICU admission ─ For life threatening exacerbations ─ Ventilator support o Non-invasive: NIPPY, BiPAP o Conventional mechanical ventilation Antibiotics: ─ Outpatient: resp fluoroquinolones: levofloxacin 750 mg PO q24h x 5 days OR beta-lactam + macrolide (amoxicillin 1000 mg PO tid + clarithromycin 500 mg PO bid) ─ Risk factors (group home / hospital infection / immunocompromised): ceftriaxone (1 g IV q24h) + azithromycin (500 mg IV q24h x 5 days). Step-down to oral therapy when tolerated ─ Susceptible for pseudomonas / recent use (within 3 months) of antibiotics or cortisone: piptazo (piperacillin / tazobactam); 3.375 gm IV q6h) ─ MRSA: Vancomycin 1 gm IV q24h

OSCE-guide-III.doc

Page 35 of 255

History taking – Medicine

Ankle swelling – Bilateral Introduction CC Analysis of CC

Impact Red flags Differential diagnosis

uni- vs. bi- lateral OsCfD  What ↑? Walking / standing what ↓? Raising legs  How high does it go? ↑↓ AS  Local symptoms: ─ Pain / fullness / heaviness / tightness ─ Skin changes (redness / swelling / do you feel your feet warm?) ─ Nail changes  Other swellings in your body: ─ How about swelling in your face? Eye puffiness? Do you find it difficult to open your eyes in the morning? ─ How about your belly? Did you need to ↑ the size of your belt? ─ Hands, did you feel it is tight to wear your ring?  How does this affect your life?  Constitutional symptoms – for infection / cancer Differential diagnosis of BILATERAL ankle swelling:  Failure Heart  Failure Liver  Failure Kidney: history of kidney disease (changes in urine / bruising / frequency / burning / frothy urine / clear or no)  Hypoalbuminemia  Thyroid diseases Specific cause within this system (e.g. kidney)  Hx or Dx of DM  Any medications (penicellamine, gold, NSAIDs, …)  Recent sore throat  Any skin infection / rash  Hx of autoimmune disease  How about diet? Is it balanced? Any diarrhea?

PMH FH SH Case: patient with face swelling, BP 150/90, protein in urine, ketones, no blood, no glucose, no WBCs Diagnosis: nephritic syndrome (minimal changes) Investigations: ─ Kidney function tests / urinalysis / 24 hrs protein in urine / renal biopsy ─ Lipid profile / blood glucose studies ─ Hepatitis B serology / ANA / C3 and C4 Management: ─ Salt restriction / avoid fats ─ Diuretics / monitor fluids in and out ─ Anti-HTN: ACE inhibitors ─ Prednisolone

OSCE-guide-III.doc

Page 36 of 255

History taking – Medicine

Ankle swelling – Unilateral Introduction CC Analysis of CC

Impact Red flags Differential diagnosis

uni- vs. bi- lateral OsCfD  What ↑ / ↓?  How high does it go? ↑↓  If pain  PQRST AS  Local symptoms: ─ Pain / fullness / heaviness / tightness ─ Skin changes (redness / swelling / do you feel your feet warm?) ─ Nail changes  Other joints? Toes? Other ankle?  How does this affect your life?  Constitutional symptoms – for infection / cancer Differential diagnosis of UNILATERAL ankle swelling:  Any trauma, any twist in your ankle?  Gout; previous attacks, screen kidney  for kidney stones  Infection, sepsis, cellulitis; fever, pus, discharge, tenderness  Gonorrhea septic arthritis; Sexual history, penile discharge? Unprotected sex recently?  DVT Specific cause within this system (e.g. gout)  Tell me more about your diet? Too much protein?  How about alcohol?  Medications? Pain meds (aspirin) / diuretics (furosemide, thiazides)?  Hx of cancer / chemotherapy (cytotoxic drugs) / radiation?  Family hx of gout / kidney stones?

PMH FH SH DVT: see the physical examination section

OSCE-guide-III.doc

Page 37 of 255

History taking – Medicine

Congestive heart failure – CHF 68 years old man comes to ER with 4 weeks of SOB Analysis of CC

Impact

Red flags DD

PMH FH SH

Clarification

1- When do you say SOB; what do you mean? Cardiac or chest? ─ Is it difficult to breathe in and out?  cardiac / anemia ─ Is it difficult to breathe out?  COPD / asthma 2- Do you have any hx of asthma? Lung disease? ─ Any wheezes? Chest tightness? Cough? 3- Do you have any hx of heart disease? ─ No  newly dx ─ Yes  ? acute on top of CHF ─ Any racing heart? Dizziness? LOC? Any hx of HTN?  Is it first time? Or you had it before? When and how were you OSCfD PQRST diagnosed? How about treatment? ↑↓  Is it related to activity? How many blocks were you able to walk? And now?  How about at rest? And at night?  Left ventricle: ─ SOB? How many pillows do you use? ─ Do you wake up at night gasping for air? ─ Cough / crackles?  Right ventricle: ─ Any swelling in your LL? How high does it go? Related to position / standing? Weight gain? ─ Eye puffiness? Swollen face? Pain on the liver?  Other cardiac symptoms: ─ Chest pain? Nausea/vomiting? Sweating? ─ Heart racing / dizziness / LOC? Do you feel tired?  Constitutional symptoms – for infection / cancer  Risk factors for ischemic heart diseases – IHD Causes (that precipitated acute on top of CHF):  Compliance  Diet  Medical DM / Kidney / Liver diseases HTN / heart attacks SAD

DD (Causes that precipitated acute on top of CHF): 

Compliance: Are you receiving treatment? Which medications do you take? How much? For how long? Any change in medications? Change in dose? ─ Do you take it on regular basis? Any chance that you may skip one or more doses? ─ Do you take it by yourself or do you need help? ─ Did you get your Digoxin level measured before / recently? ─ Did you start new medication? Rx or (OTC) over the counter? e.g. indomethacin ─ How about water pills? ─ Are you under regular F/U? How often? When was the last time? Were you symptoms free at that time? Diet: ─ Do you have special diet? Salt-free diet? Do you monitor that? ─ Any new changes in diet? ─ Any chance of salty food, e.g. pickles, canned food, dried meet and fish ─



OSCE-guide-III.doc

Page 38 of 255

History taking – Medicine



Medical: Do you take medications on regular basis? Any new medication? Advil? Any hx of thyroid dx, any sweating / diarrhea? Any hx of heart disease / HTN ( A Fib) / heart attack / CAD (ischemia) / did you feel your heart bouncing (arrhythmias)? Any congenital or valvular disease / Chest pain / tightness / dizziness / light headedness / LOC? ─ Any chest / lung disease (wheezes, cough, chest tightness) ─ Any kidney disease? Renal failure? ─ Any bleeding? Anemia? ─ ─ ─

New York Heart Association functional Classification of heart failure:  Class I: ordinary physical activity does not cause symptoms of HF  Class II: comfortable at rest, ordinary physical activity results in symptoms  Class III: marked limitation of ordinary activity; less than ordinary physical activity  symptoms  Class IV: inability to carry out any physical activity without discomfort; symptoms may be present at rest Investigations:  Labs: CBC / lytes / ABG (arterial blood gases) / glucose / INR / PTT / serial cardiac enzymes (q8h x 3) / ECG / fluid balance  Chest x-ray findings of CHF: (1) Enlarged heart, (2) Upper lobe vascular redistribution, (3) Kerley B lines (thin linear pulmonary opacities caused by fluid or cellular infiltration into the interstitium of the lungs), (4) Bilateral interstitial infiltrates, (5) Bilateral small effusions Treatment: ─ Acute heart failure: o Treat acute precipitating factors (e.g. ischemia. arrhythmias) o L Lasix (diuretics)  ↓ pre-load (furosemide: 40-500 mg IV) o M Morphine; 2-4 mg IV – decreases anxiety and preload (venodilation) o N Nitrates (venous and arterial dilator  ↑ kidney perfusion) o O Oxygen o P Positive airway pressure (CPAP/BiPAP) – decreases preload and need for ventilation / Position (sit patient up with legs hanging down unless hypotensive) o In ICU or failure of LMNOP: sympathomimetics (dopamine or dobutamine) ─ Chronic heart failure (long term management): o ACEI (slow progression and improve survival) or ARBs (if ACEI not tolerated) o Beta blockers: slow progression and improve survival  Should be used cautiously, titrate slowly because may initially worsen CHF  Side effects: fatigue / bradycardia  If pt on β-blockers  exacerbation  stop the β-blockers for 2 days o Digoxin (if A Fib OR symptomatic on ACEI) o Diuretics: symptom control, management of fluid overload; furosemide 80 mg OD (furosemide opposes the hyperkalemia induced by beta-blockers, ACEIs) • Spironolactone for class Ill-b and IV CHF already on ACEI and loop diuretic • If still uncompensated: Implantable Cardioverter Defibrillator (ICD) o Anti-arrhythmic drugs: for use in CHF with arrhythmia can use amiodarone, betablocker, or digoxin o Anticoagulants: warfarin for prevention of thromboembolic events Digoxin overdose: ─ Anorexia, nausea, vomiting ─ Bradycardia, dizziness, LOC ─ ECG: PVC, heart block ─ Vision: yellow hallos around objects

OSCE-guide-III.doc

Page 39 of 255

History taking – Medicine

Heart racing For few weeks Introduction … But first I would like to ask you, at the moment, how do you feel? CC Analysis of Clarification When do you say your heart is racing; what do you mean? ─ Do you feel your heart is going fast CC ─ Or is skipping beats ─ Can you tap it for me please? … ─ It sounds irregular for me!  Is it first time? Or you had it before? When and how were you OSCfD PQRST diagnosed? How about treatment?  Is it related to activity? How many blocks were you able to ↑↓ walk? And now?  How about at rest? And at night?  When was the last attack? And what is the duration of the longest attack?  Is it related to caffeine, chocolate, coke, any other type of food?  SAD (cocaine or any other stimulant) Impact  CVA (any weakness / numbness / difficulty finding words / visual problems)  Heart failure (SOB / limitation of activity / swelling in your legs / how many pillows do you use??  Other cardiac symptoms: ─ Chest pain? Nausea/vomiting? Sweating? ─ Heart racing / dizziness / LOC? Do you feel tired? Red flags  Constitutional symptoms – for infection / cancer  Risk factors for ischemic heart diseases – IHD DD  Do you take medications on regular basis? Any new medication?  Any hx of thyroid dx, any sweating / diarrhea?  Any hx of heart disease / HTN ( A Fib) / heart attack / CAD (ischemia) / did you feel your heart bouncing (arrhythmias)? Any congenital or valvular disease / Chest pain / tightness / dizziness / light headedness / LOC?  Any chest / lung disease (wheezes, cough, chest tightness)  Any kidney disease? Renal failure?  Any bleeding? Anemia? PMH DM / Kidney / Liver diseases FH  Family history of sudden death at a young age? (cardiomyopathy)  HTN / heart attacks SH SAD Physical  Vitals examination  Cardiac exam (looking for mid-diastolic, mitral stenosis, rumbling character)  Thyroid exam  Neurological exam: brief / gross motor and reflexes

OSCE-guide-III.doc

Page 40 of 255

History taking – Medicine

The patient daughter has a concern: my mother was diagnosed with AF, Should I worry about this? 1.

2.

This is a reasonable concern?  AF may lead to embolic event (CVA)  AF may lead to heart failure  AF may lead to V. Fib However, this is not uncommon condition, and it is treatable with medications

Causes for AF: 1- IHD 2- Hypertension / CHF 3- Hyper-thyroidism 4- Medications (e.g. digoxin, some antiarrhythmic meds – class I) 5- Electrolytes imbalance 6- Too much alcohol (holiday heart) 7- Cardiomyopathy 8- Valvular heart diseases 9- Congenital heart diseases 10- Loan AF 11- Myocarditis 12- COPD / pneumonia 13- Cardiac surgery 14- Cocaine 15- Any condition that lead to tachycardia in a susceptible person

Causes for TACHYCARDIA 1- Exercise 2- Pregnancy 3- Caffeine / stimulants 4- Anemia 5- Hypovolemia 6- Fever 7- Stress 8- Smoking 9- Hyper-thyroidism 10- Pheochromocytoma

Atrial fibrillation Stable < 48 hours If in doubt  TEE

> 48 hours  Rate control: ─ β-blockers ─ Ca ch blockers (diltiazem, verapamil) ─ Digoxin (in patients with heart failure)  Anti-coagulation (3 weeks prior and 4 weeks after cardioversion)

Unstable    

SOB BP < 90/60 Chest pain Confusion

Cardioversion:  Electrical: 150 joules for A Fib (50 joules for A Flutter)  Pharmacological: procainamide; 1 g / 1 hr infusion Anti-coagulation: Assess stroke risk: determine CHADS2 score in patients with non-valvular AF Risk factor Points CHADS2 score Anti-coagulation CHF 1 0-1 Aspirin 81-325 mg daily Hypertension 1 ≥ 2 moderate risk Warfarin factors or any high risk Age > 75 yrs 1 factor (prior stroke, Diabetes 1 TIA or embolism, Stroke / TIA 2 mitral stenosis, prosthetic valve)

OSCE-guide-III.doc

Page 41 of 255

History taking – Medicine

Fall Orthostatic hypotension 76 years old male patient came to clinic because he fell few days ago. He was getting out of bed, when he fell to the ground Introduction HPI: analysis of CC Associated Symptoms

For any Fall, LOC or seizure

During

Before

After

Impact Red flags

DD

OSCE-guide-III.doc

Did you hurt yourself? How do you feel? Was this the first time, or did it happen before? Was it related to Emotions? Coughing? Urination? Did you lose conscious? Did you hit your head? Were you alone or with someone? Did your wife describe it to you? Is she with you? If it is ok with you, after we finish, I would like to speak with her to get some info.  Were you shaking? Certain part of your body or whole?  Were you breathing? Did you turn blue?  Did you bite your tongue? Roll your eyes? Wet yourself?  Were you able to take few steps or did you fall immediately (orthostatic hypotension)?  Before you lose consciousness, did you feel: ─ Dizzy, lightheaded, nausea, vomiting, any chest pain, heart racing (cardiac) ─ Things are spinning around you (vertigo) ─ Weakness, numbness, vision changes (stroke) ─ Any flashing light, strange smells (epilepsy) ─ Sweaty, shaky, hungry (hypoglycemia)  How long did it last?  How did you regain consciousness? By yourself or did you need intervention?  After you regain consciousness; were you able to recognize the surroundings? Able to talk? Able to move?  Did you feel any weakness, numbness?  Did you hurt yourself? How do you feel now?  Constitutional symptoms  Risk factor for IHD Any geriatric  Balance patient; ASK about:  Vision  Hearing  Urination  Diseases: arrhythmia / CAD, CVA, seizure, hypoglycemia (already analyzed – before the event)  Environment: is your room well lit? Any chance you tripped?  Do you take any medications? Do you have a list? ─ Go through it one by one ─ Which one was added / changed recently? ─ Each medication: ask about the disease, when started? ─ Any OTC? Aspirin? Who prescribed it to you?  Are you getting enough fluids     

Page 42 of 255

History taking – Medicine

Counselling:  Inform the patient ─ The most likely explanation to what happened is a condition called “postural orthostatic hypotension”. It means drop in the blood pressure with change of posture. ─ Explain the pathophysiology: o When we change position from lying or sitting to standing, blood tends to pool in the lower extremities, and this leads to drop in blood pressure. Normally, blood vessels in our body react by narrowing in order to prevent this and to maintain normal blood pressure. o In patients having orthostatic hypotension, and this could be due to age / medications / DM or combination, their blood vessels fail to react fast enough, this leads to pooling of blood in lower extremities  ↓ amount of blood reaching to heart  ↓ blood reaching the brain  they end-up losing their consciousness temporarily. ─ Consequences: this might happen again ─ Investigations: o Blood works / CBC / differential / lytes / kidney and liver function tests o ECG  Preventive measure: ─ Contact the psychiatrist to check the poly-pharmacy, to discuss with him the possibility of decreasing the dose or changing medications. ─ Meanwhile, if you are changing positions, do this slowly, on steps, e.g. from lying down, sit for a couple of minutes on the bed before standing up, and before you stand up, push your feet against the ground for few seconds.  I will give you brochures and web sites in case you need to read more.

Notes:  The patient will have a list of medications: ─ Lipitor ─ Hydrochlorothiazide  ask about fluids intake ─ β-blocker ─ Aspirin  ask about bleeding ─ Lorazepam ─ Oxazepam  I can see that you are taking 2 sleeping pills, who prescribed them to you? The same doctor or no? ─ Metformin ─ B12 / B complex 

If the patient looks sad / depressed  you look down for me, any chance you are depressed

OSCE-guide-III.doc

Page 43 of 255

History taking – Medicine

Peripheral vascular disease: Calf pain / swelling Introduction Analysis of CC

Impact

Red flags

DD

PMH FH SH

OsCfD PQRST ─ P: unilateral or bilateral ─ R: what about other joints, knees? Thighs? Feet?  What ↑ or ↓: did you notice that your pain ↑ while walking up or down hill? ─ ↑ while walking uphill: peripheral arterial disease ─ ↑ while walking downhill: spinal stenosis  Is it first time? Or you had it before? When and how were you diagnosed? How about treatment?  Is it related to activity? How many blocks were you able to walk? And now?  How about at rest? And at night?  When was the last attack? And what is the duration of the longest attack?  History of strokes / TIAs / neurological symptoms  Chest pain / SOB / heart racing  Pain after eating (intestinal ischemia)  Effect of pain on daily activities / work?  Leriche syndrome (aorto-iliac occlusive disease): numbness in buttocks & thighs / absent or decreased femoral pulses / impotence  Constitutional symptoms – for infection / cancer  Risk factors for ischemic heart diseases – IHD ─ Smoking? How much and for how long? ─ High blood pressure? For how long? Controlled or not? ─ Diabetes mellitus ─ Cholesterol measured? When? What was it? Peripheral Arterial Disease versus Spinal Canal Stenosis Vascular symptoms Neuro symptoms  Cold feet / ulcers  Weakness / numbness / tingling  Swelling / redness  Back trauma / back pain  Delayed wound healing  Sexual dysfunction / difficulty with  Nail changes / hair loss erection Past history of heart disease / stroke / symptoms of stroke / DM / Kidney / Liver diseases Family history of heart disease / HTN / heart attacks SAD  

N.B. the 6 Ps of ischemia: Pallor / Pain / Parathesia / Paralysis / Pulseless / Polar (cold)

OSCE-guide-III.doc

Page 44 of 255

History taking – Medicine

Urinary symptoms: 

Obstructive (prostatic disease in ♂  anuria): Difficulty to initiate urine? Do you need to strain? ─ Any changes in the stream? ─ Any dripping? ─ After you pass urine, do you feel that you emptied your bladder completely or do you need to go again? Irritative (frequency  UB disease): ─ How many times do you go to the washroom? o How about before? Any change? o How about during night time? How does this affect your sleep? How about your concentration and mood? ─ Do you need to rush to washroom? Are you able to make it all the time? ─ Have you ever lost control or wet yourself? ─ Any burning sensation? Any flank pain? ─ Fever / night sweats / other constitutional symptoms Urine analysis (changes): ─ COCA + B (content: frothy / cloudy / not clear) ─





Summary of irritative symptoms: FUND Frequency / Urgency / Night time / Discomfort

OSCE-guide-III.doc

Page 45 of 255

History taking – Medicine

Anuria Introduction Analysis of CC

Impact

Red flags

DD

PMH FH SH

Empathy – how do you feel right now?  OsCfD  PQRST  What ↑ or ↓  Is it first time? Or did it happen before? When and how were you diagnosed? How about treatment? Associated symptoms:  Obstructive symptoms  Irritative symptoms  Urine analysis (changes): COCA ± Blood Local symptoms:  Any problems with passing stools? What? When?  Any masses in the groin / pelvic mass / pain?  Abdominal pain? Distension? Metastasis  Back: pain / weakness / numbness  Liver: yellow / itchy / urine / stools  Lungs: cough / phlegm / hemoptysis  Brain: headache / nausea / vomiting Renal failure Generalized swelling / face puffiness / itching Sexual Sexual dysfunction  Constitutional symptoms – for infection / cancer  Risk factors for cancer prostate / bladder ─ Were you ever diagnosed with prostate disease? Screened for prostate diseases? (DRE or PSA) ─ Family history of prostate disease / cancer? ─ Ca bladder (radiation / exposed to chemicals / aniline dye) ─ Smoking? Alcohol?  Renal stones: Have you ever had a renal stone? Any history of colicky pain in flanks? Have you ever passed a small crystals or stone during voiding? Hx or repeated UTIs?  Medications: glaucoma / anti-psychotic meds / anti-cholinergic drugs; like those used for incontinence; e.g. Ditropan (Oxybutynin), Detrol (Tolterodine)  2 Neuro: ─ Back problem: trauma – metastasis – cauda equine (spoiled himself with stools / buttocks numbness) ─ Stroke (diagnosed / weakness / numbness / difficulty)  2 Cancer: ─ Cancer prostate ─ Ca bladder (hematuria) AMPLE DM / anemia / polycystic kidney disease / renal stones SAD

Most likely diagnosis: BPH Other possible diagnoses: UTI / prostatitis / Ca prostate Investigations: urea & creatinine / urinalysis / renal US / DRE & PSA / TRUS If cancer is suspected: bone scan / CT Treatment: ─ Watchful waiting: may resolve spontaneously ─ Medical treatment: α-adrenergic antagonists (doxazosin, terazosin) / 5-α-reductase inhibitors (finasteride) ─ Surgery: open surgery / TURP / minimally invasive (stent / laser ablation / cryosurgery)

OSCE-guide-III.doc

Page 46 of 255

History taking – Medicine

Hematuria Introduction Analysis of CC

Impact

Red flags

DD

PMH FH SH Investigations:

Empathy – how do you feel right now?  OsCfD  Timing: ─ Initial versus terminal or total ─ Diurnal variation  What ↑ or ↓  Painful or Painless  Is it first time? Or did it happen before? When and how were you diagnosed? How about treatment? Associated symptoms:  Obstructive symptoms  prostate disease  Irritative symptoms  UB disease  Urine analysis (changes): COCA ± Blood Local symptoms:  Any problems with passing stools? What? When?  Any masses in the groin / pelvic mass / pain?  Abdominal pain? Distension? Metastasis Renal failure Generalized swelling / face puffiness / itching Sexual Sexual dysfunction  Constitutional symptoms – for infection / cancer  Risk factors for cancer prostate / bladder / RENAL ─ Were you ever diagnosed with prostate disease? Screened for prostate diseases? (DRE or PSA) ─ Family history of prostate disease / cancer? ─ Family history of cancer bladder or kidney? ─ Ca bladder (radiation / exposed to chemicals / aniline dye) ─ Smoking? Alcohol?  Renal stones: Have you ever had a renal stone? Any history of colicky pain in flanks? Have you ever passed a small crystals or stone during voiding? Hx or repeated UTIs?  Medications: blood thinners / aspirin / bleeding from other sites?  Pseudo-hematuria: ─ Diet: eating too much beet ─ Medications: Rifampicin ─ Other bleeding: bleeding per rectum / vaginal bleeding  AMPLE  History of hemolytic anemia / polycystic kidney DM / anemia / polycystic kidney disease / renal stones SAD

(1) Kidney: urinalysis (casts / crystals / C&S / cytology) / ultrasound (abd/pelvic) / IVP / KFTs (2) Bladder: cystoscopy (3) Prostate: PSA / TRUS (4) Others: CBC / differential / INR Case: patient on warfarin for A. fib for 2 yrs; went to walk in clinic for sore throat and was prescribed Biaxin, developed hematuria. Diagnosis: coagulopathy.

OSCE-guide-III.doc

Page 47 of 255

History taking – Medicine

Renal stones Risk Factors ─ Hereditary: RTA, G6PD, cystinuria, xanthinuria, oxaluria, etc. ─ Dietary excess: Vitamin C, oxalate, purines, calcium ─ Dehydration (especially in summer months) ─ Sedentary lifestyle ─ Medications: thiazides ─ UTI (with urea-splitting organisms) ─ Hypercalcemia disorders: hyperparathyroidism, sarcoidosis, histoplasmosis, etc. Investigations ─ Screening labs o CBC: elevated WBC in presence of fever suggests infection o Electrolytes, Cr, BUN  to assess renal function o Urinalysis: R&M (WBCs, RBCs, crystals), C&S ─ Imaging o Kidneys, ureters, bladders (KUB) x-ray to differentiate opaque from non-opaque stones (e.g. uric acid, indinavir) / 90% of stones are radiopaque o CT scan: no contrast; distinguish radiolucent stone from soft tissue filling defect o Abdominal ultrasound: may demonstrate stone (difficult for ureters) / may demonstrate hydronephrosis o IVP (not usually done): anatomy of urine collecting system, degree of obstruction, extravasation ─ Cystoscopy for suspected bladder stone ─ Strain all urine  stone analysis ─ If recurrent stone formers, conduct metabolic studies o Serum electrolytes, Ca, PO4, uric acid, creatinine and urea o PTH if hypercalcemic Approach to renal stone:

OSCE-guide-III.doc

Page 48 of 255

History taking – Medicine

Treatment – Acute: ─ Medical: o Analgesics (Tylenol #3) o NSAIDs help lower intra-ureteral pressure o ± antibiotics for UTI o ± (antiemetic + IV fluids) for vomiting ─ Interventional: o Ureteric stent (cystoscopy) o Percutaneous nephrostomy (image-guided) ─ Admit if necessary: o Intractable pain o Intractable vomiting o Fever (? infection) o Compromised renal function o Single kidney with ureteric obstruction / bilateral obstructing stones Treatment – Elective: ─ Medical: o Conservative if stone < 5 mm and no complications o Fluids to increase urine volume to > 2 L/day (3-4 L if cystine) o Specific to stone type:  Calcium oxalate stones: thiazides / potassium citrate (alkalinization of urine)  Calcium struvite: antibiotics for 6 wks (stone must be removed to treat infection)  Uric acid: allopurinol / potassium citrate (alkalinization of urine to pH 6.5 to 7) / shockwave lithotripsy not effective  Cystine: alkalinize urine (bicarbonate / potassium citrate) / penicellamine / captopril (forms complex with cystine) / shockwave lithotripsy not effective ─ Interventional: o Procedural / surgical: If stone is > 5 mm or presence of complication o Kidney  Extracorporeal shockwave lithotripsy (ESWL) if stone < 2.5 cm  Percutaneous nephrolithotomy; indications: + Size > 2.5 cm + Staghorn + UPJ obstruction + Calyceal diverticulum + Cystine stones o Ureter  ESWL is the primary modality of treatment  Ureteroscopy (extraction or fragmentation) if failed ESWL / Ureteric stricture o Bladder  Transurethral cystolitholapaxy  Remove outflow obstruction (TURP or stricture dilatation} Management of UTI: ─ Investigations: o Urine for culture and sensitivity o Blood: CBC / differential o Imaging (if suspect complicated pyelonephritis or symptoms do not improve with 72 hours of treatment): Abd/pelvic U/S / IVP / Cystoscopy / CT ─ Pregnant: amoxicillin 500 mg TID x 7 days ─ Non-pregnant: o Septra (sulfamethoxazole and trimethoprim) DS (800/160): 1 tab bid x 7 days o /OR/ Ciprofloxacin 500 mg bid x 7 days ─ Pyelonephritis: o Ceftriaxone (third-generation cephalosporins): 1 g IV q24hrs x 2 days o Then continue oral ciprofloxacin x 7 days ─ Abscess: + drain

OSCE-guide-III.doc

Page 49 of 255

History taking – Medicine

Incontinence Obstructive / 62 years old female, with hx of 3 years of urinary incontinence Introduction Empathy – how do you feel right now? Analysis of  OsCfD CC  What ↑ or ↓: lifting objects / coughing / straining  Is it first time? Or did it happen before? When and how were you diagnosed? How about treatment?

Impact Red flags

DD

Associated symptoms: If at any time there is a frequency or  Obstructive symptoms some new symptom  analyze it  Irritative symptoms first then resume!  Urine analysis (changes): COCA ± Frequency in ♀  UTI Blood Local symptoms:  Any problems with passing stools? What? When?  Any masses in the groin / pelvic mass / pain?  Any perineal skin lesions?  How does it affect your life? Daily activities?  Constitutional symptoms – for infection / cancer  Risk factors (MGOS):  Menopausal symptoms, and HRT M  LMP  Gynaecological history G  Previous abdominal or pelvic surgeries  Obstetric: How many pregnancies? Route of delivery? O  Sexual: Repeated infections / dryness / dyspareunia S  Overflow incontinence  Urge incontinence  Detrusor overactivity: CNS lesion, inflammation / infection (cystitis), bladder neck obstruction (tumour, stone)  Stress incontinence  Urethral hypermobility: childbirth, pelvic surgery, aging  Intrinsic sphincter deficiency (ISD): pelvic surgery, neurologic problem, aging and hypoestrogen state

Diagnosis: ─ History ─ Urinalysis + C&S (if infection suspected) ─ Urodynamics ─ Stress test Treatment of urge incontinence Treatment of stress incontinence ─ Bladder habit training ─ Weight loss ─ Botox (botulinum toxin) injection ─ Kegel’s exercises ─ Medications: anti-cholinergics; ─ Bulking agents Tolterodine (Detrol), Oxybutynin ─ Surgery (slings, TVT / TOT4, artificial (Ditropan), TCAs sphincters) N.B. Causes of reversible urinary incontinence (DIAPERS): Delirium, Inflammation / Infection, Atrophic vaginitis, Pharmaceuticals / Psychological, Excess urine output, Restricted mobility, Stool impaction 4

TVT: Tension-free Vaginal Tape, TOT: Trans Obturator Tape

OSCE-guide-III.doc

Page 50 of 255

History taking – Medicine

Lump – Neck Swelling Introduction Analysis of CC: The lump

Associated (local) symptoms

Impact

Red flags

Differential Diagnosis PMH FH Physical exam

Can you point to it? OSCfD / Anything special at that time? Fever? Rash? Is it painful? PQRST Can you estimate its size for me? Is it like a lent, olive, lemon, or larger? Did it change in size? How fast was the change in size?  Did you try to feel it? Does it feel soft / rubbery / hard?  Do you feel it is fixed or moving?  Any skin changes? Redness? Ulcers?  Any history of trauma?  Is it the only one? Did you notice other lumps in your body? How about other side of your neck? Arm pits? Groins?  Rule out infection: Any recent flu-like symptoms? Do you feel tired/ fatigue? History of sinusitis/ Pain in your face? Runny nose? Pain/discharge in ears? Any sore throat/ oral ulcers/ tooth pain? Difficulty swallowing? Neck stiffness/pain? Headache? Vomiting?  Thyroid (if central): heat vs. cold intolerance / sweating / hand shaking / heart racing / diarrhea vs. constipation  How does this affect your life?  Do you feel tired? ? HIV  Easy bruising? Repeated infections? ? Leukemia  Constitutional symptoms  Bone pains / Tender points HEAD SSS  risky behaviour:  A: includes recent travel  SAD: how about injection drugs? Did you share needles?  Sexual hx: Detailed (safe sex, last time, how many partners). Did you notice any vaginal discharge/ bleeding? Any pain/ blisters/ warts? Discoloration/ itchiness? HIV / Lymphoma / Leukemia / Infectious mono-nucleosis    

History of cancer History of cancer / lymphadenopathy  Vital signs  Neck exam / Thyroid exam if the swelling is central  LNs / Lymphatic system / LNs in groin / pelvic exam  Liver / Spleen

Notes: ─ ─

Whenever there is IV drugs  screen for liver symptoms / HIV Whenever there is risk for STIs  screen for liver symptoms and PID

OSCE-guide-III.doc

Page 51 of 255

History taking – Medicine

Lump – Breast Introduction Analysis of CC: The lump

Can you point to it? Is it one breast or both? Where did you notice it? You can ask verbally, is it LT / RT? Upper / Lower? Outer / Inner? How about the other breast?

 

DO NOT POINT WITH YOUR HANDS OR FINGERS! OSCfD / Anything special at that time? Fever? Rash? Is it painful? PQRST Can you estimate its size for me? Is it like a lent, olive, lemon, or larger? Did it change in size? How fast was the change in size?  Did you try to feel it? Does it feel soft / rubbery / hard?  Do you feel it is fixed or sliding (moving)?  Any skin changes? Redness? Ulcers?  Any history of trauma?  Is it the only one? Did you notice other lumps in your body? How about your neck? Arm pits? Groins?  Is it related to your period? Does it change with the period?  Any nipple changes? Discharge? Bleeding? Itching? Associated (local)  Rule out infection: Any recent flu-like symptoms? Do you feel tired/ fatigue? symptoms Impact  Headache/ vomiting? (consequences of  Back pain/ weakness/ numbness/ tingling in arms or legs? cancer:  Chest pain/ cough/ phlegm/ wheezes/ heart racing? metastasis)  Liver: yellow discoloration/ itching/ urine/ stools? Red flags  Constitutional symptoms  Bone pains / Tender points Risk factors of cancer: MGO  Menstrual history: first period / last period / regular?  G: OCPs?  Obstetric: History of pregnancies? Number of pregnancies? First pregnancy at what age?  Breast feeding?  Diet rich in fat  PMH or FH of cancer breast / ovarian carcinoma Differential  Benign disease Diagnosis  Trauma  fat necrosis PMH History of cancer breast / ovarian carcinoma FH History of cancer breast / ovarian carcinoma DD for Breast Mass: ─ Breast Cancer ─ Sclerosing adenosis ─ Fibrocystic changes ─ Lipoma ─ Fibroadenoma ─ Neurofibroma ─ Fat necrosis ─ Granulomatous mastitis (e.g. TB, ─ Papilloma / papillomatosis sarcoidosis) ─ Galactocele ─ Abscess ─ Duct ectasia ─ Silicon implant ─ Ductal / lobular hyperplasia   

OSCE-guide-III.doc

Page 52 of 255

History taking – Medicine







Galactorrhea (prolactinoma): normal prolactin level: 10-20 ng/ml (non-pregnant)  If < 40 ng/ml: follow-up  If > 40 ng/ml: CT and bromocriptine  If progressive ↑ / headache / affecting vision  surgery Benign Breast Lesions: ─ Non-proliferative lesions: o Aka fibrocystic changes, chronic cystic mastitis, mammary dysplasia  Benign condition characterized by fibrous and cystic changes in the breast. No ↑ risk of breast cancer / Age 30 to menopause / ↑ pre-menstrual.  Breast pain, focal areas of nodularity or cysts often in upper outer quadrant  Treatment: Evaluation of breast mass and reassurance / Analgesia (ibuprofen, ASA) / If > 40 years old: mammography every 3 years or biopsy ─ Proliferative lesions: o Fibroadenoma:  Most common benign breast tumour in women under age 30  Risk of subsequent breast cancer is increased only if fibroadenoma is complex, there is adjacent atypia or a strong family history of breast cancer  Clinical features: nodules: smooth, rubbery, discrete, well-circumscribed, nontender, mobile, hormone-dependent. Needle aspiration yields no fluid  Investigations: Core or excisional biopsy required  Treatment: Generally conservative; serial observation. Consider excision if size 2-3 cm, rapidly growing on serial US, if symptomatic or pt preference o Intra-ductal Papilloma  Solitary intra-ductal benign polyp  Present as nipple discharge (most common cause of spontaneous, unilateral bloody nipple discharge), breast mass, nodule on U/S  Treatment: excision of involved duct to ensure no atypia ─ Other lesions: o Fat Necrosis  Uncommon, result of trauma (may be minor – commonly a tight bra, positive history in only 50%), after breast surgery (i.e. reduction)  Firm, ill-defined mass with skin or nipple retraction, ± tenderness  Regress spontaneously, but complete imaging ± biopsy to R/O cancer o Mammary Duct Ectasia  Obstruction of a subareolar duct  duct dilation, inflammation, and fibrosis  May present with nipple discharge, bluish mass under nipple, local pain  Risk of secondary infection (abscess, mastitis)  Resolves spontaneously o Abscess  Lactational vs. periductal / subareolar  Unilateral localized pain, tenderness, erythema, subareolar mass, nipple discharge, nipple inversion  Rule out inflammatory carcinoma, as indicated  Treatment: initially broad-spectrum antibiotics and I&D (incision and drainage), if persistent total duct excision (definitive)  If mass does not resolve: fine needle aspiration (FNA) to exclude cancer, U/S to assess for presence of abscess Breast Cancer: ─ 1/9 women in Canada will be diagnosed with breast cancer in their lifetime ─ Risk factors:  Prior history of breast cancer  1st degree relative with breast cancer (greater risk if relative was premenopausal)  Increased risk with high breast density, nulliparity, first pregnancy >30 years old, early menarche (< 12 yrs), late menopause (> 55 yrs), >5 years HRT  Decreased risk with lactation, early menopause, early childbirth

OSCE-guide-III.doc

Page 53 of 255

History taking – Medicine











Investigations o Mammography  Screening: every 1-2 years for women age 50-69 / If positive family history in 1st degree relative: every 1-2 years starting 10 years before the youngest age of presentation  Diagnostic: investigation of patient complaints (discharge, pain, lump)  Follow-up after breast cancer surgery  Findings indicative of malignancy: mass that is poorly defined, spiculated border, micro-calcifications, architectural distortion, normal mammogram does not rule out suspicion of cancer based on clinical findings o Other radiographic studies:  Ultrasound – differentiates between cystic and solid  MRI – high sensitivity, low specificity  Galactogram / ductogram (for nipple discharge): identifies lesions in ducts  Metastatic workup as indicated (usually after surgery or if clinical suspicion of metastatic disease) – bone scan, abd U/S, CXR, head CT Diagnostic Procedures o Needle aspiration: for palpable cystic lesions; send fluid for cytology if blood or cyst does not completely resolve o Fine needle aspiration (FNA): for palpable solid masses; need experienced practitioner for adequate sampling o U/S or mammography guided core needle biopsy (most common) o Excisional biopsy: only performed as second choice to core needle biopsy; should not be done for diagnosis if possible Genetic Screening: consider testing for BRCA 1/2 if: o Patient diagnosed with breast AND ovarian cancer o Strong family history of breast / ovarian cancer (e.g. Ashkenazi Jewish) o Family history of male breast cancer o Young patient ( 7 mmol/L Random blood sugar (RBS) > 10 mmol/L + symptoms Glucose tolerance test (GTT) > 11.1 mmol/L What is pre-diabetes? Impaired glucose tolerance o Fasting blood sugar (FBS) 6.1 – 6..9 mmol/L o Glucose tolerance test (GTT) 7.8 – 11 mmol/L o o o

Introduction





Impact

Red flags

Pre-diabetes: does not mean that you have diabetes, but it shows that you have an increased chance of having it, about 1–5% per year. It also shows increased risk of you having complications in the large blood vessels causing heart diseases, strokes and peripheral vessel diseases Diabetes: ─ Increase of blood sugar in our blood due to deficient or ineffective insulin. ─ Explain the role of insulin in helping cells to utilize glucose, two types of DM, type I and type II. ─ With one reading we can not say that you are prone or have DM, so let me ask few questions, to see if you have the symptoms of DM!

 Symptoms of hyperglycemia

   

Eat more, drink more, pee more even at night Blurred vision Tired / weight loss Yeast infections, are there itching / rashes in your groins, in the toes and finger webs?  Do your wounds get long time to heal? N/V, abdominal pain, dehydration, LOC

Symptoms of Ketoacidosis Symptoms of If patient is on insulin: sweating, shaking, palpitation, fatigue, hypoglycaemia headache, confusion, seizures Complications of  Micro-vascular: nephropathy / neuropathy / retinopathy high blood sugar  Macro-vascular: CAD / peripheral arterial dis / impotence  Lifestyle: too much simple sugars, lack of exercises, overweight, family history  Medications: steroids / beta blockers (β-blockers are contraindicated in DM: it causes hyperglycemia / and it masks hypoglycemia)

PMH

Medications: used long term steroids, thiazides, phenytoin, clozapine or other antipsychotics, HTN, Cholesterol, CAD, CVD, kidney, hospitalization FH DM in first degree relatives SH  Sexual function: any concerns  Smoking  From the conversation we had, it looks like you are likely to get DM. However I am going to examine you and do blood tests (FBS, Hb A1C – which shows your blood sugar level over the past 3 months, lipid profile, micro albumin / Cr ratio, ECG).  I strongly recommend you to work on lowering your chance of having diabetes by half by: watching your diet (healthy balanced diet, avoid saturated fats and simple sugars, choose low glycemic content foods), exercising (30 -45 min of moderate exercise for 4-5 days/wk) and life style changes (limit Na, alcohol, caffeine, stop smoking).  I can refer you to diabetes educational program if you wish.  Treatment targets: Hb A1C < 7 FBS 4 – 6 Lipids: LDL < 2, Triglycerides < 1.5 or TC/HDL < 4 BP < 130/80

OSCE-guide-III.doc

Page 74 of 255

Emergency Medicine

Emergency Medicine

OSCE-guide-III.doc

Page 75 of 255

Emergency Medicine

Emergency Room Trauma ATLS: Advanced Trauma Life Support protocol Primary survey: - Any trauma (kicked, car accident, thrown from height, …) - Manage over the phone: trauma or meningitis - Secondary survey: patient in the ER after car accident, primary survey was done, do the secondary survey N.B. if knife: leave it in place, fix with gauze

Non-trauma Cardiac Chest pain: - MONA ± β-blockers - STEMI: catheterization, thrombolytics - No ST elevation: heparin, angiography Arrhythmias: - V Fib - V Tachy ACLS: Advanced Cardiac Life Support code Heart block: old patient - DNR - Advanced directive

Non cardiac -

Acute abdomen for 24 hrs, ↓BP, ↑HR Upper / lower GIT bleeding Severe headache for 2 hours Seizure for the last 20 minutes 16 years brought unconscious to the ER Patient receiving blood, counsel him

Management: Trauma I A B C D AMPLE Head to toe Management

OSCE-guide-III.doc

Medical I A B C D OCD PQRST ↑↓ Associated symptoms Risk factors PMH Focused physical exam Management

Page 76 of 255

Emergency Medicine

Trauma I: introduction: - Because it is a trauma case, I would like to activate the ATLS protocol - I would like also to get protection for me and my team; gloves, gowns, goggles and masks -

Mr … I am Dr …, the physician in charge in the emergency room; o If the patient is wearing a neck collar, proceed o Pt is not wearing a neck collar: Mr … please do not move your head, nurse can you please fix his head, we need to put neck collar

-

I understand that you are here because you had a car accident How are you feeling / doing right now? o I would like to make sure that you are stable, I will check with the nurse and we will start the management then I will be asking you more questions. o I can see that you are in a lot of pain, please bear with me for few minutes, and I will give pain killer as soon as I can. o Doctor, where is my wife? How is she doing? Was she with you? I can see that you are concerned about your wife, I will look for her and I will get back to you as soon as I can, meanwhile my first concern is to make sure you are stable

We will start the primary survey now: Airways / Breathing Mouth / O2 / inspect chest, neck / listen to lungs, heart Can you please open your mouth? Nurse, what is O2 saturation, plz?

Inspect the chest

Open the collar window, or fix pt head and remove anterior part: ─ Trachea ─ Jugular veins (JV) Listen to lungs Listen to heart sounds

OSCE-guide-III.doc

Mouth is clear; no FB, no dentures, no vomitus Pt is talking to me that means airways are patent Can you give him O2 – 4 L with a nasal canula Any change in saturation? Can you plz let me know if any change in saturation happens! By inspection, chest is symmetrical, no bruises, no open wounds, no paradoxical movements of the chest, no use of accessory muscles for breathing Trachea is central, JV not engorged, bilateral air entry, normal heart sounds (HS) S1 and S2 Trachea JV Air HS Diagnosis entry shifted Engorged ↓ same normal tension away side pneumothorax shifted depleted ↓ same normal Hemosame side thorax side central engorged bilatera muffle cardiac l d temponad e Usually no cardiac temponade in the exam

Page 77 of 255

Emergency Medicine

+ If ↓↓ BP and ↑↑ HR / other signs of tension pneumothorax  nurse, I need to put a large needle (16 / 14 G) in the 2nd intercostal space at MCL (upper border of the 3rd rib); ─ Is there any gush of air? ─ Check the trachea centrality and air entry ─ We need to put a chest tube in the 5th intercostal space + If ↓ BP and ↑ HR / other signs of hemothorax  nurse, I need to put chest tube in the 5th intercostal space at anterior Axillary line; ─ What is the amount of blood? ─ If > 1.5 L  stat surgery ─ Otherwise, monitor; if > 200 ml/hr  surgery Circulation Vital signs / fluids / withdraw blood samples / look for source of bleeding Can I get the vital signs please

Comment, patient is … hypo- / hyper- / tension, comment on HR, pt is stable / unstable I would like to have two large IV lines, 16 G in both anti-cubital fossae: ─ One to start fluids: bolus 2 L ringer lactate or normal saline ─ The other line is to withdraw samples for: CBC/differential/lytes /+/ blood grouping and cross matching / and prepare 6 units of blood (4 matched and 2 “O”) /+/ stat glucose /+/ INR/PTT/LFT /+/ Bun/creatinine /+/ toxic screen/alcohol level /+/ continuous cardiac monitoring/cardiac enzymes and ECG Can you please inform me with the vitals; after the bolus fluid is done and every 5-10 minutes or if there is a change in the vitals Look for the source of bleeding Abdomen:  Inspect the abdomen  bruises I am going to look at and feel your  Palpate the abdomen  rigidity and guarding abdomen  If positive; I am suspecting intra-abdominal bleeding, I would like: ─ To get stat surgery consult ─ To arrange for FAST (focused abdominal sonogram for trauma) ─ To do DPL (peritoneal lavage) I am going to press on your pelvis  Press from the sides  Press open book  If positive; I am suspecting pelvic fracture: ─ Cut pt sheet and wrap around the pelvis to support, and check blood on penile meatus ─ Stat orthopedics consult Lower extremities By inspection, patient lower extremities are symmetrical, no abnormal posture or deformity. No inequality in length, no pain, no deviation  If positive: I am suspecting femur fracture; ─ Check the pulses ─ Thomas splint ─ Stat orthopedics consult Log rolling I need more team members to roll the patient on his left side:  To check for external source of bleeding  To press on the spinal processes  To perform digital rectal exam

OSCE-guide-III.doc

Page 78 of 255

Emergency Medicine

I would like to get trauma X-ray series: for neck, chest, LSS and pelvis D: D1: Deficits / Disability D2: Detoxification Neuro screen / I am going to shine light in your eyes? Can you please squeeze my fingers, do not let them go Can you wiggle your toes? Do you feel me touching you here, here, and here Glasgow coma scale – eyes AVPU

D3: Drugs

Pupils are round, symmetrical and reactive Patient is gross neurologically free

Alert Verbal

4 3

Pain 2 Unresponsiveness

1

AMPLE A Do you have any allergies? M Do you take any medications on regular basis? PMH, any history of HTN, heart attack, stroke, DM, any long term disease P L  Last meal  Last tetanus shot  LMP E Event:  Can you describe to me want happened?  Car accident! Were you the driver or passenger / front passenger?  Were you wearing your seat belt?  Did you hit your head? Did you lose your conscious?  Do you remember what happened, before and after the accident? Conclusion: I am suspecting an intra-abdominal bleeding; we are waiting for (surgeon, orthopedics surgeon) to intervene Summary: If you are done  go for secondary survey: Introduction to examiner  Expose the patient Hello  Examine him head to toe, looking for fractures, Neck collar more detailed neurological examination Introduction to patient A/B / C / order x-rays / D / AMPLE

OSCE-guide-III.doc

Page 79 of 255

Emergency Medicine

NOTES: FLUIDS: - Trauma / GIT bleeding: we always start with 2 L bolus o If the patient is stable for the beginning  do not give anything more o It the patient was not stable, but becomes stable after the first 2L bolus  give maintenance fluids o If patient was not stable, and remains unstable  start bld transfusion: 1 unit of packed RBCs for every 3 units of fluids, and continue till you find source of bleeding Stable 2 L bolus Stable Give nothing Unstable 2 L bolus Stable Give fluids – for maintenance Unstable 2 L bolus Unstable Start blood transfusion – 2 RBCs Then continue 1 (RBCs) : 3 (NS) -

-

-

Anaphylactic shock: o 0.5 L bolus o Give epinephrine / steroids / anti-histaminics (Benadryl) Acute abdomen (pancreatitis / DKA): o 1-2 L bolus o Followed by 1 L / hour till the urine output improves Heart attack: o KVO (keep vein open) 100 cc / hour If trauma, ↓ BP, ↓ HR with warm extremities  neurogenic shock (spinal cord injury)  give only 2 L of fluids then give vasopressors

For the exam:  Tension pneumothorax  Hemothorax  Rupture spleen  Fracture pelvis  Fracture femur

OSCE-guide-III.doc

Page 80 of 255

Emergency Medicine

Unconscious Patient – Neuro 1- Introduction: - Mr … I am Dr … I am the physician in charge in the emergency room now, - Mr … Mr … if you hear me open your eyes please. Tap on the shoulder, do you hear me  I would like to activate the ACLS code please / start primary survey  Check the mouth, listen for patent air way  Give 4L O2 via nasal A  What is the O2 please canula B  Trachea central, chest is moving  Monitor O2 for need  Listen to lungs, heart to intubation  2 large IV lines; for I would like to get the vital signs please: C  Based on vitals, no ↑ in BP and no ↓ in HR (as seen IV fluids and to in Cushing triad) withdraw samples  No fever  No abnormal breathing patterns  Mr … I will open your eyes, and shine light in it, pupillary reaction  both D pupils are round, symmetrical and reactive  Mr … if you hear me can you please move your eyes up and down  NO locked-in syndrome  D3: universal antidotes: thiamine 100 mg 2- Glasgow Coma Scale  if < 8  intubate (ask about DNR) 1 2 3 4 5 6 No Pain Voice Spontaneou N/A N/A Eyes response s Words Confused Normal N/A Verbal No sounds Sounds Extension Flexion Withdrawal Localizes Obeys Motor No movements (decerebrat (decorticat commands e response) e response) 3- Cranial nerve exam: excluding the motor and the sensory: - Inspection: o Face is symmetrical, no deviation of the angle of the mouth o Both eyes are symmetrical, no nystagmus - I would like to do a fundoscopic examination, to look for disc edema, or retinal hemorrhage - Reflexes: Pupillary reflex (II, III) / Corneal reflex (V, VII) / Gag reflex: (IX, X) 4- Upper extremities: - Inspection: no abnormal posture or contractures - Tone: check the wrist and elbow in both sides  No cog wheel, no lead pipe rigidity, no clasp knife spasticity - Reflexes: Biceps / Triceps - Check the radial pulse 5- Lower extremities: - Inspection: - Tone: check by leg rolling  rigidity, elevate the knee rapidly  spasticity - Reflexes: Knee reflex / Ankle reflex / I would like to do Babinski reflex / Clonus - Check dorsalis pedis pulse - If the neck is cleared (by CT), I would like to do nuchal rigidity, Kernig's sign, Brudzinski's sign 6- To check for brain death: I would like to arrange for: dolls eyes, caloric reflex test

OSCE-guide-III.doc

Page 81 of 255

Emergency Medicine

Unconscious Patient – Diabetic 16 years old female found unconscious in her class, next 10 minutes manage and counsel Introduction: - Ms … I am Dr … I am the physician in charge in the ER, - Ms … … if you hear me; can you open your eyes please? Tap on the shoulder, do you hear me  I would like to activate ACLS code please / start primary survey A B

C

D

 Check the mouth, listen for patent air way   What is the O2 please  Trachea central, chest is moving   Listen to lungs, heart  I would like to get the vital signs please: ↑ BP and ↑ HR  2 large IV lines; for IV fluids5 and to withdraw samples6 When you ask the nurse for stat glucose by finger prick:

Give 4L O2 via nasal canula Monitor O2 for need to intubation

Hypoglycemia Hyperglycemia  Stat 100 mg thiamine IV  Stat insulin 10 units IV  Stat 50 ml D50 (Dextrose 50%) IV  Stat 100 mg thiamine IV  If no IV line  glucagon IM  2 L fluids At that time, the patient will Orient her; … your blood sugar was low, your class-mates start to regain her conscious brought you here, you are in the ER in hospital, you are doing well now, how do you feel right now? Patient states that she is  Reassure her worried she will lose her  I can help by giving you a doctor’s note exam / or other important  This is a very serious condition, you need medical appointment! attention for some time it is not safe to leave D1: Brief neurology  Start D5 (Dextrose 5%): 250 ml / hr D3: Dextrose  Nurse, I would like to monitor her blood glucose every 5-10 minutes

+ In case of DKA and physical exam: History  The same as in hypoglycemia (see below)  The causes are (5 Is): insulin missed / infection / intoxication / ischemia / infarction Physical exam  Brief neuro exam  Brief DM exam + In case of hypoglycemia: History Are you diabetic? Analysis Diabetic  When were you diagnosed? And how? history  Do you take insulin?  Have you had coma (DKA or hypoglycemia) before?  When was your last DM follow-up visit? Any reason?  At that time; were you controlled? Symptoms free?  When was your last Hb A1C test? What was it?  How about last few days, were you measuring your glucose? EVENT  This morning, did you get breakfast, your insulin? Did you check your glucose? Did you exercise?  Before you lost conscious, hoe did you feel? Hungry / shaky / dizzy / sweating? 5 If the HR is normal and other VS are normal, you can give only 50 ml/hr to keep vein open (KVO) But if ↑ HR  give 2 L fluids for follow-up 6 For any female patient: β-HCG with the blood works you will order

OSCE-guide-III.doc

Page 82 of 255

Emergency Medicine

Impact

Red flags DD

PMH

 

If pt dx < 5 years  no worry about complications If pt dx > 5 years screen for complications (urine changes / visual changes / numbness and tingling in her feet)  Constitutional symptoms  How much insulin do you take? Do you make sure you eat after your insulin dose?  Any ↑ or ↓ in your weight? Stress?  Did you start new medication? HEAD SSS /+/ Pregnancy

Counselling: - What is your understanding about diabetes mellitus? Pathophysiology: - It is a condition related to our blood sugar. Whatever we eat, the food contains different components, including sugar. The food travels through the food pipe to our stomach, to our bowels where it is absorbed and goes to all our body. Our organs (brain / muscle) use this sugar as source of energy. In order for muscles to use this sugar, it needs a key to enter into cells, this key is the insulin. - We have two types of DM, type I and type II. - Patients with DM type I, their body does not produce insulin, so we need to compensate for that by giving it from external source. Complications: - High blood sugar is harmful for our bodies, because it affects all our blood vessels, the small and big ones, and may give a lot of complications! It might cause kidney, eye, or nerves injury and harm on the longer term. - On the other hand, low blood sugar is even more dangerous; do you know why? Because our brain can not survive without blood sugar for more than 5-7 minutes, it is the only source of energy to our brains. Prevention: - What happened to you is a very serious condition, and it might happen again. The best way to treat is to prevent this from happening; by: o Make sure that you always eat after your insulin dose o Monitor your blood sugar frequently o If you exercise, adjust your insulin dose based on your blood sugar level - Now, if this happens again, do you know how to identify it before you totally lose your conscious? o Whenever you feel hungry / sweating / shaky / dizzy / heart racing o You need to stop, and immediately eat a candy / chocolate / juice o So, you need to keep glucose tablets in your bag, to take it in case of emergency • If you are at home; keep monitoring your blood sugar, • If you are out; reach to the nearest ER Emergency measures: - If you exercise, there is a special type of injections (glucagon emergency kit); if your blood sugar drops suddenly, use it, or other people can use it to inject you. - That is why it is important that you have a bracelet that mentions you are diabetic, so if you lose conscious and some one finds you, they can identify the situation and provide help. Follow-up: - You should see your family physician within few days, and he can refer you to “diabetes” clinic, for more education and assessment. - I will still give you some brochures and web sites in case you would like to know more. Notes: If you are the family physician, what referral will you do for a diabetic patient? - Diabetes clinic / Foot specialist / Dietician - If DM type I > 5 years, OR type II at any time: Ophthalmologist / Nephrologist / Neurologist

OSCE-guide-III.doc

Page 83 of 255

Emergency Medicine

Unconscious Seizing Patient – DT / Epilepsy / Brain tumour / … Patient arrives to the ER with his wife, on the way he had attack of seizures, and received 1 dose of diazepam, he is unconscious now. In the next 10 minutes; manage. Introduction

Very brief introduction to wife, I will make sure he is stable then I will ask you more questions Mr … ; Patient is unresponsive, I will start my primary survey: can you hear me  A: can you open your mouth (open and comment) / trachea central / JV not engorged STABILIZE  B: listen to lungs and apex / normal air entry on both sides / normal heart sounds  C: can I get the vitals please! Normal! 2 large IV lines please ─ One to give IV fluids 50 ml/hr to keep vein open (KVO) ─ The other one is to withdraw samples  D1; deficits: pupillary reaction  D3; drugs: universal antidotes  thiamine 100 mg / if O2 is ok, no need for naloxone, if blood sugar is ok, no need for dextrose If at any time, the patient starts to seize, give ativan 2 mg IV and reassess ABCD History Event  First time to seize?  Can you describe what happened? Did he fall to the ground? (wife)  Before he seized; did he shout? Starred at the wall? Complained of strange smell?  Was all his body seizing or part of it? For how long? Did he bite his tongue? Rolled up eyes? Did he wet himself? Was breathing?  Did he regain conscious alone or with intervention? Cause  History of epilepsy? Medications for epilepsy?  And mood stabilizers medications?  RECENTLY, did he complain of: Neurological / Constitutional symptoms  Any history of trauma / head injury?  Recent ear infection?  SAD: sweating / shaking  Any medications / blood thinners PMH  Long term disease; e.g. HTN, DM, kidney, lung, or heart disease  Previous hospitalization / surgeries?  FH of epilepsy FH Examination  Vitals from the examiner  Glasgow coma scale (if < 8  arrange for intubation) Neurological examination:  Cranial nerve examination  UL and LL: tone and reflexes Management  Stat neurology consult  Stat CT brain

OSCE-guide-III.doc

Page 84 of 255

Emergency Medicine

N.B. Causes of seizing: - Epilepsy relapse - Brain tumour / brain hemorrhage - Infection / meningitis - Organ failure - Electrolytes imbalance - Delirium Tremens - Withdrawal from sleeping pills - Cocaine overdose How to identify if this is narcotics overdose? - While doing AB; if O2 saturation is low even after you give O2, and respiratory rate is low; jump to shine light in the pupils  if pin-point-pupil  give stat naloxone 0.2 – 2.0 mg IV - Then back to ABCD Any unconscious patient: - A; if fluids  suction - B; if not breathing  mask and bag - C - D - Then assess Glasgow coma scale, if < 8  intubate (ask for DNR here) Common 5 causes for unconscious patients: - Hypoglycemia / DKA - Epilepsy - Meningitis - Stroke lateralization - Heart block

OSCE-guide-III.doc

Page 85 of 255

Emergency Medicine

Heart Attack – Chest Pain (MI or Heart Block) 4 cases: - Chest pain for 45 minutes, with normal ECG - Chest pain for 45 minutes, with ST elevation - V. fib: chest pain and the patient is intoxicated - V. fid: chest pain and the patient is calm Introduction Brief history

ABCD

MONA

Oxygen Aspirin Nitrates

Morphine

± β-blockers

No ST elevation

ST elevation

    

… How are you feeling right now?  Os Cf D  PQRST  ↑...↓ – position or deep breathing  Patient is talking to me  air ways ok.  Listen to chest to exclude pneumothorax  breathing is ok  C and D briefly  4 L O2 via nasal canula  Aspirin 325 mg  Plavix 300 mg  0.3 mg puff X 3 times; 5 minutes apart - If no low BP - If the pt did not consume Viagra recently  If patient is in pain after 3 puffs of nitrates and there is NO hypotension or bradycardia  morphine 2 mg - If antero-lateral infarction (V2/3/4/6 & aVL): 5 mg morphine - If (II, III, aVF)  usually involves the right ventricle, and the morphine increase the pre-load  2 mg morphine only Given to all patients; EXCEPT:  History of asthma  ↓ BP, ↓ HR, heart block, and inferior MI (II, III, aVF)  Cocaine (unopposed alpha  ↑ blood pressure)  labetalol 20 mg (2 mg/min for 10 min) You give metoprolol 2 mg IV - Stat cardiology consult - Catheterization lab  angiography  Heparin (ask about the hospital protocol for heparin – usually 5000 U IV then 1000 U/hr by infusion) or give enoxaparin 1 mg/ kg) - Stat cardiology consult - Catheterization lab  angioplasty  Heparin (enoxaparin 1 mg/ kg)  If the cardiologist is not available  start thrombolytics, but first rule out the contra-indications for thrombolytics: - 2 recent (< 2 weeks): Recent surgery / Recent bleeding - 2 brain: Brain tumour / Bleeding in brain ± stroke - 1 heart: Aortic dissection

Chest pain presents with heart racing / SOB / nausea / vomiting / sweating History will be: chest pain analysis / cardiac symptoms / risk factors If blood pressure is low: we only give oxygen / aspirin / and plavix If inferior MI (II, III, aVF)  I need 15 lead ECG / do not give β-blockers Risk of bleeding with thrombolytics is 1%, but being serious, this needs consent

OSCE-guide-III.doc

Page 86 of 255

Emergency Medicine

Case 1: Chest pain with initial normal ECG Chest pain for 45 minutes, manage Introduction Pain history

ABCD

Brief history

… How are you feeling right now? Os Cf D PQRST (R: how about your jaws / back / arms) ↑...↓ – position or deep breathing Start O2 4 L by nasal canula … vitals please: comment and ask for regular VS check 2 IV lines: KVO / take samples and serial ECG - ECG normal: order Serial ECG every 5 minutes / if any ↓ VS / any change in pain - How did you come here today? - Did you take any aspirin? aspirin 325 mg / plavix 300 mg - Have you recently used Viagra? If no  nitroglycerine puffs 0.3 mg X 3; 5 minutes apart  D1: brief neuro exam  pt is grossly neurological free  Any results back?  Order chest x-ray?  Cardiac symptoms: heart racing / SOB / nausea / vomiting / sweating / dizziness / LOC  Chest symptoms  Constitutional symptoms  DD: GIT (difficulty swallowing / acidic taste / GERD / heart burn / hx of PUD) / Trauma / relation to respiration / calf pain / swelling  PMH: medications / blood pressure / blood glucose / cholesterol / SAD (especially for smoking and cocaine) / stress  FH: heart diseases / heart attacks      

Another ECG Normal ECG

Do physical exam: Vitals / compare BP in both arms – ? aortic dissection General status of the patient Eyes / mouth Heart and chest examination LL edema

    

OSCE-guide-III.doc

Page 87 of 255

Emergency Medicine

ST elevation

 

 

 

Morphine 2 mg (up to 5 mg in antero-lateral MI) β-blockers: metoprolol 2 mg IV (if NO contraindications: history of asthma, ↓ BP, ↓ HR, heart block, inferior MI, cocaine) Mr …, based on your symptoms and your ECG, it seems that you have a heart attack. (Am I dying Dr?). This is a serious condition, however you are stable and doing well, you are in a safe place and we have experience in dealing with that. Heart attack means one or more arteries providing blood supply to your heart are blocked by a clot, we need to reopen this clot. Stat cardiology consult Do we have catheterization lab? What we need to do now is to use medication called “thrombolytic” or “clot buster”. I would like first to make sure that you are a good candidate; recently have you had any surgery or bleeding? Have you ever had stroke before? Were you diagnosed or told that you have brain mass? Lesion? Based on the ECG, I do not see signs consistent with cardiac aneurysm, and based on the BP, it does not look like having dissection. Based on this, I can say that you are a good candidate for thrombolytics, it is a life saving procedure, however like any other medical intervention, it has side effects; it might cause bleeding in 1% of cases, this includes stroke, so we need your consent if you would like to have it! Heparin (ask about the hospital protocol for heparin – usually 5000 U IV then 1000 U/hr by infusion) or give enoxaparin 1 mg/ kg) Can you get the thrombolytics kit please

ST elevation:  Lateral MI  Inferior MI

The patient wants to leave the hospital: - I would like to make sure he is competent, not under influence of alcohol or any substance, and to rule out suicidal ideation - I would explain to the patient: diagnosis / treatment / side effects of treatment / complications of not receiving treatment / alternatives - I will document this, and I will ask the patient to sign a LAMA (leaving against medical advice), and I will let him go

OSCE-guide-III.doc

Page 88 of 255

Emergency Medicine

Case 2: Chest pain with STEMI Introduction … How are you feeling right now?  Os Cf D Pain history  PQRST (R: how about your jaws / back / arms)  ↑...↓ – position or deep breathing ABCD  Start O2 4 L by nasal canula  … vitals please: comment and ask for regular VS check  2 IV lines: KVO / take samples and serial ECG - ECG: ST elevation in V 2/3/4, aVL  ant-lat MI - How did you come here today? - Did you take any aspirin? aspirin 325 mg / plavix 300 mg - Have you recently used Viagra? If no  nitroglycerine puffs 0.3 mg X 3; 5 minutes apart  D1: brief neuro exam  pt is grossly neurological free  Any results back?  Order chest x-ray?  Cardiac symptoms: heart racing / SOB / nausea / vomiting / sweating / Brief history dizziness / LOC  Chest symptoms  Constitutional symptoms  RISK factors  PMH: medications / blood pressure / blood glucose / cholesterol / SAD (especially for cocaine) / stress  FH: heart diseases / heart attacks ST elevation  Morphine 2 mg (up to 5 mg in antero-lateral MI)  β-blockers: metoprolol 2 mg IV (if NO contraindications: history of asthma, ↓ BP, ↓ HR, heart block, inferior MI, cocaine overdose) Mr …, based on your symptoms and your ECG, it seems that you have a heart attack. (Am I dying Dr?). This is a serious condition, however you are stable and doing well, you are in a safe place and we have experience in dealing with that. Heart attack means one or more arteries providing blood supply to your heart are blocked by a clot, we need to reopen this clot.  Stat cardiology consult  Do we have catheterization lab? What we need to do now is to use medication called “thrombolytic” or “clot buster”. I would like first to make sure that you are a good candidate; recently have you had any surgery or bleeding? Have you ever had stroke before? Were you diagnosed or told that you have brain mass? Lesion? Based on the ECG, I do not see signs consistent with cardiac aneurysm, and based on the BP, it does not look like having dissection. Based on this, I can say that you are a good candidate for thrombolytics, it is a life saving procedure, however like any other medical intervention, it has side effects; it might cause bleeding in 1% of cases, this includes stroke, so we need your consent if you would like to have it!  Heparin (hospital protocol or give enoxaparin 1 mg/ kg)  Can you get the thrombolytics kit please Have time Physical exam: listen to lung bases / S1 and S2 / compare BP both arms

OSCE-guide-III.doc

Page 89 of 255

Emergency Medicine

Case 3: Chest pain – v fibrillation / v tachy -

-

-

I would like to initiate code blue – ACLS Is this is the patient ECG? Is it the last one? We need to start defibrillation! o Done. o Can you give me the last ECG please?  NORMAL ECG How do you feel? Do you feel drowsy? Because you have just had cardiac arrest, we would like to make sure you are stable. Manage as the first case  the chest pain with normal ECG

Case 4: Chest pain – v fibrillation – intoxicated patient -

-

-

I would like to initiate code blue – ACLS Is this is the patient ECG? Is it the last one? We need to start defibrillation! o Done. o Can you give me the last ECG please?  NORMAL ECG How do you feel? Do you feel drowsy? Because you have just had cardiac arrest, we would like to make sure you are stable. The patient will be rude / angry / aggressive. ? intoxicated o If the nurse gave you “cocaine” and said it was found with the patient; ask her to keep it with the hospital security o Mr … you have just had a cardiac arrest, this is a very serious condition, and might happen again, you should not leave, it is important to stay; we want to make sure you are stable. o If agitated  1 mg lorazepam injection

Cocaine (arrhythmias / HTN / neurological manifestations): - What were you doing in the party? Any alcohol? Drugs? - We will give you a medication that interacts with cocaine, it is very important to tell us if you took any cocaine recently! (if cocaine  Ca channel blockers) - Any weakness / numbness / tingling / vision problems / hearing problems? - Do you hear voices / see things? Do you think someone want to hurt you? -

Manage as the first case  the chest pain with normal ECG

OSCE-guide-III.doc

Page 90 of 255

Emergency Medicine

Heart Block 2 cases: - One of them is DNR (must be dated, valid, and signed) - The other case is: do not intubate / do not defibrillate. You can still pace maker 1- Introduction: - Is this is the last ECG for this patient? I do not see any signs of V. fib or V. tachy. I would like to see the patient first to make sure he is stable, and then I will look at the ECG. - Mr … I am Dr … working in the ER, do you hear me? - I would like to activate the ACLS code please / start primary survey  Check the mouth, listen for patent air way  Give 4L O2 via A  What is the O2 please nasal canula B  Trachea central, chest is moving  Monitor O2 for  Listen to lungs, heart need to intubation  I would like to get the vital signs please; C  2 large IV lines; for IV fluids and to withdraw samples 2- Glasgow Coma Scale  [if < 8  intubate (ask about DNR)] - This patient Glasgow coma scale is … - What is this patient code status? Any advanced directives? o Is it signed, dated and valid? o What does he have? - We will respect his wishes, we will not … (if DNR  do nothing) - If no DNR: D2 Third degree heart block:  1 mg atropine  any changes?  Pace maker: - Rate: 20 more than his base heart rate - Leads on sternum and apex D1 Brief neurological scan, pupils, … Collateral  Do we have his file? history  Can we contact his family physician? Or family member?  Does he have a med alert?  Do we have paramedics report?  Can we check his belongings? He is taking …, thiazides and digoxin +  I need to get his K level / digoxin level and I would like to check the dose for digi-bind and digi-fib Physical  Neuro exam exam  Cardiac exam Notes: - For any unconscious patient: ask about advanced directives or DNR! What is this patient code status? - Whenever the examiner or the nurse tries to give you an ECG at the room entrance, assess for V. fib or V. tachy and report: there are no signs of V. fib or V. tachy. I would like to see the patient first to make sure he is stable.

OSCE-guide-III.doc

Page 91 of 255

Emergency Medicine

Headache Introduction CC Analysis of CC

Headache for 2 hours (very acute – very serious) Os Cf D  Is this your first time  Did you get any trauma?  Would you describe it as the worst headache in your life? Thunder clap? - Can you please lie down?  Put the bed 45°, I would like to make sure u r stable! - ABCD: IV lines / D1: Pupils  PQRST  ↑...↓ – position or coughing  Acute neuro: fever / neck pain / stiffness / vision / hearing / gait / falls / weakness / numbness  The patient says: I am diabetic  stat blood glucose (prick) HTN / blood thinners / kidney diseases Kidney cysts / disease / aneurysm Cocaine

Associated symptoms PMH FH SH

If the patient loses his conscious  manage as unconscious patient  If ↑ BP and ↓ HR  I am suspecting Cushing triad, put the bed in 45° ABCD  D1: Pupils, D3: thiamine 100 mg Secondary  Glasgow coma scale As in the case of the survey unconscious patient Physical exam

Management

   

Orientation (if conscious) Quick cranial nerves Quick neuro (tone/ motor / sensory / reflexes) Nuchal rigidity, Kernig's, Brudzinski's signs

   

CT scan stat ± LP Others: Chest x-ray / Urinalysis Blood works (CBC / differential / lytes) Septic workup (samples / C&S)

   

OSCE-guide-III.doc

Stat neurosurgery consult Empiric antibiotics IV mannitol 20% – 1 gm / kg Intubate: - To keep air ways patent - To ↓ intra-cranial pressure

? subarachnoid hemorrhage ? meningitis (FEVER)

Treatment of meningitis: vancomycin 1 g IV q12h + ceftriaxone 2 g IV q12h ± ampicillin 2 g IV q4h (if >50 years or hx of alcohol use or immunocompromise )

Page 92 of 255

Emergency Medicine

Acute Abdominal Pain OsCfD /+/ PQRST  will give you an idea In abdominal pain cases:  If you suspect certain diagnosis  scan for the risk factors for that disease  If no diagnosis  continue GIT symptoms and general differential diagnosis You suspect obstruction  nausea / vomiting (COCA+B / coffee ground material) AND bowel movements  if vomiting  screen for dehydration  If you dx obstruction  check risk factors of obst  then rest of GI symptoms  If not obst  scan GIT  near-by systems  PMH for systemic disease If you suspect kidney stones  screen with renal symptoms  If you dx renal stone  check risk factors (diet, medications, hx of renal stones, uric acid, bone pains / fractures)  then rest of urinary symptoms Intestinal obstruction Intro Analysis of CC

… But first I would like to ask you, how do you feel now? Analysis: OsCfD: gradual, started colicky, and now continuous dull pain / PQRST / What ↑ or ↓ (position / eating / bowel movements / vomiting)  Screen for obstruction: ─ Nausea/ vomiting o Relation to pain, which started first, does it relief pain o COCA + Blood (coffee ground material) Impact  Screen dehydration (dizziness / light headedness / thirsty / LOC)  Bowel movements ─ How about any blood? Any time? ─ Still passing gas? Red flags Risk factors for intestinal obstruction:  Previous surgery? What? When?  Fever/ night sweats/ chills / appetite / loss of weight / lumps & bumps  PMH or FH of cancer or benign tumour  Hx of Crohn’s disease (hx of abd pain/ bloody diarrhea) / family hx  Hx of hernia / groin mass  Gall bladder stones / right upper quadrant pain Differential  Gastroenteritis: diagnosis ─ What did you eat yesterday? Place that you are not used to? ─ Diarrhea / blood in stools? ─ Anybody else ate with you and suffered from the same problem  Renal: flank pain / burning sensation / going more to washroom / stone  Liver: yellowish discoloration / itching / dark urine/ pale stools  Hx of HTN / SOB / cough / phlegm (aortic dissection) PMH / FH / SH  



X-ray findings of small intestinal obstruction: (1) Multiple air/fluid levels, (2) Dilated loops of small intestine, (3) No air under the diaphragm. Management: (1) NPO / NG tube, (2) Oxygen mask, (3) IV fluids, (4) Stat surgical consult, (5) Foley’s catheter, (6) Correct electrolytes.

OSCE-guide-III.doc

Page 93 of 255

Emergency Medicine

Right / Left lower quadrant abdominal pain – middle aged female  History: nausea / vomiting / fever / LMP / vag discharge / risky behaviour  Physical exam: MUST mention pelvic and bimanual exam  DD: (Left side): ectopic / PID / ovarian cyst / torsion / kidney stone  DD: (Right side): ectopic / PID / ovarian cyst / torsion / kidney stone / appendicitis / intest obstruction  Investigations: β-HCG / CBC / differential / abd-pelvic US / culture of the vag and cervical secretions  Long term complications: abscess / ectopic / infertility / intestinal obstruction / peritonitis  Signs at PV exam: left adnexal mass / cervical motion tenderness Acute abdomen in a female  missed period (ectopic), bleeding (abortion), discharge (PID) PID  Diagnostic plan: ─ Pregnancy test – β-HCG ─ CBC / ESR ─ Cervical culture (for Gonorrhea and Chlamydia) ─ Syphilis serology  What is the treatment of pelvic inflammatory disease? ─ Cefoxitin 2 g IV every 6 hours X 2 days (covers anaerobic bacteria) ─ Doxycycline 100 mg orally BID X 2 weeks ─ Remove any IUD (if present)  What are the indications of hospitalizing the patient? (1) Pregnancy, (2) Pelvic abscess on U/S scanning / high fever (> 38.5 °C), (3) PID at young age, (4) Recurrent PIDs, (5) Failure to respond to outpatient management, (6) Immunodeficiency (patients with HIV infection) or severe illness  Complications of PID: abscess / ectopic / infertility / intestinal obstruction / peritonitis Left lower quadrant abdominal pain – elderly  Diverticulitis  Signs: ─ Fever ─ Peritoneal signs: +ve cough tenderness / percussion tenderness / tenderness / rebound tenderness / DRE: severe pain  DD: diverticulitis / abscess / cancer  Investigations: ─ AXR, upright CXR: o May be normal o Localized diverticulitis (ileus, thickened wall, SBO, partial colonic obstruction) o Free air may be seen in 30% with perforation and generalized peritonitis ─ CT scan (optimal method of investigation). 97% sensitive, very useful for assessment of severity and prognosis. Very helpful in localizing an abscess ─ Hypaque (water soluble) enema – SAFE (under low pressure): o Saw-tooth pattern (colonic spasm) o May show site of perforation, abscess cavities or sinus tracts, fistulas ─ Barium enema: contraindicated during an acute attack: risk of chemical peritonitis (perforation) ─ Sigmoidoscopy/colonoscopy: o Not during an acute attack, only done on an elective basis o Take biopsies to rule out other diagnoses (polyps, malignancy)

OSCE-guide-III.doc

Page 94 of 255

Emergency Medicine

 

Management: (1) NPO / NG tube, (2) Oxygen mask, (3) IV fluids, (4) Stat surgical consult (5) IV antibiotics (IV ciprofloxacin 500 mg BID / IV Metronidazole 500 mg TID) Indications for surgery for diverticulitis: ─ Unstable patient with peritonitis ─ Hinchey stage 2-4 (large abscess / fistula / ruptured abscess / peritonitis) ─ After 1 attack if: (a) immuno-suppressed, (b) abscess needing percutaneous drainage ─ Consider after 2 or more attacks, recent trend is toward conservative management of recurrent mild/moderate attacks

Diabetic ketoacidosis  acute abdominal pain Analysis

Impact Red flags DKA

DD – causes of DKA

Management of DKA

                 

    

OSCE-guide-III.doc

Pain: OCD / PQRST / ↑↓ Vomiting: number of episodes + COCA AS: fever / malaise / cough / urinary symptoms / diarrhea / gastroenteritis Diabetic history Dehydration: feel weak / pee less / dry tongue and skin Hyperglycemia symptoms: thirst, polyuria, polydipsia, and nocturia Acidosis: shallow rapid breathing or air hunger (Kussmaul or sighing respiration), abdominal tenderness, and disturbance of consciousness Complications; of diabetes mellitus Non-compliance OR wrong dose Recent surgery Pregnancy Trauma MI Infection IV fluids – NS (1 L/hr x 2 hrs then 500 ml/hr x 2 hrs then 250 ml/hr x 4 hrs) Foley’s catheter Insulin drip 2 units / hour – check glucose and lytes every 2 hours When glucose reaches down to 15  fluids will continue as maintenance, 2/3 : 1/3 of D5W : NS + 20 mEq KCl/L. 4:2:1 rule: 4 ml/kg/hr for the first 10 kg, then 2 ml/kg/hr for the next 10, then 1 ml/kg/hr for the next whatever Serial blood glucose ABG / serum ketones CBC / lytes Septic workup (chest x-ray / blood cultures / urinalysis) ECG (for the ↑ in K+)

Page 95 of 255

Emergency Medicine

Acute Abdomen Patient came to ER with abdominal pain / vomiting / diarrhea / BP 90/60 / HR 120 Acute pancreatitis / dissecting aortic aneurysm (no vomiting) / perforated peptic ulcer Introduction

Analysis

Impact Red flags DD

PMH

I can see that you have a lot of pain, bear with me for few minutes and I will give you a pain killer as soon as I can.  In the moment, I would like to make sure you are stable  What are the vitals pleas? Stable Unstable Proceed to I am going to start my primary survey  ABCD  When you send blood works: add lipase / amylase history ↓ Did you vomit blood? How about coffee ground? (if yes: order blood) Os Cf D / PQRST / ↑↓ / relation to position / breathing / eating  Vomiting  COCA + Blood  Change in the bowel movements  Dehydration  How do you feel right now? What are the vitals please?  Constitutional symptoms Liver / GB  Yellowish discoloration / itching / dark urine / pale stools?  Recent flu-like illness?  Do you have hx of gall bladder stones? Repeated attacks abd pain? Stomach  Hx of PUD? GERD? Acidic taste / heart burn?  Alcohol? How much? When was the last time? Did u drink > usual?  Gastroenteritis (What did you eat yesterday? Place that you are not used to? Diarrhea / blood in stools? Anybody else ate with you and suffered from the same problem?) Medications  If vomited blood: Do you take steroids / NSAIDs / blood thinners? Kidney  Flank pain? Burning sensation? Dark urine? Frequency? Aorta  Hx of HTN / atherosclerosis / DM / ↑ cholesterol / smoking / SOB Trauma  Did you have trauma?  Medications / allergies / long term disease? 

Physical exam: abdominal exam Face Eyes for jaundice / mouth for dehydration / ulcers 7 8 Inspection  No Cullen’s sign / no Grey Turner sign  Cough tenderness

7

Cullen’s sign: peri-umbilical ecchymosis. It arises from spread of retroperitoneal blood associated with: pancreatitis / ruptured ectopic preg / ruptured aortic aneurysm / ruptured spleen / perforated duodenal ulcer 8 Grey-Turner sign: ecchymoses of the skin of the flanks, also with retroperitoneal bleeding

OSCE-guide-III.doc

Page 96 of 255

Emergency Medicine

Auscultation Percussion Palpation



Patient in severe pain: I can see that you are in pain, can you bear with me for few minutes then I will give you pain killer as soon as I can. ─ In order for me to examine you properly, I need to get good look at your abdomen, for that reason, can you please lie on your back! ─ Do you feel relieved like that, I understand. It will be only few minutes, do you want to give it a try? Slowly! Do you want me to help you! It is crucial to reach a proper diagnosis! ─ If still refusing  offer 2 mg morphine S.C. Finally she will lie down. ASK FOR X-RAY (3 view x-ray abdomen)  ? perforation ─ Perforation  severe guarding, will not be able to proceed ─ Obstruction ─



Patient is obviously in severe pain, I will not be able proceed with examination Management

I am suspecting acute peritonitis (? perforated peptic ulcer)  NPO / nasogastric tube  O2 mask  IV fluids (250 ml/hr) / Cross match blood

I am suspecting pancreatitis   

NPO / nasogastric tube O2 mask ICU admission





Pain killer: Meperidine 1 mg / kg or Fentanyl Stat SURGICAL consult



Pain killer: Meperidine 1 mg / kg or Fentanyl Stat GIT consult

  

3 view abdominal x-ray Abd U/S CT abdomen

  

3 view abdominal x-ray Abd U/S for gall stones CT abdomen



Lab: CBC / blood sugar / calcium / amylase / lipase



Lab: CBC / blood sugar / calcium / amylase / lipase Albumin level / serum Ca





DD:  Perforated PUD: vomiting coffee ground material  Aortic dissection: NO vomiting / severe pain shooting to the back  Acute pancreatitis: ─ NO upper GIT bleeding ─ Fever (due to chemical irritation not infection) ─ Pain improves when leaning forward ─ Paralytic ileus ─ Tetany Ethical question: The patient girl friend is on the phone, she is asking about his condition?! ─ I am still doing my examination, ─ I can assure you that he is well taken care of, and we will do our best to help him, ─ All the details of his medical information is absolutely confidential, and I can not release



OSCE-guide-III.doc

Page 97 of 255

Emergency Medicine

Upper GIT bleeding Patient vomited blood; esophageal: varices / gastric: perforated PUD Introduction ABCD Analysis

Impact Red flags DD

PMH FH SH

Because this is a case of GI bleeding, … protection … Mr …, … make sure that you are stable, then I will ask you … How do you feel right now? Be aggressive with fluids – 2 L IV  Os Cf D / COCA + dehydration  Associated with pain (painless: esophagus / pain: stomach)  PQRST  Nausea / vomiting / diarrhea  Recently; any blood in stools / dark stools?  Weak / drowsy / …  Constitutional symptoms – cancer stomach Liver  Yellow / itchy / urine / stool  Alcohol; how much / for how long / more than usual? SAD?  Long term liver disease? ↑ Abdomen size? Bruises? Memory? Concentration? Numbness LL?  Risk factors for hepatitis: piercings, bld transfusion, unprotected sex Stomach  History of PUD / pain / GERD, heart burn  Risk factors for ca stomach: smoking  Do you take any blood thinners? Aspirin? Steroids? NSAIDs? Medications Any medications / allergies Family history of gastric cancer   

↑ or ↓ fluids (if stable: fluids 250 ml/hr) and monitor vitals General appearance of the patient: cachectic / distressed / … I would like to check if there is any postural drop in the blood pressure Exam Liver exam: extra-hepatic signs of liver cell failure /+/ Bruises Abdominal exam: epigastric mass / pain / liver / ascites If painful: manage as acute abdomen case (perforated PUD) Management STAT GIT consult for UPPER GIT endoscopy IV pantoprazole (80 mg bolus then 8 mg/hr) IV octereotide (25 mcg/hr)  portal circ VD  ↓ portal pressure Abdominal x-ray Admission to ICU Longer term management:  If portal HTN: non-selective β-blockers  Advice on cutting down the alcohol  Advice to follow-up with the family doctor Vitals General

         

OSCE-guide-III.doc

Page 98 of 255

Emergency Medicine

Lower GIT bleeding Introduction  Because this is a case of GI bleeding, … protection …  Mr …, … make sure that you are stable, then I will ask you …  How do you feel right now? ABCD Be aggressive with fluids – 2 L IV  D1: you might find some deficits  previous cerebro-vascular accident (may be history of weakness and numbness) Analysis  Os Cf D / COCA + dehydration  Associated with abdominal pain  PQRST / ↑ – ↓ / related to meals  Vomiting blood? Nausea / vomiting? Impact  Weak / drowsy / … Red flags  Constitutional symptoms – cancer colon DD Liver  Liver: yellow / itchy / urine / stool Colon  Hx of dx / screening / for Ca colon? When? What was the result?  Recent change in bowel movement?  Recent changes in calibre of stools?  Risk factors for Ca colon: Fm hx of ca colon / diet (↓ fibres ↑ fat)?  Risk factors ischemic colitis? Atherosclerosis (similar to CAD)? Medications  Do you take any blood thinners? Aspirin? Steroids? NSAIDs? PMH Any medications / allergies FH Family history of colon cancer SH SAD Vitals ↑ or ↓ fluids (if stable: fluids 250 ml/hr) General  General appearance of the patient: cachectic / distressed / …  I would like to check if there is any postural drop in the blood pressure Exam  Abdominal exam: ─ If soft abdomen, and no findings (PE disproportionate to the pain)  ischemic colitis Management  STAT SURGERY consult  NPO / IV fluids / Foley’s catheter  Admission to ICU  Abdominal x-ray  CT angiography DD

With pain ─ Cancer colon: chronic intermittent ─ Ischemic colitis: acute / sudden

Painless ─ Diverticulosis ─ Angiodysplasia ─ Upper GIT bleeding ─ Aspirin ─ Hemorrhoids

Colitis: Radiation / Infectious / Ischemic / IBD (UC > CD) Abdominal x-ray findings: ─ Intestinal ischemia: thumb print sign ─ Ischemic colitis: pneumatosis intestinalis (coli) N.B. ischemic bowel  metabolic acidosis

OSCE-guide-III.doc

Page 99 of 255

Emergency Medicine

ECG  Normal  V fib /+/ V tachy /+/ Torsades du pointes  A fib /+/ Atrial flutter  ST elevation: o Pericarditis: all leads o MI:  V 2/3/4 ± V5/6, aVL: antero-lateral MI (left coronary)  II, III, aVF: inferior MI (right coronary, posterior and inferior surfaces)  Hear block – third degree /+/ Bundle branch block  Hyperkalemia /+/ Hypokalemia /+/ Hypercalcemia  Digitalis toxicity

OSCE-guide-III.doc

Page 100 of 255

Emergency Medicine

1. Rate:  Regular: 300/number of big squares (R-R)  Irregular: Number of “R”s x 6 2. Rhythm  check for P wave in lead II Tachyarrhythmias:  Sinus tachycardia  Irregular irregularities: A. Fibrillation  Saw teeth (regular irregularities): A. Flutter  Rapid SVT  Ventricular arrhythmias: Premature ventricular beats / V. Tachy / V. Fib / Torsades de pointes Bradyarrhythmias: (< 60/min)  Sinus bradycardia  Heart block: - 1st degree: P-R intervals increasing, but every P  QRS. - 2nd degree:  Mobitz type I: P-R intervals increasing, with missing QRS  Mobitz type II: P-R intervals constant, with missing QRS - 3rd degree: P-P has a rate, and the R-R has another rate 3. Axis Normally, QRS in leads I, II, III are positive (upwards ↑).  Right axis deviation: QRS in I is negative (downwards ↓); I and III facing.  Left axis deviation: QRS in II, III is negative (downwards ↓); I and III opponents. Diagram showing how the polarity of the QRS complex in leads I, II, and III can be used to estimate the heart's electrical axis in the frontal plane:

Lead I negative and aVF positive: Rt axis deviation / Lead I positive and aVF negative: Lt axis deviation. 4. Bundle Branch Block:  Normally QRS in V1 is downwards ↓, if in V1: QRS is upwards ↑ & wide: RBBB.  Wide QRS in V6 (M mountain): LBBB. A mnemonic to remember ECG changes is WiLLiaM MaRRoW, i.e. with LBBB there is W in V1 and M in V6 and with a RBBB there is M in V1 and W in V6

OSCE-guide-III.doc

Page 101 of 255

Emergency Medicine

5. ST segment: Angina

STEMI AND Non-STEMI

6. Others: Hypokalemia ST segment depression, inverted T waves, large U waves, and a slightly prolonged PR interval.

OSCE-guide-III.doc

Hyperkalemia 1- Flat P wave 2- Wide QRS 3- Spiked T wave

Page 102 of 255

Emergency Medicine

Phone calls THE SEIZING CHILD – PHONE CALL The mother is on the phone, panicked as her child is seizing for 3 minutes Introduction + Reassurance

Stabilize

Analyze the event During

After

Fever

Rule out BINDE PMH FH Counselling

- Good evening, I am Dr … the physician in charge at the (clinic/ER), may I know who is on the line? - Before I proceed, I would like to know the name of your child, your phone number and your address please! In case the line is disconnected, hang on, and do not call me, I will call you back. - What is your child weight? - I know this is stressful time for you; however, I need your help here. I will ask you some questions and give you some instructions. - Is this his first time? - Did you call the ambulance? I will ask the nurse to call for you - Is your child alert/conscious? Is he breathing? What is his color? - Can you support and monitor him, remove any object near him? - Can you put him on his side?  analyze during - Do not put anything in his mouth, do not fix him - Monitor till he stops seizing  assess for focal symptoms  after - Can you describe the seizure for me please! - Is it the first time? Or did it happen before? - Did you watch that? - Did he lose conscious? Which happened first, seizing or LOC? - Did he fall from height? Did he hit his head? - Was he shaking? Certain part of the body or whole? - Is he breathing? Did he turn blue? - Any tongue biting? Did he wet himself? Roll up his eyes? - How long did it last? How did it stop? Spontaneously? - Did he regain consciousness? After he regains consciousness; is he able to recognize you? Able to talk? To move? Confused? - Does he feel any weakness, numbness? - Does he have fever? When did it start? Any medical advice? Any diagnosis? Treatment? Is he taking the medications? Any reason? - Constitutional symptoms Meningitis / pneumonia Neurological deficits / diabetes mellitus Febrile seizure / epilepsy The seizing child counselling

Notes:  Febrile seizure vs. meningitis: 1st time send the ambulance, 2nd time: send the ambulance if: the seizure is > 15 minutes or > 2 attacks in 24 hours

OSCE-guide-III.doc

Page 103 of 255

Emergency Medicine

THE CHILD SWALLOWED MEDICATIONS / CAUSTIC MATERIAL The mother is on phone, panicked as her child swallowed medication / caustic material at home Introduction + Reassurance

- Good evening, I am Dr … the physician in charge at the (clinic/ER), may I know who is on the line? - Before I proceed, I would like to know the name of your child, your phone number and your address please! In case the line is disconnected, hang on, and do not call me, I will call you back. - What is your child weight? - I would like to ask you to take a deep breathe to calm down; I will need your help here. I will ask you some questions and give you some instructions, this is important for the sake of your child, ok. Primary survey - Is your child alert? + Stabilize Yes NO - Good to hear that - Do you know how to do CPR? - Yes, start it now - No, guide her: - If < 8 yrs 5 / 1 - If > 8 yrs 15 / 2 ABC - Is he breathing? Talking to you? - Can you check the child lips color? - Does he recognize you? - Can you check his mouth, and if there is any of the medications can you take it out? - If there are any remnants of the chemical on his mouth or face, can you wash it with some water? - Did you call the ambulance? I will ask the nurse to call the ambulance and we will send it to your home now - You need not to give him/her any thing by mouth - You do not need to do anything; you can just hold him/her? Analyze the event - Can you tell me what happened? - Which medication did he take? If does not know; who is taking this medication? Why for? Was it recently renewed? Is there vitamins? Sleeping pills? Aspirin? When ambulance arrives, please give them any bottle that the child might have taken medications from? - How much did he take? - For how long was he/she left alone? Any reason? Is it the first time? - If yes, analyze the event Or did this happen - Are there other children at home? Any history of repeated illnesses at before? home? Repeated visits to ER? BINDE - B: Was he a full term? Did he need special attention? Any congenital anomalies? - I: Are his/her shots up-to-date? - N: Is he a fussy baby? - D: Is he a difficult baby? PMH

OSCE-guide-III.doc

Page 104 of 255

Emergency Medicine

EMERGENCY TRAUMA – PHONE CALL A nurse is calling you from a remote rural medical center; she has a patient of trauma after a car accident, BP 90/60 and HR 120. Manage over the pho ne. A/B: Mouth / O2 / inspect chest, neck / listen to lungs, heart - I am Dr …, the physician in charge in the emergency room; may I know who is on the line? - Can you describe the case / situation for me! - Now, I would like to get your phone number (and address if reasonably far), and patient name, and if by any chance the line gets disconnected, do not try to call me, just hang the line and I will call you back, ok. - Are you a nurse? Do you have someone else with you to help? - Is the patient alert? - Does he have a neck collar? - Open his mouth, check it for any FB or blood, and make sure it is clear. Can you feel air coming from patient mouth? - What is his O2 saturation? Can you give him O2 please, 4 L/min via a nasal canula - Can you open the neck collar window and check his trachea and neck jugular veins, is the trachea central or not? Are the JV engorged? - Can you look to his chest; is it bilateral symmetrical movements of the chest wall? - Can you listen to air entry on both sides? - I am suspecting tension pneumothorax, I would like you to insert a wide bore needle (16 or 14 G) into the second intercostal space, in the MCL (on the upper margin of the third rib) o If she said, I did not do it before, or I am afraid I can not do it, or who is responsible for this: encourage her and tell her that you are responsible - Did you get a gush of air? o Yes, that is great, you have just saved the patient life - Can you re-check the trachea centrality, jugular veins and HR again for me please o If stable: can you please fix a chest tube in the 5th intercostal space, anterior Axillary line C: Vital signs / fluids / withdraw blood samples / D: - Can you please insert two wide bore (16 G) IV lines in the patient ante-cubital fossae, and give fluids (2 L NS bolus) and then withdraw samples for … - Can you find any source of bleeding? - D3: Give universal antidotes: o Give 100 mg thiamine o If ↓ RR  naloxone o If ↓ blood sugar dextrose - Can you tell me his Glasgow coma scale! Check his papillary reaction (D1). - I would like also to intubate the patient What are the requirements to transfer patient from a center to another center? - Accompanied by two trained medical personnel (paramedics, nurses, physicians) - Intubated and on ventilator - Secured IV lines and fluids - pre-arrangement with the place that will receive the patient

OSCE-guide-III.doc

Page 105 of 255

Physical Examination

Physical Examination

OSCE-guide-III.doc

Page 106 of 255

Physical Examination

Medical Physical Exam ─ ─ ─ ─ ─ ─ ─ ─ ─

─ ─



─ ─

Always be nice and smile Use the alcohol anti-septic Introduction Drape / cover the patient properly Warm your hands before touching the patient Warm your stethoscope If you are going to make painful manoeuvres: explain before / apologize for the pain / do not repeat Make sure you put the patient in the proper position at ease Whenever you notice the patient is in pain, tell him that you will give pain medication as soon as you can If you ask the patient to walk, make sure he will not fall, surround him Make sure you talk to the patient: I am going to look, feel, press, tap, listen (use simplified non-medical language) Report to the examiner – every thing – in medical terms, by inspection …, by palpation, percussion, … Cover the patient after you finish Thank the patient and the examiner

Introduction: - Good evening Mr …, I am Dr … I am the physician working in the clinic today / I am the physician in charge in the ER now. I understand that you are here because you have been having … For the next few minutes I am going to do physical exam for your … and I will need to ask you questions during my exam. Also, I will be asking you to do some movements and manoeuvres, if you feel any discomfort or pain, please do not hesitate to let me know and stop me - If you have any questions or concerns please feel free to ask me / to bring it up - If SOB: during my exam, if you feel that you can not continue, please stop me If there is history taking and then physical exam: - I will be asking you some questions, then I will do physical exam. Hopefully towards the end, we reach a working plan Vital signs: - If vitals are given: based on the vitals, the patient is stable, I would like to proceed. Or the patient is unstable! Or comment: with mil fever - If the vitals are missing one; e.g. the temperature: ask about it specifically - Vitals are not gives: o I would like to get the vitals before I start! o I am going to start my exam by measuring your vital signs that is your blood pressure, heart rate. And I will start by measuring your heart rate

OSCE-guide-III.doc

Page 107 of 255

Physical Examination

Abdominal examination:  Introduction / Vital signs / General inspection of the patient: pt is sitting comfortably … - Inspection - Auscultation: bowel sounds / bruits (aortic / renal / iliac) - Percussion - Palpation: superficial / deep / special tests Respiratory examination:  Introduction / Vital signs / General inspection of the patient: pt is sitting comfortably … - Inspection: face / hand / neck / chest / back - Palpation: tenderness / tactile fremitus / chest expansion - Percussion: dullness / percussion note / diaphragmatic excursion - Auscultation: regular / special tests o Then end with cardiology exam Cardiac examination:  Introduction / Vital signs / General inspection of the patient: pt is sitting comfortably … - Inspection: face / hand / neck / chest / heart (PMI) - Palpation: apex / left para-sternal areas for heaves / valvular areas for thrills - Auscultation: in Z format A-P-T-M o Leg exam for edema o Lung bases - If full CVS exam  peripheral vascular assessment: abdominal bruits / legs pulses palpation / chest exam Musculoskeletal examination:  Introduction / Vital signs / General inspection of the patient: pt is sitting comfortably … - Inspection: SEADS (scars / erythema / atrophy / deformity / swelling) / specific findings (bulk of muscles / bony symmetry) - Palpation: (TTC) tenderness / temperature / crepitus / effusion - ROM: active (if normal, NO need to do the passive) / passive / against resistance - Special test: mechanical (shoulder / elbow / hip / knee / ankle) o To complete my exam, I would like to do:    

Check the pulses of the limb (upper or lower) Brief neurological examination of the limb One joint above and one joint below examination The other side joint

Neurological exams:  Introduction / Vital signs / General inspection of the patient: pt is sitting comfortably … - Orientation: what is your name sir? Where are you? Time? Place? - Cranial nerves - Upper and lower extremities: o Inspection o Palpation / bulk o Tone o Motor power o Sensory o Reflexes - Gait / Romberg test - Cerebellar signs / Coordination - Cortical sensations: two points discrimination

OSCE-guide-III.doc

Page 108 of 255

Physical Examination

Abdominal examination: - Introduction - Vital signs - General inspection of the patient: o By general inspection, pt is lying down comfortably, no obvious distress o Can I take a look at your eyes, would you please look downwards? No jaundice. Upwards please? No pallor o Would you please open your mouth: no signs of dehydration or vomiting o Can I take a look at your hand?  Temperature is fine / and skin is moist  Normal capillary refill (< 3 seconds)  No obvious clubbing - I am going to drape you now! o Bed flat o Can you please put you hands to your sides o Allowing the patient to bend his/her knees so that the soles of their feet rest on the table will also relax the abdomen! -

Inspection; I will take a look at your abdomen: o By inspection: abdomen is flat, not distended, normal contour, umbilicus is inverted, abd is moving with respiration, no scars from previous surgeries, no bruises, no obvious bulge or mass o Would you please look to the left side, can you please cough  Check pt face for tenderness  there is cough tenderness  If no  cough another time  no herniation along the middle line. Thank you, you can look back if you wish

-

Auscultation; I am going to listen to your abdomen, warm my stethoscope: o McBurney’s point: normal bowel sounds (hyperactive / decreased) o 2 inches above the umbilicus  no aortic bruits o 2 inches above / 2 inches bilateral from umbilicus  no renal bruits o 2 inches below / 2 inches bilateral from umbilicus  no iliac bruits

-

Percussion: now, I am going to tap on your abdomen, can you point to your painful are. I am going to start away from there: o Percuss in 2 X 2 lines, and percuss to side  for ascites o No percussion dullness / normal tympanic percussion note / no percussion tenderness / no ascites

-

Palpation: o I am going to feel your abdomen. Start away from the painful area:  I am checking (name the 4 quadrants or the 9 areas of the abdomen); (NO) tenderness, guarding or rigidity o I am going to apply more pressure now: no obvious masses, no organomegaly o I am going to feel your kidneys now (bimanual)  no enlargement, no tenderness of the kidneys o I am going to do some special tests:  Murphy’s sign (Rt costal margin)  can you take a deep breath  Rebound tenderness: I am going to press and release my hand, can you tell me which causes more pain! (any point except McBurney’s)

OSCE-guide-III.doc

Page 109 of 255

Physical Examination

 

Press at McBurney’s point  is it painful Press left iliac area, tell me if there is pain: • Not painful • Pain to the left side: left iliac tenderness / negative Rovsing's sign • Pain to the right side  positive Rovsing's sign /+/+/+/+/+/+/+/ COVER THE PATIENT ABDOMEN /+/+/+/+/+/+/+/  Psoas sign: Rt LL extended; I am going to press on your thighs down, can you press up against my hand (actively flex thigh at the hip)  Obturator sign: Rt hip flexed, knee flexed; can you relax yourself, I will rotate your leg (int and ext rotation), if pain  +ve sign

-

-

Can you please sit up now; I am going to: o Tap on your back, check the costo-vertebral angle for renal tenderness o Listen to the back of your lungs  no crepitation Can you please lie down; thank you. I would like to complete my exam by doing pelvic and vaginal exam / and digital rectal exam o Inguinal LNs o Pelvic, vaginal and bimanual exam; looking for:  Bleeding and/or discharge  Adnexal masses / tenderness  Cervical motion tenderness  To obtain samples for culture and sensitivity o Digital rectal exam; looking for:  Bleeding  Piles / haemorrhoids  Fissures

How to differentiate between spleen and kidney by palpation: - Spleen changes position with respiration (goes down with inspiration) - You can feel the lower border of the spleen - Spleen does not ballot If there is history of Crohn’s disease: - Inspection: o General: truncal obesity / moon-like face • Red eyes • Mouth: ulcers o Hands: nail changes / clubbing / skin rash / psoriatic changes / joint tenderness o Abdomen: stria / scars o Lower limbs: erythema nodosum o DRE: fissures / fistulae - When he sits up  press on the sacro-iliac joints  sacroiliitis

OSCE-guide-III.doc

Page 110 of 255

Physical Examination

Liver Examination Patient vomited blood 20 minutes ago, perform focused examination for liver cell failure - Because the patient is vomiting blood, I would like to ask for protection for me and my team please (gloves / gowns / masks / goggles) - Introduction / Vital signs - Orientation: I am going to ask you some questions which are part of my physical exam. Do you know where you are now? What is the time? Do you know why you are here?  Patient is oriented to people, time and place - General: o o o o o

o o

-

Inspection: o

-

o

2 X 2 lines tap Liver: MCL (from above downwards and from down upwards) liver span Spleen: ant axillary line  last intercostal space / ask pt to take deep breathe in / then repercuss for the spleen Ascites: from midline to the side, no dullness  so there is no need to perform the shifting dullness (to be clinically palpable: ascites > 500 ml)

Palpation: o

o

-

Listen to liver: no hepatic rub / hum / or bruits Listen to spleen: no rub / no hum

Percussion: o o o

-

No bruises / normal hair distribution / no caput medusa / no dilatation of collateral veins / no obvious ascites / no scratch marks

Auscultation o o

-

Patient is lying comfortably no signs of obvious distress Eyes  sclera: no jaundice Mouth: no fetor hepaticus / no clots / no vomitus Face: no parotid gland enlargement Hands: no clubbing / nail changes / palmar erythema / Dupuytren’s contracture / atrophy of thenar and hypothenar muscles / look for IV marks / stretch your hands please (flapping tremors) Can you please lower your gown: no gynecomastia / no spider nevi Check lower limbs edema

Liver: start from the right iliac fossa and go upwards, while the patient is breathing in and out (push during inspiration, do not move your hand from the patient)  margin of liver is not palpable, not tender, and not nodular. Spleen: patient elevates his LEFT side 45°, support from left back. Start from above the umbilicus towards the spleen  spleen is not palpable

I would like to complete my exam by doing: o Digital rectal exam: for hemorrhage / piles o Check for testicular atrophy

   

DD of hepatitis: (alcoholic / viral / drug-induced) / cirrhosis / liver cancer Worst prognostic signs: ascites / encephalopathy Investigations: AST:ALT > 2:1, GGT, albumin, INR, CBC Treatment of encephalopathy: treatment of precipitating causes / Lactulose (15-30 ml tid) / Rifaximin (550 mg PO bid)



DD of ascites: liver cirrhosis / renal failure / heart failure / TB / malignancy / Hypoalbuminemia Treatment of ascites: Sodium restriction (20-30 mEq/d) / Diuretic therapy / Therapeutic paracentesis may be performed in patients who require rapid symptomatic relief for refractory or tense ascites



OSCE-guide-III.doc

Page 111 of 255

Physical Examination

Nasal bleeding  Hematological Examination -

Because the patient is bleeding, I would like to ask for protection for me and my team please (gloves / gowns / masks / goggles)

-

Introduction Vital signs

-

General: o Patient is lying comfortably no signs of obvious distress  Nose: open the speculum antero-posteriorly  Use the otoscope for ENT (nose / ear / mouth) o Look for bruises / petichae  if you find them  continue hematological exam

-

I ma going to feel your glands: o Occipital / post-auricular / pre-auricular / sub mandibular / sub mental / anterior / posterior cervical / supra-clavicular o I would like to check the axillary LNs (ant / post / med / lat / apical) o I would like to check the inguinal LNs:  Superficial (transverse, along the groin crease)  Deep: • Transverse: along the groin crease • Longitudinal: medial to the femoral artery (Lat  Med: NAVAL – Nerve Artery Vein LNs) o I would like also to check the popliteal LNs Check for bony pains: sternum and 3-5 spots on the vertebral column (I am going to press on your back)

-

CAN YOU LIE DOWN PLEASE? - Listen to heart / lungs - Abdominal exam: o Inspection o Auscultation o Percussion: liver / spleen o Palpation: liver / spleen o I would like to: perform vaginal and pelvic exam -

Finally, I would like to do: o Fundoscopic retinal exam o Orientation o Brief neurological exam:  Hands: pull/push/ do you feel the same  Legs: pull/push/ do you feel the same Patient is grossly neurologically free N.B. focal neurological deficits to DD between TTP and HUS.

OSCE-guide-III.doc

Page 112 of 255

Physical Examination

Questions: - Diagnosis: o ITP (Immune Thrombocytopenic Purpura)  Most common cause of isolated thrombocytopenia  Diagnosis of exclusion (i.e. isolated thrombocytopenia with no clinically apparent cause) - Investigations: o CBC: thrombocytopenia o Peripheral blood film: decreased platelets, giant platelets • Bleeding time: increased / PT and aPTT: normal • Anti-platelets antibodies o Bone marrow: increased number of megakaryocytes (critical test to rule out other causes of thrombocytopenia for age > 60 years; e.g. myelodysplasia) o Markers of hemolysis: increased unconjugated bilirubin, increased LDH, decreased haptoglobin o Kidney function tests (urea / creatinine for HUS) - Treatment: o Steroids (methylprednisolone 1 g/d for 3 days, then prednisone 1.5 mg/kg/day) o Immunoglobulins (if low platelet count): IVIG 1 g/kg/d X 2 days o Splenectomy o Vaccination (pneumococcus, meningococcus, HIB) - DD: o ITP (Immune Thrombocytopenic Purpura) o TTP (Thrombotic Thrombocytopenic Purpura) o HUS (Hemolytic Uremic Syndrome): ITP TTP HUS  Remitting / relapsing  Predominantly adults  Predominantly children course  Thrombocytopenia  Severe thrombocytopenia  Mild fever  Micro-angiopathic  Micro-angiopathic hemolytic anemia hemolytic anemia  Splenic discomfort (MAHA) (MAHA) (mild engorgement)  Renal failure  Renal failure  Neurological symptoms (headache, confusion, focal deficits, seizures)  Fever  CBC and blood film: decreased platelets and schistocytes Investigations (both TIP, HUS)  PT, aPTT, fibrinogen: normal  Markers of hemolysis: increased unconjugated bilirubin. increased LDH, decreased haptoglobin  Negative Coombs' test  Creatinine, urea, to follow renal function  Stool C+S (HUS)  Plasmapheresis ± steroids Management (both TIP, HUS)  Platelet transfusion is contraindicated (increased microvascular thrombosis)  Plasma infusion: if plasmapheresis is not immediately available  TTP mortality – 90% if untreated

OSCE-guide-III.doc

Page 113 of 255

Physical Examination

Chest Examination - Introduction - Vital signs; especially: tachypnea / temperature - General inspection of the patient; comment on respiratory distress - Inspection: o General: ▬ ▬ ▬ ▬

o

Neck; can you lower your gown please ▬

o

Trachea is central, no accessory ms used for breathing / LNs

Chest: ▬ ▬

-

Face: symmetrical, no nasal flaring / laboured breathing Eyes: jaundice / pallor Mouth / can you flip your tongue please: no central cyanosis / dehydration / exudates or secretions Hands: no nicotine staining / clubbing / peripheral cyanosis / normal capillary refill

Chest is symmetrical, no accessory ms used for breathing, no intercostal retraction Look from the side: no increase in the A/P diameter of the chest / no barrel chest

Palpation; I am going to feel your chest o Tenderness: ▬

o

Check the ant chest wall: no tenderness

Tactile fremitus: can you say 99 for 4-5 times, whenever you feel my hands on your chest  normal tactile fremitus ▬ ▬

Any lung pathology  ↑ tactile fremitus (↑ conduction) Any pleural pathology  ↓ tactile fremitus (insulation

o Chest expansion: normal chest expansion /+/ I will continue to examine you from the back, then I will check the front again /+/ Can you cross your arms please - Inspection: chest is symmetrical, no scars, swellings, or deformity - Palpation; I am going to feel your chest o Tenderness: no tenderness o Tactile fremitus: can you say 99 for 4-5 times, whenever you feel my hands on your back - Percussion; I am going to tap on your chest o Apex (2) / back (6) / sides (2)  normal percussion note: - Auscultation; now, I am going to listen to your back o Whenever you feel the stethoscope touching your back, can you please take a deep breathe in and out from your mouth o Normal vesicular breathing sounds bilaterally, no rhonchi, no wheezes - Now, I am going to demonstrate the special tests that should be done if there is consolidation, with auscultation: o Can you say letter “E”  egophony (normal: ee / over affected area: ay) o Can you say “99”  ↑ vocal fremitus o Whisper 1/2/3: whispering pectorology (↑ in audibility) /+/ I will now go to examine you from the front again /+/ - Percussion; I am going to tap on your chest – on both sides o Apex (2) / MCL (6) / sides (2)  normal percussion note: - Auscultation; now, I am going to listen to your lungs o Mid clavicular line – both sides o Normal vesicular breathing sounds bilaterally, no rhonchi, no wheezes - Cardiology exam: Auscultation

OSCE-guide-III.doc

Page 114 of 255

Physical Examination

Pneumonia ▬



CURB 65 score – Pneumonia Clinical Prediction Tool Confusion Urea: > 7 mmol, BUN > 19 Respiratory rate: > 30 breaths / min Blood pressure: systolic < 90 and diastolic < 60 65: or older in age Management 0–1 Can treat as outpatient 2–3 Consider hospitalization 4–5 Consider ICU

Mortality 30 ─ Dyslipidemia Diagnosis: ─ Visit ONE: o If hypertension urgency or emergency (sBP > 210 or dBP > 120) or end organ damage (e.g. confusion)  diagnose HTN o Else (provided 2 more readings during same visit)  Search for target organ damage: history (cardio-vascular risk factors) / examination  Investigations: ▬ CBC / Na+, K+ / fasting blood sugar / lipids (total cholesterol, HDL, LDL, TG) ▬ Kidney function tests / Urinalysis / Renal Doppler ▬ ECG / Echocardiogram ▬ For secondary HTN: TSH / Plasma aldosterone / renin levels / 24 hours urine for metanephrines / VMA  Life style modifications (↓ salt / ↓ alcohol / ↓ cholesterol / exercise)  Follow-up visit within 4 weeks ─ Visit TWO; within 4 weeks o If (target organ damage OR diabetes mellitus OR chronic kidney disease OR blood pressure > 180/110 mmHg)  diagnose HTN o Else (BP: 140-179 / 90-109 mmHg)  24 hours BP monitor (diagnose if mean awake sBP >135 mmHg and/or dBP > 85 mmHg or mean 24 hours sBP >130 mmHg and/or dBP > 80 mmHg) Management: ─ Target BP is < 140/90 mmHg, < 130/80 if DM or chronic kidney disease ─ Life style modifications (initial management): o Smoking cessation and decrease alcohol consumption o Diet: ↓ salt / ↓ cholesterol and saturated fats / follow Canada's Guide to Healthy Eating o Weight: maintain healthy BMI (18.5-24.9) o Moderate intensity dynamic exercise: 40-60 minutes, 4-6 times/week ─ Pharmacological: o First line: Diuretics; e.g. hydrochlorothiazide 12.5 – 25 mg PO od Except:  DM: ACEIs; Ramipril 2.5 – 5 mg PO od  Gout: Amlodipine (5 mg PO od) OR Candesartan (4 – 8 mg PO od)  Elderly (especially if IHD): ─ ACEIs ─ β-blockers: metoprolol 25 mg bid Especially if CHF / EXCEPT: asthma / bradycardia  Pregnant: ─ Hydralazine: 10 mg PO qid for few days then 25 mg PO qid ─ OR α-methyl dopa: 250 mg PO bid  If > 3 cardiovascular RF: statins / ASA o If partial response to standard dose monotherapy, add another first-line drug  Do NOT give β-blockers and Ca ch blockers  may cause heart block  Do NOT give ACE and ARBs  both ↑ K+,  Available combinations: Altace plus (ramipril + diuretic) / Diovan H o Notes on ACEIs:  Contraindications of ACEIs: Angio edema / Bilateral renal artery stenosis  ACEIs are nephroprotective except in acute renal injury  nephrotoxic  If patient on ACEIs developed cough  switch to ARBs ─ HTN emergency: Hydralazine: 20 – 40 mg IV or IM, repeated as necessary, decrease the dose in case of renal impairment

OSCE-guide-III.doc

Page 120 of 255

Physical Examination

SOB – shortness of breathe Patient who had a car accident 24-48 hours ago developed SOB. Complications: 1st day: atelectasis / fat embolism. 3rd day: DVT / PE

Brief cardio-pulmonary hx Vitals General

Neck Chest

Trachea JVP Inspection

Palpation

Percussion Auscultation

Lower limbs exam

Common causes: Cardiac (CHF) / Lungs (asthma / COPD / pneumonia / PE) Life threatening conditions: MI / PE / pneumothorax / aortic dissection I would also like to rule out orthostatic hypotension (? hypovolemia)  General assessment of the patient  Orientation (SOB  hypoxia  confusion)  Face: eyes / no signs of respiratory distress / mouth (no signs of dehydration / central cyanosis)  Hand: temperature / capillary refill / skin / no peripheral cyanosis / no nicotine staining / comment on both pulses  Sacral edema  Trachea centrality  JVP: not engorged  Symmetry  Accessory muscles breathing / intercostal retraction  Obvious pulsations / PMI  If car accident (bruises on chest wall)  Tenderness  Tactile fremitus  Apex  Para-sternal heaves  Chest back for any dullness  Heart (ask the patient to turn slightly to left)  Lungs: 8 spots scan for air entry  Lung bases  Inspection: SEADS  Palpation: temperature / capillary refill / peripheral edema / pulses / press for pain in calf muscle area  Special tests: - Measure leg circumference 10 cm below tibial tuberosity; difference should be < 2.5 cm between the two calves - Homans' sign10: there is pain in the calf or popliteal region with examiner's abrupt dorsiflexion of the patient's foot at the ankle while the knee is flexed to 90 degrees

Indications for intubation: ABG showing poor PO2 (60s) / elevated PCO2 (80s) / acidosis / GCS score < 8

10

A positive Homans' sign does not positively diagnose DVT (poor positive predictive value), and also negative Homans' sign does not rule out the DVT diagnosis (poor negative predictive value), and there is theoretical possibility of dislodging the DVT.

OSCE-guide-III.doc

Page 121 of 255

Physical Examination

DVT Clots clinical probabilities / risk factors (Virchow's Triad): ─ Vascular injury (endothelial damage); e.g. recent surgery ─ Venous stasis; immobilization (post-MI, CHF, stroke, post-operative, obesity, long travel or flight, chronic venous insufficiency) inhibits clearance and dilution of coagulation factors ─ Hyper-coagulability; aging, surgery, trauma, malignancy, antiphospholipid antibody syndrome, hormone related (pregnancy, OCP, HRT, SERMs) DD: muscle strain or tear, lymphangitis or lymph obstruction, venous valvular insufficiency, ruptured popliteal cysts, cellulitis, and arterial occlusive disease

Wells Clinical Score for Deep Venous Thrombosis Clinical Parameter Score Active cancer (treatment ongoing, or within 6 mo or palliative) +1 Paralysis or recent plaster immobilization of the lower extremities +1 Recently bedridden for >3 d or major surgery < 4 wk +1 Localized tenderness along the distribution of the deep venous system +1 Swollen unilateral superficial veins (non-varicose) +1 Entire leg swelling +1 Calf swelling >3 cm compared with the asymptomatic leg +1 Pitting edema (greater in the symptomatic leg) +1 Previous DVT documented +1 Alternative diagnosis (as likely or greater than that of DVT) -2 A Wells score can be interpreted in a binary (likely vs. unlikely) or ternary (low, moderate, or high probability) fashion. Ternary interpretation Binary interpretation ≥3 High probability score ≥ 2 DVT likely 1 or 2 Moderate probability score < 2 DVT unlikely ≤0 Low probability In the unlikely group (low-to-moderate risk of DVT)  D-dimer assay (ELISA): ─ Negative  rule-out DVT ─ Positive  duplex ultrasonography o Negative  rule-out DVT o Positive  treat for DVT In the likely group (moderate-to-high risk of DVT)  duplex ultrasonography ─ Negative  repeat clinical evaluation and ultrasonography in 1 week ─ Positive  treat for DVT When discordance exists between the pretest probability and the duplex ultrasonographic study result, further evaluation is required. ─ Other non-invasive tests include MRI. ─ Venography is the gold standard, but is expensive, invasive and higher risk

OSCE-guide-III.doc

Page 122 of 255

Physical Examination

Initial treatment of DVT: ─ Unfractionated heparin (UFH): requires bolus (7500-10,000 ill), followed by continuous IV infusion (1000-1500 IU/h). Advantages: rapidly reversible by protamine. Disadvantages: must monitor aPTT ─ Low molecular weight heparin (LMWH): administered SC. Predictable dose response & fixed schedule Long-term treatment: ─ Warfarin: standard treatment; o Should be initiated with heparin overlap – dual therapy for at least 5 days. Discontinue heparin after INR >2.0 for two consecutive days. o Warfarin should be dosed to maintain INR at 2-3. Monitor INR twice weekly for 1-2 weeks, then weekly until INR stable, then every 2-4 weeks o Duration of anticoagulant treatment (with warfarin unless otherwise noted): ─ First episode DVT with transient risk factor: 3 months ─ First episode DVT with ongoing risk factor (e.g. cancer, antiphospholipid antibody) or > 1 risk factor: consider indefinite therapy ─ Recurrent DVT (2 or more episodes): indefinite therapy ─ IVC filters: useful in those with contraindications to anticoagulant therapy, recurrent thromboembolism despite adequate anticoagulation, recurrent embolism with pulmonary HTN, or those who require emergent surgery without time to initiate anticoagulation ─ Special considerations: o Pregnancy: treat with LMWH during pregnancy, then warfarin for 4-6 weeks post-partum o Surgery: avoid elective surgery in first month after venous or arterial thromboembolic event ─ Preoperatively: IV heparin may be used up to 6 hrs pre-operatively; warfarin should be discontinued for at least 4 days pre-op. Surgery safe when INR 100 beats per minute + 1.5 Recent surgery or immobilization (within the last 30 d) + 1.5 Clinical signs of deep vein thrombosis +3 Alternative diagnosis less likely than pulmonary embolism + 3 Hemoptysis +1 Cancer (treated within the last 6 mo) +1 Score 0-1 2-6 ≥7

Clinical Probability of PE Low Intermediate High

OSCE-guide-III.doc

score > 4 score ≤ 4

Simplified Wells PE likely PE unlikely

Page 123 of 255

Physical Examination

Evaluation of suspected PE: ─ Low clinical probability of PE  D-dimer: o Negative: ruled out o Positive  CT scan with contrast: ─ Negative: ruled out ─ Positive: ruled in ─ Intermediate OR high probability: o CT scan: ─ Negative: ruled out ─ Positive: ruled in

Notes: 1. Use D-dimers only if low clinical probability, otherwise, go straight to spiral CT or V/Q 2. If using V/Q scan (CT contrast allergy or renal failure): ─ Negative V/Q scan rules out the diagnosis ─ Inconclusive V/Q scan requires leg US duplex to look for DVT (q2d) ─ High probability V/Q scan only rules in the diagnosis if have high clinical suspicion

CXR of PE: may be normal / wedge-shaped infiltrate / unilateral effusion / raised hemidiaphragm Treatment of PE: ─ Admit for observation (patients with DVT only are often sent home on LMWH) ─ Oxygen: provide supplemental O2 if hypoxemic or short of breath ─ Pain relief: analgesics if chest pain – narcotics or NSAIDs ─ Acute anticoagulation: therapeutic-dose SC LMWH or IV heparin – start ASAP o Anticoagulation stops clot propagation, prevents new clots and allows endogenous fibrinolytic system to dissolve existing thromboemboli over months o Get baseline CBC, INR, aPTT ± renal function ± liver function o For SC LMWH: dalteparin 200 U/kg once daily or enoxaparin 1 mg/kg bid – no lab monitoring – avoid or reduce dose in renal dysfunction o For IV heparin: bolus of 75 U/kg (usually 5,000 U) followed by infusion starting at 20 U/kg/hr – aim for aPTT 2-3 times control ─ Long term anticoagulation: o Warfarin – start the same day as LMWH/heparin – start at 5 mg PO od – overlap warfarin with LMWH/heparin for at least 5 days and until the INR is in target range of 2-3 o LMWH instead of warfarin for pregnancy; active cancer, high bleeding risk o Duration of long-term anticoagulation treatment: ─ If reversible cause for PE (surgery, injury, pregnancy, etc.): 3-6 months ─ If PE unprovoked OR ongoing major risk factor (active cancer): indefinite ─ IV thrombolytic therapy: o If patient has massive PE (hypotension or clinical right heart failure) o Hastens resolution of PE but may not improve survival or long-term outcome ─ Interventional thrombolytic therapy (massive PE is preferentially treated with catheter directed thrombolysis by an interventional radiologist, works better than IV thrombolytic therapy and fewer contraindications) ─ IVC filter: only if recent proximal DVT + absolute contraindication to anticoagulation

OSCE-guide-III.doc

Page 124 of 255

Physical Examination

Peripheral Arterial Disease Examination Introduction Vitals General Inspection

Palpation

Abdomen

Neurology

Burger test

Special tests

Based on the vitals, the patient is stable, I would like to proceed … Drape the patient / expose the lower limbs (triangular)  I would like to take a look at your feet, can you please remove the socks; do you want me to help you?!  SEADS  No signs of arterial insufficiency: no hair loss / no shiny tight skin / no hypertrophic nails  Temperature  Capillary refill (< 3seconds)  Pulses: dorsalis pedis / posterior tibial / popliteal / (to examiner) I would like to check the femoral arteries  Drape the patient / I would like to examine your abdomen / can you please uncover your abdomen  Listen for bruits (aortic / renal / iliac)  Check for light touch, here is a piece of cotton, this is how it feels; can you please close your eyes! Tell me when you feel it touching you! Check both lower limbs from distal to proximal  If light touch is ok, do not proceed with more tests I would like to raise your legs, for 1-2 minutes, if you feel any pain / numbness / tingling please let me know, check the color of the foot. Then dangle the feet and check the color  no pallor on elevation, no rubor on dependence  Burger test is negative. I would like to arrange for ankle / brachial index 

Investigations: ▬ CBC ▬ Fasting blood sugar / lipid profile ▬ ECG ▬ Angiography (side effects: nephrotoxic / allergy / aneurysm risk) ▬ Doppler U/S study of the arterial tree both lower limbs Treatment: ▬ Life style modifications (refer to HTN) ▬ Foot care ▬ Graded exercise ▬ Surgery (if severe disability)

OSCE-guide-III.doc

Page 125 of 255

Physical Examination

Diabetic Foot Diabetic patient with long hx of diabetes, has an ulcer for few days Introduction Vitals General Inspection

Based on the vitals, the patient is stable, I would like to proceed …  



   

Palpation

  

Abdomen

 

Neurology



  

 



Burger test Special tests Balance (if DM focused exam case)

OSCE-guide-III.doc

Drape the patient / expose the lower limbs (triangular) I would like to take a look at your feet, can you please remove the socks; do you want me to help you?! Describe the ulcer: location (in the sole at base of 1st metatarsal), shape (round, irregular), size (… cm), margins not elevated, no active bleeding or oozing No other ulcers in the same foot / check the other foot / check in between toes  no evidence of infection in between toes / in nails No pigmentation around the medial and lateral malleoli SEADS – quadriceps wasting / swollen joints No signs of arterial insufficiency: no hair loss / no shiny tight skin / no hypertrophic nails Temperature Capillary refill (< 3seconds) Pulses: dorsalis pedis / posterior tibial / popliteal / (to examiner) I would like to check the femoral arteries Drape the patient / I would like to examine your abdomen / can you please uncover your abdomen Listen for bruits (aortic / renal / iliac) Check for light touch, here is a piece of cotton, this is how it feels; can you please close your eyes! Tell me when you feel it touching you! Check both lower limbs from distal to proximal If light touch is ok, do not proceed with more tests Light touch sensation is absent distal to the level of … cm above ankle Proprioception: I will move your toe, close your eyes please, tell me is it up or down. Then move to the next joint. Start with head of metatarsal, medial malleolus, tibial tuberosity, … Vibration: tuning fork, here is the sensation you will feel, tell me when it stops  intact / decreased / absent Monofilament test: to distinguish between the light touch and pressure sensation / 10 points on the foot (9 on the sole, and one on the dorsum above the big toe meta-tarso-phalangeal joint) Ankle reflex; if you have time: knee reflex and Babinski

I would like to arrange for ankle / brachial index  

Gait ± ataxia Romberg test: positive (with open eyes: pt can balance himself by vision; while with closed eyes: pt loses balance

Page 126 of 255

Physical Examination

Neurological Examination o o o -

Introduction Vital signs General inspection of the patient: pt is sitting comfortably …

Orientation: what is your name sir? Where are you? Time? Place? Cranial nerves Upper and lower extremities: o Inspection o Palpation / bulk o Tone:  Just relax please, let me do everything for you. I am going to check the tone in your Rt arm …  Tone is normal, no hypo or hyper tonia o Motor power (5  0)  5 full power  4 less than full power (like Lt hand in Rt handed person)  3 can do the movement against gravity  2 can do the movement with the gravity eliminated  1 muscle twitches, not able to initiate movements  0 no power – no movements o Sensory:  Light touch: • Pin prick or piece of cotton • First check on forearm or sternum • Can you close your eyes please • Distal to proximal • Bilateral  sensation is equal bilaterally  Posterior column (B12 deficiency / alcohol / syphilis): • Vibration sense: tuning fork / test on sternum / tell me when it stops / start distal / if intact move on / if not intact go proximal on the next joint • Proprioception: eyes closed / start with the big toe or thumb / is it moving or not? / is it up or down? o Reflexes:  0 absent  1 weak (hyporeflexia)  2 normal  3 hyper reflexia  N.B. Babinski reflex: I am going to tickle the bottom of your foot: • Planter flexion: normal response • Big toe dorsiflexion and toes fanning: UMNL (e.g. stroke)

OSCE-guide-III.doc

Page 127 of 255

Physical Examination

-

-

-

-

Gait – ATAXIA o Can you take few steps for me please? o Protect the patient, surround him with your arms, and walk with him Romberg test o Can you put your legs together! o Can you close your eyes please! o Watch (protectively) for few seconds!  Ataxia due to peripheral neuropathy (B12 deficiency / DM / syphilis): ↑↑ with eyes closed  Cerebellar ataxia: no ↑ with closed eyes (always on) Cerebellar signs (stroke / alcohol / tumours / para-neoplastic / …): o Nystagmus:  Can you follow my finger please (move it side to side) • Physiological: transiently then corrected • Central: horizontal or vertical • Peripheral: horizontal only. Conditions: benign positional vertigo / acute labyrinthitis / drugs o Finger to finger:  Patient hand must be extended  Move the examiner hand  Check both upper limbs o Finger to nose test: lesion in the cerebellum on the same side.  Intentional tremors  Loss of coordination o Heel to shin: lesion in the cerebellum on the same side Cortical sensations: two points discrimination Mini-mental status exam

OSCE-guide-III.doc

Page 128 of 255

Physical Examination

Cranial Nerves Examination -

Vital signs Comment on the patient general condition

CN I: - Do you have problems with smells? Can you please close your eyes? - What is that? Coffee / ammonia - What is that? Ammonia / coffee CN II: The optic nerve: - Visual acuity: Do you wear glasses? reading / color (Snellen chart at 1 foot distance – 35 cm) - Visual fields: eye by eye / by confrontation (when you see my fingers wiggling) - Pupillary reflex: I am going to shine light in your eyes, please look straight to the wall, each eye: direct and consensual (afferent: CN II, efferent: CN III) - I would like to do fundoscopy examinations, looking for: disc edema, retinal hemorrhage, neovascularisation, nipping of the veins CN III, IV, VI: - By inspection; both eyes are symmetrical, no deviation, no nystagmus, no head tilting, no ptosis (CN III: opens, CN VII: closes) - I would like you to follow this pen, without moving your head please, and if you see things double or blurred at any time, please let me know  move the pen in large Hshaped manner, then conversion - Normal extra-ocular muscles movements, no nystagmus or double vision CN V: - Motor: o By inspection: no atrophy of the temporal or masseter area o Can you please clench, feel the temporalis and masseter o Can you open your mouth against my hand? - Sensory: o This is a piece of cotton, and this is how it feels, I am going to touch your face, and whenever you feel it, please tell me. Can you close your eyes please? o Touch the face in symmetrical areas; cover the ophthalmic, maxillary, and mandibular areas. Does it feel the same? o Facial sensation of the trigeminal nerve is intact and equal on both sides - Reflexes: o Corneal reflex (afferent: CN V, efferent: CN VII)

OSCE-guide-III.doc

Page 129 of 255

Physical Examination

CN VII: (mainly motor) - Motor: o Face is symmetrical, no deviation of the angle of the mouth, normal naso-labial folds, no drooling o Can you please copy me:  Raise your eye brows, can you frown (wrinkle your forehead) please?  Can you close your eyes, and do not let me open them?  Can you blow your cheeks? Whistle?  Can you show me your teeth? o If there is a mouth angle deviation, it is deviated to opposite side of lesion o UMNL vs. LMNL:  UMNL: intact upper face muscles (the nucleus receives double sided innervation)  LMNL (Bell’s palsy)  ptosis; all face muscles are affected - Sensory: o To complete the facial nerve examination, I would like to check the sensations in the outer 2/3 of the tongue o I would like to check the ext auditory canal and the tympanic membrane to rule out herpes zoster of the facial nerve - Reflexes: o Jaw reflex o Corneal reflex (afferent: CN V, efferent: CN VII) CN VIII - Check by whispering (ABC – CBA), while rubbing fingers in front of the other ear OR by rubbing your fingers - Because the hearing is normal, I am going to skip Weber and Rinne tests o Rinne: place the tuning fork in front of ear, then on the mastoid process o Weber: place the tuning fork on the forehead CN IX, X: - Patient voice is normal, no hoarseness - Can you swallow a sip of water please? Normal swallowing - Can you open your mouth please? Soft palate is symmetrical, uvula is central o Uvula deviates to the opposite side of the lesion - To check the reflexes: I need to do the gag reflex CN XI: - Can you please shrug your shoulders? - Turn your head to the right, and to the left. I am going to resist you. I feel for the opposite side sterno-mastoid CN XII: - Can you please open your mouth? Can you stick your tongue out? o Tongue is central, no deviation. No fasciculations or atrophy of tongue. o If there is a lesion, the tongue deviates towards the lesion side - Can you please move it to the right and to the left? Can you stick it against your cheeks?  Normal movements of the tongue

OSCE-guide-III.doc

Page 130 of 255

Physical Examination

Tremors ? Parkinson disease Introduction Vital signs General comment Inspection

Shaky right hand I would like to rule out any orthostatic hypotension Pt is sitting comfortably …, with Rt hand tremors

Tremors

Right hand tremors, not obvious on the left hand Count from 10 to 1 please  tremors ↑ with mental activity  consistent with Parkinson disease, and rules out anxiety  Stretch your hand plz / no fine tremors  r/o hyperthyroidism  No flapping tremors  rule out liver failure  Finger to nose / no intentional tremors  r/o cerebellar dis Patient tremors consistent with Parkinson disease, resting tremors, beads rolling, and limited to Rt hand (…). No head nodding (No) limited facial expression, decreased eye blinking, drooling Face Palpation Check the wrist and elbows: Rigidity  (No) cog wheeling Positive with parkinsonism  (No) lead pipe rigidity  (No) clasp knife spasticity Positive with stroke Standing / walking  Would you please stand up! Do you need help  patient finds Postural instability difficulty in standing up  Can you walk few steps for me please: comment with + or - Stooped posture - Shuffling (festinating) gait - Decreased arm swinging - Patient turns in blocks Special Tests  Rapid alternating movements (hand supination & pronation / oppose thumb to fingers)  dysdiadochokinesia  Can you please repeat “British constitution”  monotonous  Can you write a sentence for me  micrographia  Can you draw a spiral parallel to this (draw spiral on paper) I would like to do the mini-mental status exam 

Treatment of Parkinsonism:  Pharmacologic ▬ Mainstay of treatment: Sinemet (levodopa / carbidopa). Levodopa is a dopamine precursor, carbidopa decreases peripheral conversion to dopamine o Levodopa related fluctuation: delayed onset of response (affected by mealtime), end-of-dose deterioration (i.e. “wearing-off”), random oscillations of on-off symptoms o Major complication of levodopa therapy is dyskinesias ▬ Treatment of early PD: DA agonists, amantadine, MAOI ▬ Adjuncts: DA agonists, MAOI, anticholinergics (especially if prominent tremors), COMT inhibitors  Surgical: thalamotomy, pallidotomy, deep brain stimulation (thalamic, pallidal, subthalamic), embryonic dopaminergic stem cell transplantation

OSCE-guide-III.doc

Page 131 of 255

Physical Examination

Thyroid Exam Introduction Vital signs General

Thyroid Exam Inspection Palpation

Percussion Auscultation

11 12

BP, HR Can you stretch your hands: - Fine tremors - Palms for sweating - Nail changes - Hair loss (hypothyroidism)  Examine the eyes: - Exophthalmos – stand by the patient (stand behind the right shoulder and look from above) - Lid lag (can you follow my finger without moving your head – from above downwards)  Proximal muscle weakness: - Can you shrug your shoulders (bilaterally against my hand) please  Knee reflex: brisk11 reflex  Peritibial myxedema: indicates hyper-thyroidism Patient is sitting on a chair Can you swallow12 please?  no apparent thyroid enlargement  Thyroid gland: - From behind the patient, bi-manually - Then while swallowing a sip of water  thyroid movement is normal, I do not feel any masses, nodules, and no tenderness  Lymph nodes: - Sub-mandibular and cervical  DIRECT percussion on upper part of sternum  Checking for retro-sternal extension (no retro-sternal dullness)  BOTH lobes  For thyroid bruits 

Reflexes grades: 0 absent 1 hypo 2 normal 3 hyper (brisk) 4 hyper with clonus (ankle) Whenever you ask the patient to swallow, give a sip of water, it is difficult to swallow on an empty mouth

OSCE-guide-III.doc

Page 132 of 255

Physical Examination

Dermatomes

OSCE-guide-III.doc

Page 133 of 255

Physical Examination

Neck Examination Patient complaining of pain in the neck Vitals General Inspection



Palpation



    

 

ROM

   

Powers



Special tests

 

Can I get the vital signs please Comment on patient general condition: patient is sitting comfortably … I would like to take a look at your neck; can you lower your gown please? From the back: SEADS (Swelling / Erythema / Atrophy / Deformity / Scars) From the side: normal cervical and thoracic curvatures Can you cough please, any pain - No neck pain with Valsalva manoeuvre I would like to feel you back please - Temperature is normal - Tenderness: pressing on the spinal processes and para-vertebral muscles, trapezius and sternomastoid Thyroid: can you swallow for me please? Check the LNs: no enlarged LNs Can you touch your chest by your chin? Can you look to ceiling? Can you turn your head to the right? And to the left? Can you tilt your head? To the right and to the left? - Normal flexion, extension, rotation and lateral flexion Test again against resistance - Neck pain is not associated with muscle contractures Spurling test: stretching the nerve  reproduces the pain Upper limb examination and neurological screening

Neurological screen

Part of my exam is to check your upper extremities, can you roll up your sleeves please! Inspection Upper extremities are symmetrical, normal bulk, no atrophy / SEADS Palpation I am going to feel your shoulder; deltoid, biceps, triceps, forearm, thenar, hypothenar  are symmetrical / no deformity / no atrophy Motor Power Deltoid C5 Biceps C5/6 Triceps C7/8 Sensory C4: deltoid C5: biceps – lateral aspect Test light C6: thumb C7: middle finger C8: touch little finger T1: elbow – medial aspect Reflexes  Biceps and brachio-radialis C5/6  Triceps C7/8 Radial pulse Pulse Post encounter: what is the level of the lesion? C6 nerve root lesion (C5-C6): weak biceps, weak biceps reflex C7 nerve root lesion (C6-C7): weak triceps, weak triceps reflex Diagnosis: Osteoarthritis of the cervical spine at … level X-ray findings: osteophytes of the cervical vertebrae / narrowing of disc space / subchondral sclerosis (increased bone formation around the joint), subchondral cyst formation Management: ▬ Soft neck collar ▬ NSAIDs / acetaminophen ▬ Physiotherapy ▬ If worsening: neurosurgery consult / CT myelography / nerve conduction studies

OSCE-guide-III.doc

Page 134 of 255

Physical Examination

Carpal Tunnel Syndrome Hand pain for 6 months History Analysis CC  Os Cf D / PQRST / ↑↓  Radiation: how about your elbow? Your fingers?  Triggers: is it related to time? To work? What do you do? AS  Weakness, numbness, tingling  Swelling, redness, warm  How about the other hand? Impact  How did this affect your life? Your work? Red flags  Constitutional symptoms Differential  Hx of neck pain / injury / trauma diagnosis  Hx of diabetes / symptoms  Hx of thyroid disease / symptoms of hypothyroidism  Hx of autoimmune disease / Rh arthritis / symptoms  Headache / visual disturbances / shoes are getting tight  Pregnancy / LMP PMH / FH Physical examination Vital signs Inspection  SEADS (thenar / hypothenar ms)  No nail changes, no nodules / no deformity Palpation  Temperature: is normal  Tenderness: palpate distal radial bone, styloid process, joint line, styloid process, distal ulnar bone, base of the thumb, carpal bones, metacarpal bones, digits ROM  Flexion / Extension /+/ move your hand to the right, to the left  Can you make a fist / fan your fingers  there is no obvious damage to the nerves / muscles / and tendons of the hand  Thumb movements: - Touch base of your little finger (thumb opposition) - Move it all the way to opposite direction - Point to the ceiling (with hand supine, flat) - Touch the tips of your fingers Power  Like ROM but against resistance  Thumb 90°, DIP flexed: do not let me straighten it  Biceps ROM / against resistance / biceps reflex (C6) Sensory  Check with cotton tip,  For the ring finger: check both sides: ulnar / radial Special tests  Phalen's test,  Tinel's sign / tap on the carpal tunnel Investigations: EMG / nerve conduction studies Treatment: ▬ Modify nature of work ▬ NSAIDs ▬ Wrist splint ▬ Local corticosteroids injection ▬ Surgical decompression

OSCE-guide-III.doc

Page 135 of 255

Physical Examination

Hand Laceration / Wrist Laceration Physical examination Introduction Gloves / vital signs Inspection  If big wrist laceration (BIG band aid): DO NOT remove the band aid, should be removed under anesthesia  If small laceration, remove the band aid  describe the laceration: linear / size / position / elevated margins / active bleeding / oozing  Other ulcers / other hand  SEADS (Swelling / Erythema / Atrophy / Deformity / Scars)  No nodules / no deformity Palpation  Temperature: is normal, is hand pink and warm?  Tenderness: palpate distal radial bone, styloid process, joint line, styloid process, distal ulnar bone, base of the thumb, carpal bones, metacarpal bones, digits  Capillary refill  Feel the pulse  radial ± ulnar  Allen’s test ROM  Do NOT ask patient to apply force against resistance as this may rupture a partially severed tendon. Test active ROM only  Median nerve: thumb abduction (thumb to ceiling)  Ulnar nerve: finger spread (fanning)  Radial nerve: wrist extension  Tendon examination of the hand (flexor tendons): - DIPs; flexor digitorum profundus: hold the MCP / PIP joints in extension, and ask the pt to flex DIPs. - PIPs; flexor digitorum superficialis: hold all fingers – except one you are testing – in extension and ask pt to flex the remaining finger - MCP joints: intrinsic (lumbricals) Power  Do NOT ask patient to apply force against resistance as this may rupture a partially severed tendon. Test active ROM only Sensory  Check with cotton tip or pin-prick, for light touch  Check two point discrimination on either side of each digit  For the distribution of the three nerves: ulnar / radial / median Neuro-motor examination of the hand Median Sensory Radial aspect of index finger pad Motor Flex DIP if index finger (FDP) extrinsic Motor Thumb to ceiling with palm up intrinsic (abductor pollicis brevis)

Ulnar Radial Ulnar aspect of little finger pad Dorsal web space of thumb Flex DIP of little finger (FDP), Extend wrist and thumb, extensor carpi radialis (extensor pollicis longus) Abduct index finger (first dorsal interosseous)

Structures lacerated Diminished ulnar territory sensation Allen test shows (no) refill from the ulnar circulation FDS weakness in little finger and ring finger

Ulnar nerve Ulnar artery Flexor retinaculum, ulnar two divisions of FDS

Management: clean and explore wound under local anesthesia and sterile conditions. Consult plastic surgery for micro-vascular repair. If at night, may suture the skin and arrange for pt to be seen by plastic surgeon next day.

OSCE-guide-III.doc

Page 136 of 255

Physical Examination

Back Pain Acute

 

Acute on top of chronic Chronic

Herniated disc Muscle spasm

Pain at end of day shoots to legs, increases when leaning forward:  Disc herniation  Degenerative disease (OA) Pain related to position, increases when extension (or leaning backwards), walking down the hill (arching the back)  spinal canal stenosis Pain wakes patient in morning, improves at end of day, stiff  Ankylosing spondylitis  limitation in all directions

Neurological exam

Mechanical exam

Red flags for back pain: BACK PAIN  B bladder or bowel dysfunction  A anaesthesia (saddle)  C constitutional symptoms  malignancy  K chronic disease    

P A I N

OSCE-guide-III.doc

parathesia age > 50 years old IV drug user neuro-motor deficits

Page 137 of 255

Physical Examination

Acute Back Pain Acute back pain: pain for 3 days Acute on top of chronic: pain for 3 hours – old patient Introduction

Analysis

Impact Red flags

DD PMH FH SH

OSCE-guide-III.doc

How do you feel? Do you prefer to remain standing, sit, or lie down? That is ok, whenever you want to lie down or rest, feel free to do so Os Cf D  What were you doing? What do you do for work?  Did you hear any crack sound?  Were you able to stand up and continue? Were you able to move? Did anyone help you?  Is this the first time?  How about at rest? During night?  Recently did you have any discomfort? Milder back pain? How did it affect you? PQRST  Does it shoot to your thighs? Toes?  Which is bothering you more; your back or your legs? ↑↓  How about lying down? Stretch your back? Coughing? Moving? Leaning forward or backward?  Any medication? Did it help?  How does it affect you?  Mets to lungs / liver / brain  Constitutional symptoms  Overweight In addition to your pain, did you notice any other symptoms:  Weakness, numbness, tingling  Difficulty with balance, falls  Any difficulty passing urine? How about bowel Cauda equine movement? Did you find that you soiled yourself?  Any numbness in the buttocks area?  Do you have morning erection? Any sexual dysfunction? Cancer prostate / bladder Cancer prostate

Page 138 of 255

Physical Examination

Chronic Back Pain Pain for 3 months Introduction Analysis

Do you prefer to remain standing, sit, or lie down? That is ok, whenever you want to lie down or rest, feel free to do so Os Cf D  What were you doing? What do you do for work? PQRST

Impact Red flags

DD PMH FH

Quality: stiffness Timing: is it worse in morning? Improves with time? Or is it worse at the end of the day?  Does it shoot to your thighs? Toes?  Which is bothering you more; your back or your legs? ↑↓  How about lying down? Stretch your back? Coughing? Moving? Leaning forward or backward?  Any medication? Did it help? AS  Pain other joints (knees / hips / hands) / distribution? Osteoarthritis Sero Pain other joints (knees / hips / hands) / distribution? Ankylosing negative  Eye pain, redness / mouth ulcers spondylitis symptoms  Skin changes / nail changes / hx of psoriasis  Repeated attacks of abd pain / diarrhea  Urethral discharge  How does it affect you?  Mets to lungs / liver / brain  Constitutional symptoms  Overweight In addition to your pain, did you notice any other symptoms:  Weakness, numbness, tingling  Difficulty with balance, falls  Any difficulty passing urine? How about bowel movement? Did you Cauda equine find that you soiled yourself?  Any numbness in the buttocks area?  Do you have morning erection? Any sexual dysfunction?     

 

Trauma Injury to back Osteoarthritis Ankylosing spondylitis Other rheumatic disease

SH Osteoarthritis: older patient / worse at evening Ankylosing spondylitis: ▬ Morning stiffness improves by time ▬ LSS x-ray: sacroiliitis OR fusion of SI joints ▬ ESR: ↑ ▬ HLA-B27 tissue antigen: positive Associated symptoms: inflammatory arthritis / Uveitis / psoriasis / IBD / pericarditis / aortic regurgitation Management: ▬ No cure ▬ Regular therapeutic exercises to prevent deformity (swimming / back extension exercises) ▬ NSAIDs: Indomethacin (50 mg PO bid) or Naproxen (250 mg PO bid) ▬ In severe cases: total joint replacement

OSCE-guide-III.doc

Page 139 of 255

Physical Examination

Back Joint Examination

Neurological screen

Physical examination Introduction Can you stand up please? Vital signs Inspection  Gait / balance / stance  Ask the patient to stand up from sitting position  Posture: normal cervical, thoracic, lumbo-sacral curvatures  Adam’s forward bend test (if scoliosis: the scapula will be higher) - No scoliosis or kyphosis  SEADS Palpation  Temperature  Tenderness: spinal processes, para-vertebral muscles, sacro-iliac joints (medial to dimples of Venus) ROM  Can you touch your toes with your fingers? Without bending knees  Can you arch your back? Without bending knees (stand supported by the bed foot: will not fall, less possibility of knee bending)  Slide your arms on both sides (Rt and Lt)? (stand against wall, normally the tips of finger travel > 10 cm)  Cross your arms? Turn to the Rt and Lt (pt sitting on bed)  Modified Schober's test: (midline, between the dimples of Venus) + 5 cm below + 10 cm above  bend forward  N> 6 cm diff. Special tests  Occiput-to-wall distance (tragus & nose same level): normally zero  Straight leg raise (irritation of the roots of sciatic n: L4/L5/S1/S2): elevate the lower extremity straight, when it is painful  where it does hurt?  straight leg test positive  Decrease the angle, try to dorsiflex foot  Lasègue sign  Cross straight leg raise test: elevate the other LL  trigger pain  Faber’s test (figure 4 test): to check sacro-iliac joint pathology  Femoral nerve stretch (done for patients c/o pain radiating to the anterior aspect of the thigh): patient prone, knee flexed, Motor  Hip flexion (L1/L2/L3) / extension (S1/S2)  Knee flexion (L5/S1/S2) / extension (L2/L3/L4)  Ankle dorsiflexion (L4/L5) / plantar flexion (S1/S2)  Can you walk on your heals? Normal L4/L5 muscles  Can you walk on your toes? Normal S1/S2 muscles Sensory S1: little toe L5: first web L4: medial malleolus L3: knee – med L2: thigh – ant L1: groin T10: umbilicus Reflexes Knee (L2/L3/L4 – mainly L4) / Ankle (S1/S2 mainly S1) / I would like to do the Babinski reflex (positive in UMNL) Pulse Dorsalis pedis Other clinical examinations: DRE; to rule-out cauda equina (sphincter weaknesses, reduced anal tone) N.B. dimples of Venus correspond to PSIS

OSCE-guide-III.doc

Page 140 of 255

Physical Examination

Ankle Twist Young man comes with ankle twist; history and physical examination are normal, no fractures, and no lacerations. In the next 10 minutes counsel him about the treatment History

Mechanism of trauma / injury Noise heard at the time of trauma  Where you able to walk after the injury? Symmetry / SEADS  Temperature  Tenderness: med malleolus / lat malleolus / ant joint line (between malleoli) / lat ligaments (slightly ant and inferior to the lat malleolus) / med ligaments (inferior and slightly ant to the med malleolus) / tendon Achilles / navicular head (medial bony prominence) / fifth meta-tarsal head (lat) / around the meta-tarsal heads / origin of the plantar fascia (inf med aspect of calcaneus )  Crepitus Patient is sitting on the side of the bed  Dorsi-flexion (point toes up) and plantar flexion (point toes down)  Inversion (point the bottoms of your feet towards each other) and eversion (the opposite direction) ROM against resistance / after stabilizing the ankle with your other hand  Talar drawer sign: 1 cm is significant, indicates ant talo-fibular lig rupture  Talar tilt: lat calcaneo-fibular lig rupture (inversion), med lig rupture (eversion)  Squeeze test (squeezing calf  ankle pain); if negative  NO ankle fracture  

Inspection Palpation

ROM

Power Special tests

Investigations: x-ray  Ottawa ankle rules; for ankle series: o Pain in the malleolar zone and any one of the following:  An inability to bear weight both immediately and in the emergency department for four steps  Bone tenderness along the tip of the medial or lateral malleolus  Ottawa foot rules; for foot series o If there is any pain in the mid-foot zone and any one of the following:  An inability to bear weight both immediately and in the emergency department for four steps  Bone tenderness at the base of the fifth metatarsal  Bone tenderness at the navicular bone Management:  Complete tear should be evaluated by orthopedics  stat orthopedics consult  RICE: rest (and crutches) / ice for 20 min QID x 3 days / compression (by tensor bandage) / elevation  Pain medication: NSAIDs; e.g. Ibuprofen 400 mg, PO, q6h.  Show him how to wrap it, remove the wrap, and ask him to wrap it again (to make sure he knows how to). Remember: from distal to proximal and 1/3 width overlap.  Show him how to use the crutches.

OSCE-guide-III.doc

Page 141 of 255

Physical Examination

Shoulder Joint History - Trauma to shoulder / neck? X-ray done? What is your occupation? - Neurological deficits? How does it affect your life? Vital signs General Patient condition (restlessness, discomfort, willingness to move) Inspection - Both shoulders symmetrical / clavicle level / scapula level / deltoid - SEADS (Swelling / Erythema / Atrophy / Deformity / Scars) Palpation - Temperature: compare - Tenderness: sternal notch / sterno-clavicular joint / clavicle / acromioclavicular joint / deltoid / long head of the biceps / insertion of the rotator cuff muscles / spine of the scapula / medial border of scapula / spinal processes of the cervical spine - Crepitus ROM - Active ROM: can you copy me please: - Abduction  and comment on painful arc test - Adduction  and comment on drop arm test - Forward flexion (180°) /+/ Backward extension (60°) - External rotation /+/ Internal rotation - Another faster way to check: - Hands behind your neck (abduction / ext rotation) - Hands behind back (adduction / int rotation) between shoulder blades; touch the tip of the contra-lateral scapula. - Passive ROM: If patient is unable to complete the whole range of movements actively, complete the ROM passively and comment (in inflammation: passive ROM is > active ROM) Power - Like the ROM, but against resistance Special tests 1 Painful arc (between 60° and 120°) All these tests are done to 2 Drop arm test  complete tear of supratest for subacromial spinatous tendon impingement of supra3 Neer’s test spinatous 4 Hawkin’s test 5 Jobe’s test (empty can test) 6 Lift-off test: try to push my hand away from Sub-scapularis your back Yergason’s test; palm face up test: shake For bicepital tendinitis hands, try to let your palm face upwards, I will resist you, and press on your shoulder 8 Speed’s test: supine, semi-flexed, do not let me push your arm down 9 Stability testing: For joint stability + Push ant / post + Pull down  sulcus sign 10 Apprehension test (ant and post): for dislocation To complete my exam, I would like to do: - Check the pulses of the upper limb (radial / ulnar / brachial) - Brief neurological examination of the upper limb - One joint above and one joint below examination (cervical spine / elbow) - The other shoulder examination 7

OSCE-guide-III.doc

Page 142 of 255

Physical Examination

Rotator cuff muscles: Supra-spinatous Infra-spinatous Teres minor Sub-scapularis

Abduction External rotation Internal rotation

Lift-off test

Impingement syndrome: - The most common symptoms in impingement syndrome are pain, weakness and a loss of movement at the affected shoulder Treatment: - Mild: RICE / NSAIDs / PT. Rest (cessation of painful activity), ice packs and NSAIDs may be used for pain relief. Physiotherapy (PT) focused at maintaining range of movement and avoiding shoulder stiffness. - Moderate: therapeutic injections of corticosteroid and local anesthetic may be used for persistent impingement syndrome - Severe: surgery Investigations: - U/S Possibilities: - Normal shoulder exam - Frozen shoulder -

-

Bicepital tendinitis Repeated ant dislocation Rotator cuff tear o Complete tear o Partial tear Rotator cuff tendinitis Sub-deltoid bursitis

stiff, with limited active and passive ROM (ttt: physiotherapy, NSAIDs, steroids) +ve palm face up test / speed test positive apprehension test drop arm test (ttt: surgery) pain with initiation of movement / +ve empty can test (ttt: physiotherapy, NSAIDs, steroids, surgery) similar to partial tear / +ve impingement test

Elbow ROM: flexion / extension /+/ pronation / supination Tennis elbow (lateral epicondylitis)  With the elbow fully extended, there are points of tenderness over the lateral epicondyle (origin of the extensor carpi radialis brevis muscle).  Cozen's test: pain with passive wrist flexion and resistive wrist extension.[  X-rays are used to confirm and distinguish possibilities of existing causes of pain that are not related to Tennis Elbow, such as fracture or arthritis. Golfer's elbow (medial epicondylitis) The common tendinous sheath is inserted into the medial epicondyle of the humerus  Treatment: o NSAIDs: ibuprofen, naproxen or aspirin /+/ Heat or ice o A counter-force brace or "elbow strap" to reduce strain at the elbow epicondyle, to limit pain provocation and to protect against further damage. 

OSCE-guide-III.doc

Page 143 of 255

Physical Examination

Hip Joint + Middle-age male with septic arthritis + Elderly female with osteoarthritis Vital signs General

-

Patient condition (restlessness, discomfort, willingness to move) May I ask for full exposure please? Inspection Hip joint is deeply seated joint, I am looking for the surroundings SEADS (Swelling / Erythema / Atrophy / Deformity / Scars) Scoliosis / kyphosis / pelvic tilt (level of both iliac crests) Gait: no wide stance, shuffling, drop foot, or antalgic gait Balance: Trendelenberg sign; standing on one leg (while the patient is putting his arms on the examiner shoulders), the pelvis drops Palpation - Temperature: compare - Tenderness: ASIS, iliac crest, PSIS, sacro-iliac joint, greater trochanter of the femur. I would like to check symphysis pubis and inguinal ligament. - Crepitus: over femoral head (lat to femoral art, below inguinal lig) ROM Active ROM, each one followed immediately by passively stressing (increasing) the ROM – while patient is lying - Forward flexion (120°) - Internal rotation (30°) /+/ External rotation (45°) - Create space, stabilize the contra-lateral hip with your left hand: Abduction (45°) /+/ Adduction (30°) - Backward extension (while lying prone): stabilize the lower back by your left hand, can you lift your thigh (20°) Power - Resisted isometric testing (patient lying supine) Special tests - Figure 4 test (Patrick or Faber’s test)  the leg of the examined side flexed and externally rotated with the ankle resting on the patella of the contra-lateral leg. The examiner applies counter-pressure at the opposite hemi-pelvis, and applies gentle downward force on the knee. Post hip pain indicates sacro-iliac joint pathology, while ant lat hip pain may suggest hip joint pathology - Thomas test  put your hand under pt LSS, and try to max flex the contra-lateral knee - True leg length  from ASIS (anterior superior iliac spine) to medial malleolus – on both sides - Apparent leg test  from umbilicus to medial malleolus To complete my exam, I would like to do: - Check the pulses of the lower limb (dorsalis pedis / posterior tibial / popliteal) - Brief neurological examination of the lower limb - One joint above and one joint below examination (LSS / knee) - The other hip examination Septic arthritis: Physical exam: fever / very painful joint / +ve trendlenberg test / restricted movements on all directions DD: Septic arthritis / Osteoarthritis / Osteomyelitis One diagnosis: septic arthritis / One diagnostic test: arthrocentesis (joint aspirate) Management: ▬ IV antibiotics, empiric therapy, (based on age and risk factors; oxacillin [2 g IV q4h for 4 weeks], or vancomycin [if suspecting MRSA; 20 mg/kg IV q8h, for 8 wks], combined with ceftriaxone for gram –ve, if suspecting Gonococcal: ceftriaxone; IV for 2 wks then oral for 2 wks), adjust pending C&S ▬ For small joints: needle aspiration, serial if necessary until sterile ▬ For major joints such as knee, hip, or shoulder: urgent decompression and surgical drainage

OSCE-guide-III.doc

Page 144 of 255

Physical Examination

Knee Joint Inspection

Palpation

ROM

Power Special tests

Vital signs, General - Gait and stance: normal; no antalgic (painful) gait - Bilateral joint exposure (quadriceps) - SEADS (Swelling / Erythema / Atrophy / Deformity / Scars) - No genu varum (bow legs) and no genu valgum (knock-knee) deformities - Temperature: compare - Extended knee: tenderness over patella /+/ Lateral movement of patella /+/ quadriceps muscle / quadriceps tendon / patellar ligament / Tibial tuberosity / popliteal fossa and popliteal artery - Flexed knee: tibial plateau / bilateral joint lines /+/ Collateral ligaments /+/ Femoral condyles /+/ patellar crepitus - Popliteal fossa /+/ Cuff muscles [slightly flexed knee] Knee effusion: - Fluid wave or bulge sign (or milking test): for small amount of effusion; from below and med to upward and lat. Then immediately sweep hand down the lateral aspect pushing the fluid back - Fluid ballottement test: for moderate amount of effusion - Patellar tap: for large amount of effusion Patient lying down: - Flexion (130°) and extension (180°) - Internal and external rotations: while knee is flexed 90°, point your toes in & out please - Patellar movement: medial and lateral - Patellar compression test: tight your thigh please  rough or painful movement: patello-femoral syndrome or osteoarthritis - Flexion and extension, while the knee is flexed 90° - Anterior drawer test  for anterior cruciate ligament tear - Posterior drawer test  for posterior cruciate ligament tear - Lachman test: hip / knee semi flexed (30°)  ACL tear - Check for the medial / lateral collateral ligaments (stability of knee); while flexed at 30°  no laxity nor pain - McMurray’s test (for medial and lateral meniscus tears)  feel for crepitus / patient feels pain -

-

For medial: maximally flexed knees, externally rotated foot  extend while applying varus force (from inside outwards) For lateral: maximally flexed knee, internally rotated foot  extend while applying valgus force (from outside inwards)

To complete my exam, I would like to do … -

Painful clicking  cruciate, meniscus Knee lock  torn meniscus Instability  cruciate 

DD Tenderness Investigations Treatment

OSCE-guide-III.doc

ACL - Knee giving way - Inability to continue activity

MCL - Can not descend stairs

Osgood–Schlatter disease Chondromalacia patellae  Pain on tibial tuberosity (↑ by kneeling)  Pain on lateral movement of patella X-ray (AP / LAT / skyline)  Benign self-limited condition  Non-impact activities  Continue activity as tolerated  NSAIDs  NSAIDs  Physiotherapy  Physiotherapy  Surgery for refractory cases

Page 145 of 255

Physical Examination

Obstetrics and Gynecology

OSCE-guide-III.doc

Page 146 of 255

Physical Examination

History taking – OB-GYN Introduction CC Analysis of CC

HPI

Os Cf D COCA ± Blood ↑ ↓ Associated symptoms DD Menstrual M Gynecological G Obstetric O Sexual S

PMH FH SH

OB/GYN cases History taking: - Vaginal discharge - Vaginal bleeding - Amenorrhea - Infertility Counselling: - OCPs - HRT - C-section (wants to have c-section or wants to have vag delivery after c-section) - Abortion - 22 years old pregnant  anti-natal counselling - 39 years old found she is pregnant, counsel her - 30 yrs old pregnant (36 wks), HTN/+++ ptn in urine  counsel for pre-eclampsia - PAP smear; 16 years old wants to arrange for a PAP smear - PAP smear: 38 year old had abnormal PAP smear

OSCE-guide-III.doc

Page 147 of 255

Physical Examination

MGOS history questions: Menstrual: -

When was your LMP? First day? Was your LMP similar to the previous ones? Are they regular or not? How often do you have periods? How long does it last? How many days? How about the amount? Is it large / small? How many pads/day? Any blood clots? Are your periods painful? [not painful  anovulatory (PCOS/infertility)] Any spotting / bleeding between periods? When was your first period? Was it regular? For how long it was not regular? Normal to be irregular for up to 18-24 months.

Gynecological: -

Do you have history or were diagnosed with any gynecological disease (e.g. polyps)? Do you have history of pelvic surgery or instrumentation (e.g. D&C)? Do you use contraception? What method? Since when? When was the last time?

Screening: -

Have you ever had Pap smear before? When was the last time? Any reason (if long time)? What was the result? (>40 yrs) have you had mammogram done before? When? (Is it painful doctor? Could be; we need to apply pressure on the breast to get better image) (>65 yrs) have you had your bone mineral density (BMD) done? Any reason?

Obstetrical  GTPAL: -

Have you ever been pregnant before? Any abortions (termination)? Or miscarriages (spontaneous abortion)? Number of babies you delivered? Any twins? Any children with congenital abnormalities? For each delivery: was it full term or pre-term? Vaginal or CS? Any complications like high blood pressure / high blood sugar? Family history of: repeated abortions / CS / congenital anomalies / twins

Sexual history: -

-

With whom do you live? If (alone / with family): are you in any relationship? Are you sexually active? Have you ever been sexually active? If with partner: how do you describe the relationship? Is it stable? Are you sexually active? Do you practice safe sex, and by that I mean using condoms every time? For how long you have been together? (> 6 months  stable). And before that, were you sexually active? When did you start sexual activity? How many partners have you had for the last 12 months? For the last month? What is your sexual preference? Men/ women/ both? What type of sexual activity? Have you screened or diagnosed before with STIs? HIV? Vaginal discharge? How about your partner? Any fever? Discharge? Burning sensation? Do you feel safe in this relation?

What if the male partner does not like condoms? Is it ok to consider it safe sex? Yes, provided that: - Scan the partner for STIs first - Strict monogamy relation (no extra-marital affairs) - Use alternative reliable contraception (e.g. OCPs)

OSCE-guide-III.doc

Page 148 of 255

Physical Examination

History of pregnant lady – third trimester Are you doing regular ante-natal follow-up visits?

NO

Yes

Social issue

Deal with the social issue

Last visit history / pre-eclampsia

  

Make sure the mother is stable

  

Make sure the baby is stable U/S

    

When was your last f/u visit? What was your BP? Was there any headache? Was there leg swelling? Weight gain? Any abdominal pain? Cramps? Vaginal bleeding? Discharge? Any gush of water? Is your baby kicking like before? > 6 in 2 hrs Have you done your U/S? How many times? When was the last time? Number of babies? Location of the placenta? Amount of fluids?

N.B. to make sure the mother and baby are stable: ABCDE  Activity of the baby  Bleeding  Contractions / pain  Dripping / Discharge  EDD (expected date of delivery)

OSCE-guide-III.doc

Page 149 of 255

Physical Examination

Vaginal Discharge Teenager / 5 minutes case CC How can I help you?! Analysis of CC Os Cf D COCA ± Blood / color / fishy odour? - ↑ ↓ - Related to periods - Related to sexual intercourse (bact vaginosis: ↑ discharge post-coitus) - LMP / regular / how often / similar to previous ones? M Same system HPI - Any pain? With intercourse? AS - Itching? Redness? ? Candida DD - Any blisters / warts / ulcers13? - Inguinal swellings? - Urine changes? Dysuria, frequency? Nearby systems - Bowel movements changes? GIT symptoms - Abdominal pain  OCD / PQRST / ↑↓ - ? PID  Adnexal tenderness / fever - Dissemination to liver (pain Rt upper abd) DD - Constitutional symptoms - Sore throat? Mouth ulcers? Red eyes? - Joint swelling/pain? Skin rash? Reiter’s G - IUD - PAP smear! - History of STI / PID? O Complete sexual history for both partners S PMH - Any medications? Recent use of antibiotics - Allergies - DM FH / SH - How do you support yourself? - HEAD SS / SAD Conclusion: STI because of risky sexual behaviour  Physical examination including pelvic, speculum exam / PAP smear / swabs for C&S including those for Chlamydia & Gonorrhea / saline slide microscopy / KOH / Whiff test  DD: Gonorrhea, Chlamydia, Candidiasis (whitish), Bacterial vaginosis (thin gray, clue cells), Trichomonas (frothy yellowish / greenish discharge, motile organism).  Treatment: o Gonorrhea: Ceftriaxone 250 mg IM single dose o Chlamydia: Azithromycin 1g orally single dose o Candidiasis: Miconazole 200 mg vag supp, 1 vag supp od qhs x 3 d o Bacterial vaginosis: Metronidazole 500 mg PO bid x 7 d If pregnant: Amoxicillin 500 mg PO tid x 7 d o Trichomonas: Metronidazole 500 mg PO bid x 7 d  Follow up with in 4 weeks  Her partner(s) to be notified and to come for treatment, ask about sexual health (fever, discharge)  Advice regarding safe sex (condoms, multiple partners, STIs)  Chlamydia and Gonorrhea are reportable diseases  HIV testing and other STIs screening if high risk sexual behaviour  Advise regarding PAP smear regularly, vaccination against HPV 13

Blisters: HSV (Herpes Simplex Virus) / warts: HPV (Human Papilloma Virus) / ulcers: syphilis

OSCE-guide-III.doc

Page 150 of 255

Physical Examination

Vaginal Bleeding – Non-Pregnant / Not-Known Pregnant Analysis of CC Timing: OS Cf D – When did it start to be continuous? COCA / fresh blood vs. clots  if large amounts  impact  anemia / dehydration symptoms (pallor, SOB, dizziness, fainting, heart racing) - ↑ ↓ - Related to sexual intercourse (if yes: cervical*) - For the last few weeks, how do you distinguish between your regular M periods and the bleeding? Related to periods - LMP / regular / how often / how much / similar to previous ones? - First menstrual period? Regularity? HPI - Any chance you are pregnant? How do you know for sure? Same AS system - Any nausea / vomiting / breast engorgement? Frequency? DD - Any pain? With intercourse? - Itching? Redness? - Any blisters / warts / ulcers? - Grape like tissue (hydatiform mole) - Urine changes? Urinary symptoms? Nearby - Bowel movements changes? GIT symptoms systems - Abdominal pain: OCD/PQRST (? PID / ectopic) - Pelvic fullness / heaviness? - Constitutional symptoms DD - Bleeding disorders/ tendencies? Any bleeding elsewhere (nose, gums, with stool, easy bruising)? - Blood thinners? Aspirin? - Thyroid problems? Symptoms? - Polyps, fibroids, endometriosis, gyn cancer? G - Hx pelvic surgeries? Instrumentations? - Contraception history; OCPs / IUD / HRT / mammogram - PAP smear! Was it normal? - Any previous pregnancies? Abortions? How many? Route O of delivery? - Number of partners / safe sex / when did you start activity? S - Hx of STIs PMH - Breast cancer, mammogram (if > 40 years), HTN FH - Gynecological cancer / Breast cancer SH - How do you support yourself? - SAD / HEAD SSS (if teenager) Vaginal bleeding cases: -

-

Middle age / risky behaviour / old abnormal Pap smear  cervical cancer. A 48 years patient with vag bleeding and all symptoms will be negative  dysfunctional uterine bleeding (DUB); intermittent / lose track of periods / no pain with periods. A 52 years pt / constipation / OCP/HRT / no pregnancies14  endometrial cancer / constitutional A 62 years pt with intermittent bleeding / small amount with secretions  atrophic vaginitis; (menopausal symptoms / dyspareunia / itchy vulva)  Rule out cancer (endometrial biopsy)  estrogen vaginal cream DD: Fibroid / Cervical polyp / hyper/hypo-thyroidism / Trauma / Coagulopathy / PCOS

Investigations: pregnancy test β-HCG / progesterone challenge test / hysteroscope / PAP / U/S / endometrial biopsy / TSH 14

Cervical cancer for prostitutes (risky behaviour) and endometrial cancer for nuns (no pregnancies)

OSCE-guide-III.doc

Page 151 of 255

Physical Examination

Vaginal Bleeding – Pregnant / Ante-Partum Hemorrhage   

Patient usually sleeping in a left lateral position with oxygen, very anxious, crying Reassure her & ask her to bear with you for a while to get to the bottom of her problem Ask about her feelings & empathize (impact on her and her partner)

Analysis of CC

HPI

M AS DD

G O S PMH

FH SH

Timing: OS Cf D first time or happened before COCA / fresh blood vs. clots  if large amounts  impact  anemia / dehydration symptoms (pallor, SOB, dizziness, fainting, heart racing) ↑ ↓ - LMP / regular / how often / how much / similar to previous ones? - How many weeks? date based on LMP Same - How did you know that you are pregnant? Was it planned? system - Did you have regular antenatal care? F/U visits? U/S? - What is the result of U/S? - Any pain / discomfort? (OCD / PQRST)? Contractions - Water gush / grape like tissue (hydatiform mole) - Fetal movement - Urine changes? Urinary symptoms? Nearby - Bowel movements changes? GIT symptoms systems - Abdominal pain: OCD/PQRST (? PID / ectopic) - Pelvic fullness / heaviness? - Constitutional symptoms DD - Bleeding disorders/ tendencies? Any bleeding elsewhere? - Headache, hand or feet swelling - Trauma, sexual intercourse - Polyps, fibroids, endometriosis, gyn cancer? - Hx pelvic surgeries? Instrumentations? - PAP smear! Was it normal? - GTPAL - Any previous pregnancies? Abortions? How many? How many weeks? Route of delivery? - Hx of STIs - Breast cancer, mammogram - HTN, DM, kidney disease, blood gp & Rh - Medication / allergies / hospitalization / surgeries / blood transfusion. - Gynecological cancer / abortions - How do you support yourself? - SAD / HEAD SSS (if teenager)

Threatened abortion Separation of part of the placenta Will do physical and obstetric examination. Vaginal exam only after U/S excludes placenta previa - 50/50% chance to keep or loose the baby - Admit to the hospital - Investigations: continuous vitals monitoring / CBC / INR / PTT / - Arrange for continuous fetal heart monitoring, U/S fibrinogen / Rh status / blood grouping and cross matching / US / and biophysical profile fetal monitoring - Management: O2, IV fluid, LLP, if fetus is still in - Management: distress, arrange for C/S - If she is improving / the fetus is not distressed: she will go home, Placenta previa Placenta abruption resume normal activity, come back if more blood or more pain Painless Painful (contractions) - Otherwise: bed rest / steroids / fetal monitoring / Rhogam / platelets Incomplete abortion: 3 findings on vag exam would confirm the diagnosis: (1) Cervix dilated, (2) Ruptured membranes, (3) Product of conception passed

OSCE-guide-III.doc

Page 152 of 255

Physical Examination

Abnormal Uterine Bleeding (AUB)



Dysfunctional Uterine Bleeding (DUB): Abnormal bleeding not attributable to organic (anatomic / systemic) disease. DUB is a diagnosis of exclusion. Anovulatory AUB often used synonymously with DUB. ─ Causes: o ↓ progesterone: luteal phase defect (estrogen-dependent DUB) o PCOS o Endocrinal (↓ TSH / ↑ prolactin) o Stress, weight loss, exercise o Liver and kidney disease ─



Investigations for AUB: • o o

• • o o o

• • • o o

o

Beta-hCG CBC, serum ferritin Coagulation profile (esp. adolescent): rule out von Willebrand's disease TSH, free T4 Prolactin if amenorrhea FSH, LH Day 21 (luteal phase) progesterone to confirm ovulation Serum androgens (especially free testosterone) Pelvic U/S: detect polyps, fibroids; measure endometrial thickness (postmenopausal) Sonohysterogram (SHG): very sensitive for intrauterine pathology (polyps, submucous fibroids} Hysterosalpingography (HSG) Pap test Endometrial biopsy: women > 40 years are at higher risk of endometrial cancer  Must do endometrial biopsy in all women presenting with postmenopausal bleeding to exclude endometrial cancer D&C: not for treatment; diagnosis only (usually with hysteroscopy)

OSCE-guide-III.doc

Page 153 of 255

Physical Examination



Treatment of AUB: Treat underlying disorders / if anatomic lesions and systemic disease have been ruled out, consider dysfunctional uterine bleeding (DUB) ─ Medical: o Mild DUB ─

      o

NSAIDs Anti-fibrinolytic (e.g. Cyklokapron) at time of menses Combined OCP Progestins (Provera) on first 10-14 days of each month if oligomenorrheic Mirena IUD Danazol (pseudo-menopause)

Acute, severe DUB  



Replace fluid losses, consider admission Medical treatment: ─ (a) estrogen (Premarin) 25 mg IV q4h x 24h with Gravol 50 mg IV/PO q4h or ─ (b) Ovral15 1 tab PO q4h X 24h with Gravol 50 mg IV /PO q4h Taper Ovral: 1 tab tid X 2d  bid X 2d  OD After (a) or (b), maintain patient on monophasic OCP for next several months or consider alternative medical treatment

Clomiphene citrate: consider in patients who are anovulatory and who wish to get pregnant Surgical: o Endometrial ablation; consider pre-treatment with danazol or GnRH agonists o



  o

If finished childbearing Repeat procedure may be required if symptom recurrence

Hysterectomy: definitive treatment

Post-coital bleeding / middle age: ─ STIs ─ Cervical (Cervicitis / Cervical polyp / Cervical cancer / Ectropion; if OCPs) ─ Bleeding tendencies ─ Trauma Management: ─ Vaginal and cervical swabs; C&S and gram stain ─ Polyp: Sonohysterogram ─ Biopsy ─ If ectropion: stop OCPs, cautery (silver nitrate) Cancer uterus: Risk factors: ─ ─ ─ ─ ─

Early menarche Nulliparity Weight gain HRT / estrogen therapy Unopposed estrogen

Management: ─ Endometrial biopsy; if positive o Total abdominal hysterectomy and bilateral salpingo-oophorectomy o Adjuvant chemotherapy 15

Ovral is progestin (levonorgestrel) and estrogen (ethinyl estradiol) combination OCP

OSCE-guide-III.doc

Page 154 of 255

Physical Examination

Amenorrhea CC

Did not have periods for 6 months?! Did you seek medical attention? Any recent changes? Analysis of CC During these 6 months; any irregular bleeding? Spotting?  When was your first period? What age? Was it regular? For how long it was M regular / not regular? How often? How much? LMP?  When it was regular; was it painful? (painless  anovulatory)  Did you use any contraception? When did you stop? Why? HPI AS  Any chance you are pregnant? How do you know for sure?  Any nausea / vomiting? Breast engorgement? Frequency? DD  For how long have you been trying to get pregnant?  Any previous pregnancies? Abortions?  Constitutional symptoms?  Are you under stress? Hypothalamus  Excessive exercise?  Any concerns about your weight? (anorexia)  Any headache? Vomiting in morning? Visual changes? Pituitary Difficulty seeing to sides? Milk secretions from breast?  History of thyroid disease? Heat/ cold intolerance? Bowel movements? Moist/ dry skin?  Do you have excessive hair growth? Acne? Did you Ovarian notice any weight changes? Hx of DM / thirsty / frequency? Fm Hx of PCOS?  Hx of chemotherapy? Radiotherapy? Hot flushes? Vaginal dryness? Soreness?  Any change in your voice? ↑ Muscle bulk?  Any repeated surgical procedures? D&C? Uterine G  Pelvic surgeries? Instrumentations?  PAP smear!  Any previous pregnancies? Abortions? O S  Hx of STIs PMH - Any medical conditions? Psychiatric illness? - Any medications? Recent use of antibiotics FH - Family hx of PCOS / infertility? SH - How do you support yourself? - SAD  Investigations: β-HCG / progesterone challenge test / hormonal assay (estrogen / progesterone / FSH / LH / prolactin / thyroid-TSH / serum testosterone; total and free) / US / CBC  Ovarian causes of amenorrhea: PCOS /+/ Premature ovarian failure /+/ Androgen-producing tumours  PCOS (poly-cystic ovarian syndrome): ─ History: previous pregnancy / contraception hx ─ To diagnose PCOS: must have 2 of 3 criteria: (1) oligomenorrhea / irregular menses for 6 months, (2) hyper-androgenism (hirsutism or ↑ blood level), (3) PCOS by US ─ Investigations: β-HCG / US / High LH:FSH ratio > 2:1 / Fasting blood sugar ─ Treatment: o Lifestyle modification (↓ BMI, ↑ exercise) to ↓ peripheral estrone formation o Metformin 500 mg PO tid o Clomiphene citrate; if she wants to become pregnant o Tranexamic add (Cyklokapron); for menorrhagia only o OCPs; if she does not want to become pregnant  Dysmenorrhea: ─ DD: PID, fibroid, endometriosis ─ Investigations: U/S to exclude other conditions ─ Treatment: NSAIDs (ibuprofen 400 mg tid), OCPs.

OSCE-guide-III.doc

  

Page 155 of 255

Physical Examination

Infertility Transitional statement before going in details with the history: In order for a couple to achieve pregnancy, both partners should be capable of having children and relatively healthy. For that reason, I am going to ask some questions about your health and your partner health; some of these questions are personal, but it is important to ask. And I would like to assure you that all the conversation is strictly confidential and I will not release any information without your permission! Introduction CC Analysis of CC For how long have you been trying to conceive? - Anovulation (irregular cycles / painless / no pre-menstrual syndrome) HPI DD - PCOS: Do you have excessive hair growth? Acne? Did you notice any weight changes? Hx of DM / thirsty / frequency? Fm Hx of PCOS? - Tubal occlusion: surgeries / STIs and PID / IUD - Endometriosis: dysuria / dyspareunia / dyschezia / back pain - History of chemotherapy and radiotherapy - Do you have hx or were diagnosed with any gyn disease (e.g. polyps)? Gynecology - Did you use contraception? What? Since when? When was the last time? Hx - Have you ever had Pap smear before? When was the last time? Any reason (if long time)? What was the result? - Have you ever been pregnant before? Any abortions (termination)? Or Obstetric Hx miscarriages (spontaneous abortion)? - Family history of: repeated abortions / CS / congenital anomalies / twins Coital history - For how long are you in this relation? For how long have you been trying to achieve pregnancy? - How frequent? Regularity? - Are you aware of sexual cycle and ovulation (both you and your partner) - Was he sexually active before? Did he father kids from another partner Partner before? history - Did he receive any chemo or radio therapy? - Does he complain of any testicular problem; varicose veins, inflammation? - Does he complain of any penile discharge? - Did he go through any investigations; e.g. semen analysis? PMH - Allergies - DM FH Infertility SH Investigations: ─ Semen analysis ─ Ovulation documentation (mid-luteal phase progesterone; d 21-22 / US) ─ Tubal patency (HSG / laparoscopy) When to start investigations: ─ If the woman is < 35 years  after 12 months of trying to conceive ─ If the woman is 35-40 yrs  after 6 months of trying to conceive ─ If the woman is > 40 years  start investigations after 1 months of trying to conceive

OSCE-guide-III.doc

Page 156 of 255

Physical Examination

Counselling – pre-eclampsia 36 weeks pregnant lady comes for f/u visit, BP 160/110, +++ protein in urine, Manage. Like the B12 results case Introduction I will discuss results with you Ethical challenge: travel permission History Last visit history / pre-eclampsia Make sure the mother and baby are stable U/S Obstetric history / Gynecological history PMH / Social history Counselling Explain what is pre-eclampsia Serious concerns with pre-eclampsia Management Hospitalize If insisting to leave  sign a LAMA Introduction - Good afternoon Ms … I am Dr … I understand that your blood pressure was measured and urine test was done, I have the results with me and I will discuss it with you. However, because this is my first time to see you, I need to ask you some questions, to get a better understanding of your health condition, is that ok with you? - Is this you first time to have these checks during your pregnancy? - Are you under regular follow-up? o Yes  proceed to history o No  any reason? My husband had a car accident! I am sorry to hear that; was he hurt? Was anyone else hurt? When was that? It must be difficult, how did this affect your life? Ethical challenge: travel permission o Actually I am here to get a note. - What type of notes? o Travel note, I really need to travel. - It looks like it is an important trip for you; usually pregnant ladies do not travel during this time of pregnancy! o It is a business trip that would save our financials. - I see it is important for you, however, before we proceed, let me check your health condition first, and I will start by asking you some questions: History Last visit history / pre-eclampsia - When was your last f/u visit? - What was your BP? Was there any headache? - Was there leg swelling? Weight gain? Did they do urine test? - How about before being pregnant? Any hx of high blood pressure? Make sure the mother and baby are stable: ABCDE - Activity of the baby, is your baby kicking like before? - Bleeding - Contractions / pain - Dripping / Discharge - EDD (expected date of delivery) U/S - Have you done your U/S? How many times? When was the last time? - Number of babies? - Location of the placenta? - Amount of fluids? Obstetric history: any pregnancy before / any similar conditions? Gynecological history PMH: high blood pressure Social history: SAD / support / home environment

OSCE-guide-III.doc

Page 157 of 255

Physical Examination

Counselling Explain what is pre-eclampsia - Your blood pressure is 160/110, which is high, and the urine test shows protein in large amount (+++) which is not normal, the most likely diagnosis is a medical condition called “pre-eclampsia” OR “pregnancy-induced hypertension”. - I would like to ask more questions to see how it affected you! o My dad had HTN, and lived with it, I am ok. - These are different conditions; your dad had HTN, but you have “pregnancy-induced” HTN, which is a serious condition, with very serious and may be fatal consequences. o Have you had hx of headache?  OCD / PQRST (not detailed) o Nausea / vomiting o Change in your vision? Flashing lights? Flying objects? o Any abdominal pain in your upper right part of your abdomen? o Any bruises? Yellowish discoloration / itching / dark urine / pale stools? o Any chest pain / heart racing / SOB? o Any weakness / numbness? o Any swelling in your body / face/eyes? Did you feel your shoes tight? o Did you gain weight? o Any changes in the urine? Frothy? Burning sensation? - Based on all this, the most likely explanation for your increased is “pre-eclampsia”; and this is a very serious condition, we need to admit you to the hospital to monitor you. Then, the obstetrician will assess you and may consider delivering the baby now. o But doctor, I need to travel, just 2 days and I will come back. - I understand your concern about traveling, but we have a serious situation here. - We do not know exactly why patients have pre-eclampsia. We believe it is imbalance of hormones, or it might be related to placenta, however the only treatment is delivering the baby. Serious concerns with pre-eclampsia - What happens is that there is a narrowing of blood vessels, this leads to ↓ the amount of blood reaching the baby, subsequently ↓ the amount of oxygen and nutrients. On the long term this will lead to some injury and even damage to the baby AND the mother. o This includes your heart and blood vessels, that is why you have ↑ BP, o This includes your kidney, that is why you have +++ protein in urine, o This includes your liver, that is why you may have abdominal pain, o This includes your brain, that is why you have headache, visual changes, o This includes your baby, that is why he is not kicking like before … This is not because of your pregnancy; all of these are due to this condition. - The concerns we have is that we can not predict the outcome, without the proper medical care, patients having pre-eclampsia will end up going to the next stage which is “eclampsia”; do you any idea what is “e0clampsia”? - A condition in which, the patient will start to seize, lose conscious, will not be able to breath and turn blue. The only resolution for this is delivering the baby. - Imagine that I give you the note, and they allow you to take the trip, 2 hours later while you are in the plane, you start to fall down and seize. What will happen? Nobody will be able to help you. - By this you endanger your life and your baby’s life. Management - What we need now is to admit you to the hospital and arrange for obstetrical assessment. - If insisting to leave  sign a LAMA (leaving against medical advice) - Suggest solution for her business travel, like giving a doctor note that she needs to be hospitalized. Treatment Plan - Assess severity including good history and physical exam focusing on heart, lungs, reflexes, fetus, urine analysis and BW (important CBC, liver function tests, Uric Acid) - If all above are stable, consider daily check, urine dips and fetal kick counts as outpatient. If any of above unstable may need to hospitalize as inpatient for close monitoring - Measure L/S ratio of the baby, give corticosteroids for lung maturation - MgSO4 and delivery - Blood pressure controlled often with labetalol, Ca channel blockers

OSCE-guide-III.doc

Page 158 of 255

Physical Examination

Caesarean Section – Counselling – wants to have CS Young 18-20 years old pregnant lady would like to have CS, counsel her. Introduction Any reason you want to have CS? Social issue History Last visit history / pre-eclampsia Make sure the mother and baby are stable U/S Obstetric history / Gynecological history PMH / Social history Counselling Address patient concerns Why not caesarean section? Management Refer to obstetrician Spend some time to think / stabilize Introduction Any reason you want to have CS? - I understand that you are here to discuss the possibility of CS; we will discuss this in details, but before that I would like to ask you is there any reason you would like to have CS? o I do not want to have this severe pain! - How do you know it is painful? o I had previous abortion OR o I attended my sister delivery and it was very painful experience - When was that? Did you attend? o Congratulations! How is your sister doing? How is the baby? o I understand that you saw her in pain, but people differ! And within few minutes I will be explaining different options to control labour pains! History - Let me ask you some questions to assess the condition first! o How do you feel? How is your mood? o How about this pregnancy, was it planned? How do feel about your pregnancy? How is the feeling of your partner? o Are you under regular follow-up? NO! Any reason? There may be social issue here.  Empathy: it looks like you are doing through difficult times! How are you coping?  Offer social support: being pregnant lady without support, you have priority and there are a lot support and resources in the community. I will make sure to connect you with social worker who will help you with proper support (housing / financially / for both of you and the baby) Last visit history / pre-eclampsia - When was your last f/u visit? - What was your BP? Was there any headache? - Was there leg swelling? Weight gain? Did they do urine test? - How about before being pregnant? Any hx of high blood pressure? - Did you have your blood sugar checked? How about before pregnancy? Make sure the mother and baby are stable: ABCDE - Activity of the baby, is your baby kicking like before? - Bleeding - Contractions / pain - Dripping / Discharge - EDD (expected date of delivery)

OSCE-guide-III.doc

Page 159 of 255

Physical Examination

U/S -

Have you done your U/S? How many times? When was the last time? Number of babies? Position of the placenta? Amount of fluids?

Obstetric history: GTPAL - Any pregnancy before? Any abortions or miscarriages? - What were the circumstances? How many weeks? - How did you feel about it? How did you cope with that? Gynecological history: Fibroid, Genital herpes PMH: Medications / allergy / blood transfusion Social history: SAD / support / home environment Counselling Address patient concerns - I know that you are here to talk about CS. But first let me explain some facts about delivery. The natural route for delivery is the vaginal delivery, and if there is no real indication for CS, we would prefer to go for vaginal delivery. - However, I appreciate you concerns, if your concern is the pain there is a lot of options to control it. o We can start by learning some relaxation techniques o And then on the delivery day, when you go there, there are a lot of support groups; one of them is called “doolas”; they attend with you and they provide a lot of emotional support o Finally, when you start your labour, we have good measures; and I mean what we call “epidural anesthesia”; this is very effective and safe. Where the anesthesiologist puts a needle into your back and injects a freezing substance that helps you to go through the delivery without pain (this is like the dentist freezes your mouth before doing painful procedures). It might however cause headache, bleeding, and less likely infection. Why not caesarean section - I would like to ask you; what is your understanding of CS? - It is commonly used obstetrical intervention, used when there is a problem or contraindication for vaginal delivery and if there is an emergency situation that necessitates immediate delivery; and in these cases it is life saving; for both the mother and the baby! - However, it is a major surgery, has the risks of bleeding, higher risk of infection, DVT, you stay longer in the hospital, and it will leave scar in your abdomen. Management - After all, I am not the person who makes the decision; this should be decided by the obstetrician. - I am going to refer you to the obstetrician; who will perform further and detailed assessment then discuss the results with you. - Meanwhile, I would recommend you spend some time to think about what I told you, try to stabilize yourself emotionally. -

I will give you some brochures and web sites so that you can read more about that. I will connect you with the social worker. And if at any time you have any questions or concerns, you can come to see me.

OSCE-guide-III.doc

Page 160 of 255

Physical Examination

Caesarean Section – Counselling – does not want to have CS Middle aged pregnant lady (36 weeks) is here to have her file. Three years ago, she had urgent CS for cord prolapse. Now she would like to deliver at home with the midwife. Introduction Any reason you want to have your file? Concern Deal with the patient concern What was the type of your CS? History Last visit history / pre-eclampsia Make sure the mother and baby are stable U/S Obstetric history / Gynecological history PMH / Social history Counselling What is CS? The two types of CS Risks of vaginal delivery post CS Management Prepare a copy of the file Speak with your midwife Introduction Any reason you want to have your file? - I understand that you are here to have a copy of your file for the urgent CS you had 3 years ago. This is your right, and I will ask someone to prepare a copy for you. - But first I would like to ask you some question, is it ok with you! - Is there any reason you would like to have your file? o Yes, I would like to have delivery at home this time. A lot of my friends did it at home with the midwife and they say it is much easier and relax. - Are you seeing obstetrician? Are you doing any regular follow-up visits? Any reason for that? o No, I am going to follow up with the midwife. o I did not like last time when they did CS at the hospital! - What happened last time? o They told me the baby had cord prolapse! - Were there any consequences? o How is the baby doing? How old is he? What can he do? Walk? Talk? o How about you? Any complications? Infections? Scars? Deal with the patient concern: - What is your understanding of cord prolapse? - It is a condition where the umbilical cord goes into the birth canal before the head, and then gets stuck and squeezed by the head. - This is a very serious condition. The cord delivers blood and nutrients to the baby. If blocked for long time, the baby will suffer from brain damage. - That is why they had to do urgent CS, which was a life saving procedure for the baby, and it had to be done immediately. - Do you know what the type of your CS was? Classical! History - I understand your point of view, but first let me ask you some questions about your health and your pregnancy! Last visit history / pre-eclampsia - When was your last f/u visit? - What was your BP? Was there any headache? - Was there leg swelling? Weight gain? Did they do urine test? - How about before being pregnant? Any hx of high blood pressure? Make sure the mother and baby are stable: ABCDE - Activity of the baby, is your baby kicking like before?

OSCE-guide-III.doc

Page 161 of 255

Physical Examination

-

Bleeding Contractions / pain Dripping / Discharge EDD (expected date of delivery)

U/S - Have you done your U/S? How many times? When was the last time? - Number of babies? - Location of the placenta? - Amount of fluids? Obstetric history: GTPAL - Other than the pregnancy that you had CS 3 years ago; any pregnancy before? Any abortions or miscarriages? - What were the circumstances? How many weeks? - How did you feel about it? How did you cope with that? Gynecological history PMH: Medications / allergy / blood transfusion Social history: SAD / support / home environment Counselling What is CS? The two types of CS - I would like to ask you; what is your understanding of CS? … - It is commonly used obstetrical intervention, used when there is a problem or contraindication for vaginal delivery and if there is an emergency situation that necessitates immediate delivery; and in these cases it is life saving; for both the mother and the baby! - There are two types of CS: o The transverse (done at the lower segment of uterus); it is the most common type; its advantages include: smaller scar and better healing. o The classical or vertical type; it is done less common; as we cut through the muscle fibers of the uterus it produces weaker scar; but it is indicated and actually needed in urgent case, like yours. As it allows quick access and fast delivery, because in some cases (like cord prolapse) we can not afford even few minutes more. Risks of vaginal delivery post CS - Due to the scar formed after the CS procedure; it is always recommended to deliver by CS, to avoid the tearing pressure of the uterine contractions during vaginal delivery. - If you decide to go for vaginal delivery, my concern is that the scar might undergo severe tearing pressure and might rupture, which will lead to massive bleeding. This is an obstetrical emergency that necessitates immediate intervention. Because you may end up losing your life and/or losing your baby. - I do not want to scare you, but the risks of having uterine rupture after classical CS is 12%, of which 10% of cases end up losing their lives. - For that reason: once classical CS, it is always CS. In case of counselling transverse CS: - Risks of having uterine rupture after transverse CS is 1%. - Even though, if you want to try vaginal delivery, we can not take the risk to try this at home, we can try this in the hospital, so that just in case any emergency might happen, we can intervene in the proper time. Management - I will ask someone to prepare a copy of your file - Speak with your midwife: - I am sure that your midwife is highly trained and qualified, and we share the same guidelines. I would recommend that you take your file and speak with your midwife, and I am sure she will explain the situation to you. - I will give you some brochures and web sites so that you can read more about that. - And if at any time you have any questions or concerns, you can come to see me.

OSCE-guide-III.doc

Page 162 of 255

Physical Examination

OCPs / Contraception Counselling XX is a 16 years old girl. She would like to get information about OCPs. For the next 10 minutes talk to her Introduction Concerns

─ ─ ─

History

Exclude pregnancy M G

─ ─ ─ ─ ─

O S

─ ─ ─ ─

Do you have any concerns? Why do you need / think about contraception at this point? Have you used contraception before? What is your previous experience? Why did you stop it? Any chance you are pregnant? How do you know for sure? Any nausea / vomiting? Breast engorgement? Frequency? Painful periods? / Irregular? / Heavy bleeding? Last PAP / any abnormal PAP Previous D&C GTPAL / IUD is not recommended in nullipara DETAILED If risky behaviour: OCPs will not protect against STIs Risky behaviour or previous STI/PID: NO IUD

Definition: birth control is an umbrella term for several techniques and methods used to prevent fertilization ─ Hormonal (OCPs / implants / injections) ─ IUD (contragestion: prevents the implant) ─ Barrier methods (condoms / diaphragms) ± spermicidal ─ Behavioural (fertility awareness/timing) / coitus interruptus ─ Post-coital contraception ─ Sterilization (male / female) OCPs IUD Mechanism of ─ OCPs are hormones: estrogen and ─ Mechanically prevents the implantation of action progesterone the fertilized ovum ─ OCPs prevent ovulation, increase thickness ─ Available forms: Copper / hormoneof cervical secretion coated ─ 21 tab + 7 sugar pills  28 days ─ Structural uterine anomalies Contraindications ─ HTN / CAD / Cerebro-vascular disease / DVT ─ History of ectopic pregnancy ─ Breast or gynecological cancer (past personal and family hx) ─ Undiagnosed vaginal Bleeding ─ Undiagnosed vaginal Bleeding ─ History of PID(s) ─ Active liver disease ─ Risky behaviour ─ Smoker > 35 yrs ─ Migraine Benefits ─ Regulate periods ─ Longevity ─ Independence to coitus or compliance ABCD: ─ Improves anemia (↓ bleeding) ─ ↓ benign breast lesions ─ ↓ ovarian cysts and cancer ─ ↓ risk of uterine cancer ─ ↓ dysmenorrhea

Available methods

OSCE-guide-III.doc

Page 163 of 255

Physical Examination

Side effects

Does not prevent STIs ─ Heavy periods ↑ risk of DVT / CAD ─ If hormonal coated: prog side effects: TRANSIENT: headache / wt gain / mastalgia ─ Breast tenderness ─ Weight gain ─ Headache ─ Nausea / vomiting ─ 0.1% – 5% ─ 0.5% – 2% OCPs IUD ─ Will do physical and pelvic examination, and document blood pressure, pap smear ─ Brochures and information ─ Follow up within 1 month ─ How we will start: ─ First day of next menstrual cycle ─ ─ Put package in obvious place that you see frequently ─ Take the pill in the same approximate time every day ─ Use another method of contraception for the first TWO months ─ If you miss a pill: see below ─ ─

Failure rate Plan

NOTES 

N.B.: if woman with risky behaviour: - Counsel for safe sex - OCPs do not protect against STIs



Failure rates are very high with behavioural methods: - Because it depends largely on the knowledge, experience of user and the usage technique; perfect-use versus typical-use failure rates.



What happens if I missed 1 or 2 pills (OCPs)? - 1 missed pill  take 2 pills the next day - 2 missed pills  take 2 pills a day, for the next 2 days - If missed more than 2 consecutive pills: use a backup method of contraception simultaneous to finishing up packet of pills (2 pills a day) until next menses



Woman comes to request sterilization (tubal ligation) 1- Discuss various other alternatives, make sure she knows everything about all of them (OCPs, IUDs, diaphragm, condoms, vasectomy) 2- Counsel the patient on the permanent nature of the procedure, the operative risks, and the chance of failure (1 in 200) Counsel about the risks of regret for the decision (young age, recent emotional trauma, family coercion) 3- Bring the patient back after 1 month for the bilateral tubal ligation procedure, preferably immediately after menstruation to decrease possibility of pregnancy

OSCE-guide-III.doc

Page 164 of 255

Physical Examination

OCPs

IUD

IM Injection

Advantages - Regulate periods ABCD: - Improves anemia (↓ bleeding) - ↓ benign breast lesions - ↓ ovarian cysts and cancer - ↓ risk of uterine cancer - ↓ dysmenorrhea - Longevity - Independence to coitus or compliance

- ↓ Dysmenorrhea - Effective - Independence

-

Contraindications Pregnancy Un-dx vag bleeding Cerebro-vascular dis / CAD Active liver disease Hormone-dep cancer Smoker > 35 yrs Hx of DVT / PE Migraine

-

Pregnancy Un-dx vag bleeding Structural uterine anomalies Hx of PID(s) Risky behaviour Hx of ectopic pregnancy

Condoms Diaphragm Behavioural

OSCE-guide-III.doc

Available Combined pills (E+P): low dose estrogen (20, 35, 50)

Failure rate 0.1% – 5%

TRANSIENT: - Breast tenderness - Weight gain - Headache - Nausea / vomiting - Heavy periods - Copper - If hormonal coated: prog - Hormone coated side effects: headache / wt gain / mastalgia

0.6% – 2%

- NO absolute contraindications

- Irregular bleeding - Amenorrhea after 1 year - Delayed post-use fertility

Every 3 months

< 1%

-

- Irregular bleeding - Headaches - Mood changes

Every 5 years

< 1%

Depoprovera SC implants - Longevity - Effective Nor-plant - Independence to coitus or compliance

Side effects - Does not prevent STIs - ↑ risk of DVT / CAD

Un-dx vag bleeding Acute liver disease Thrombophlebitis Thromboembolic disease

14% 20% - High failure rates up to 25% - To decrease the failure rate, can combine 2 methods

Page 165 of 255

OB-GYN

HRT counselling Introduction / overview History General

How do you describe your general health? Support DETAILED D&C / OCPs / PAP / mammogram / BMD GTPAL Dyspareunia  Irregular menstrual cycles  Vasomotor symptoms: sweating / hot flashes (hot flushes) / palpitations  Uro-genital symptoms: vaginal dryness / soreness / superficial dyspareunia / urinary frequency and urgency  Neurologic symptoms: mood changes / sleep disturbance / depression / anxiety Breast Cancer Uterine Cancer Ovarian Cancer  Early menarche  Obese  Late menopause  Diabetic  OCPs  Nullipara  No breast feeding  PCOS  Age  Past medical history of cancer or biopsy  Family history  Screen the first major risk factors for osteoporosis (see osteoporosis counselling case) What is your understanding about HRT?  Local preparations: creams / pessaries / rings  Systemic formulations: oral drugs / trans-dermal patches and gels / implants  

M G O S Menopausal symptoms

Risk factors for CANCER

Risk factors for Osteoporosis Counselling Common forms / products of therapy

Indications for HT

 

Contraindications to HT Pre-treatment evaluation Adverse effects and risks Any concerns?

OSCE-guide-III.doc

Estrogen alone Combined estrogen and progestogen Selective estrogen receptor modulator (SERM) Menopausal symptoms For SHORT term only, 1 – 2 years Osteoporosis

Hx / PE / baseline investigations Media spoke that HRT increases incidence of stroke, heart attacks and breast cancer, this was done by the (Women's Health Initiative), on the other hand, smoking, obesity, cholesterol increases the risk of these dis much more than HRT. In your case, you do not have the risk factors for cancer, and it will be beneficial for your hot flashes, vaginal dryness, and will protect you against osteoporosis … In medicine we always weight risk / benefits …

Page 166 of 255

OB-GYN

Introduction / overview: - The reproductive years of a woman’s life are regulated by production of the hormones estrogen and progesterone by the ovaries. Estrogen regulates a woman's monthly menstrual cycle and secondary sexual characteristics (e.g. breast development and function). In addition, it prepares the body for fertilization and reproduction. Progesterone concentrations rise in a cyclical fashion to prepare the uterus for possible pregnancy and to prepare the breasts for lactation. - Toward the end of her reproductive years when a woman reaches menopause, circulating levels of estrogen and progesterone decrease because of reduced synthesis in the ovary, which may lead to several symptoms, the severity of which can vary widely. - Hormone therapy (HT) involves the administration of synthetic estrogen and progestogen. HT is designed to replace a woman's depleting hormone levels and thus alleviate her symptoms of menopause. However, HT has been linked to various risks, and debate regarding its risk-benefit ratio continues Contra-indications of HRT: No absolute contraindications of hormone therapy have been established. However, HT is relatively contraindicated in certain clinical situations (similar to OCPs): - Breast and/or endometrial cancer - Undiagnosed vaginal bleeding - Acute liver disease - Thromboembolic disorders / DVT - Endometriosis / Fibroids - Diabetes, HTN, Heart disease Required baseline investigations - CBC - Urinalysis - Blood sugar levels - Fasting lipid profile - Electrocardiography - Pap test - Ultrasonography to measure endometrial thickness and ovarian volume - Mammography Possible adverse effects are as follows (similar to OCPs): - Nausea / bloating - Fluid retention - Weight gain (equivocal finding) - Mood swings (associated with use of relatively androgenic progestogens) - Breakthrough bleeding - Breast tenderness - HT may slightly increase the risk for breast cancer - There is association between HT and uterine hyperplasia and cancer - There is increased risk of thromboembolism with HT

OSCE-guide-III.doc

Page 167 of 255

OB-GYN

Needle Stick Counselling – HIV You are about to see Mrs … 33 years old female nurse, upset because she had just had needle stick after she gave an IV injection to a patient. Counsel her. Introduction Concerns History

Inform the patient about HIV

Plan

Workup

HIV infection / fatal disease / will impact her family - Can you tell me what happened? - Complete immunization record, including tetanus and hepatitis B - Previous occupational exposure to body fluids - Intravenous drug abuse - Sexual history - What is HIV? Major pathogens of concern! First of all; let me tell you the transmission rates: (no accurate studies) - Risk of blood transmission is: 0.3% for percutaneous exposure - Risk of female to male transmission is: 0.03% - Are you pregnant? Risk of intrauterine tx is: 3% with treatment and 30% without treatment - Advancement of HIV treatment Address pregnancy concerns: - Patient should receive ttt (not teratogenic) - HIV positive mothers should not breastfeed their babies - I will speak with the patient, explain the whole situation and ask him to consent for HIV status - If he agrees; we will know possibility of tx to you. If he is HIV negative, NO post-exposure prophylaxis is needed - If he refuses or if he is HIV +ve; we will have to assess what is called “exposure code” and match it with “HIV status code”; to simplify this, guidelines state we should assume you were exposed and give prophylaxis treatment: 4 weeks of 2 anti-virals (the basic regimen) - Hepatitis B vaccination 3 doses + immunoglobulins (immediately) - Blood tests for the patient if possible and for the exposed - I am going to speak with the patient now, and I will come back to you with his decision. - Any other questions or concerns?

What is HIV? - Human immunodeficiency virus (HIV) is a blood-borne, sexually transmissible virus. The virus is typically transmitted via sexual intercourse, shared intravenous drug paraphernalia, and mother-to-child transmission (MTCT), which can occur during the birth process or during breastfeeding. - The major pathogens of concern in occupational body fluid exposure are HIV, hepatitis A, hepatitis B, hepatitis C, and hepatitis D. These pathogens are viruses that require percutaneous or mucosal introduction for infectivity. The major target organs are the immune system (HIV) and the liver (hepatitis).

OSCE-guide-III.doc

Page 168 of 255

OB-GYN

Advancement of HIV treatment: - It used to be fatal - Currently, it is not cured, but controllable, may be in the future they can develop a cure to it - A lot of anti-virals were developed since 1990s till now, with ↑ efficacy and ↓ side effects, we started by giving many pills q4h, now it is once or twice a day - We follow the guidelines with monitoring of what is called “viral load” and “cell counts” of the patient immunity cells. To decide when to start treatment Workup: - Source patient (if available) o HIV o Hepatitis B antigen o Hepatitis C antibody - Victim/health care worker o HIV; testing now, at 1 month, and at 3-6 months o Hepatitis B surface antibody / titre (if vaccinated) o Hepatitis C antibody; testing now and after 4 and 8 weeks - Prior to initiating retrovirals: o Pregnancy test (stat) – if she is not pregnant o CBC count with differential and platelets o Serum creatinine/BUN levels o Urinalysis with microscopic analysis o AST/ALT levels / Alkaline phosphatase level o Total bilirubin level The ethical questions that might arise: - We can solve all this by calling the lab and adding HIV status check for the list of blood works of the patient, we had just send his blood to the lab! o You are right, this will save us the whole prophylaxis plan, however, we need to speak to the patient first; we can not do this HIV status test with the patient knowledge and consent. o What I am going to do –to help you– is that after we finish, I will go to speak with the patient, explain the whole situation to him, and ask his permission that we do this HIV statue test. - Even if the patient refused, we can ask Dr …, his surgeon, may be he knows! o May be Dr … knows the patient status or may not, but if the patient refuses to let us know, it is patient confidentiality, we can not ask Dr … about this information unless the patient consents we can ask the surgeon.

OSCE-guide-III.doc

Page 169 of 255

OB-GYN

Counselling – PAP smear CC

I understand you are here because you have some inquiries/worries about your last PAP test, is this right? How can I help you today? HPI M  Previous Pap test? How many? How frequent? Any abnormal Pap test? G Any previous colposcopy?  Contraceptive history GTPAL O  RISK factors for cervical dysplasia: S - Early age of sexual activity - Risky behaviour: unprotected sex / multiple partners - Smoking AS Same system - Any pain? With intercourse? - Discharge? Itching? Redness? - Any blisters / warts / ulcers? - Inguinal swellings? Nearby systems - Urine changes? Dysuria, frequency? - Bowel movements changes? GIT symptoms - Abdominal pain  OCD / PQRST / ↑↓ General - Constitutional symptoms PMH Any allergy / medication / disease FH Gynecological tumours SH If teenager: HEAD SSS COUNSELLING: - What do you know about (LGSIL)? What would you like to know? - Have you had any experience with … in the past? - Have you [read / talked to someone / searched the internet] about this issue? Worried about PAP results - PAP smear or test is done to screen for any changes that might happen in the cervix, before it turns to serious disease (to early detect pre-malignant lesions). - At the cervix there is transitional zone between two types of cells, it undergoes rapid growth, if there is irritation due to HPV, it might turn malignant. It takes years from the moment it begins to grow abnormally to the moment it becomes malignant, that is why we do frequent PAP tests, to detect it before it turns into malignant tumour. - The results come back from PAP test either ASCUS (Atypical squamous cells of Undetermined Significance) / LG-SIL (low grade squamous intra-epithelial lesion) / or HG-SIL (high grade squamous intra-epithelial lesion)

OSCE-guide-III.doc

Page 170 of 255

OB-GYN

-

-

-

-

For ASCUS: o Woman ≥ 30 yrs  HPV DNA testing  If negative  repeat cytology after 1 year  If positive  colposcopy o Woman < 30 yrs  repeat cytology in 6 months  If negative  repeat after 6 months  still negative  routine screening  If ≥ ASCUS  colposcopy For LG-SIL: o Colposcopy o Or repeat cytology after 6 months  If negative  repeat after 6 months  still negative  routine screening  If ≥ ASCUS  colposcopy For HG-SIL: o We send you for colposcopy For colposcopy, we will refer you to the gynaecologist who will perform special procedure, during which, the gynaecologist will take a biopsy, and send it for further investigations; o If the biopsy is negative, we will repeat the PAP after 6 months o If the biopsy is positive, we will do more investigations to establish a diagnosis and may need to do another larger biopsy called cone biopsy

Treatment options: - Laser - Cone biopsy - LEEP (loop electrosurgical excision procedure) Colposcopy - Colposcopy is a magnification of the cervix (10-12 times), the procedure may cause some discomfort but is not painful. - The gynaecologist will insert a speculum (the same instrument used for Pap test), and then she/he will use a special magnification device (the colposcopy) to visualize the cervix. - The gynaecologist will apply acetic acid (vinegar) that helps make the vascular patterns more visible, application of this acetic acid may give an itchy sensation. - Then if the gynaecologist suspects a lesion, she/he will need to take a biopsy, you will feel a punching sensation, and you might experience a little discomfort and spotting for few days. - You need not to have anything inserted into your vagina for 24 hours before and 2 days after the procedure (no vaginal intercourse, no douching), and you might need to take some OTC medications (Advil) for few days after the procedure.

OSCE-guide-III.doc

Page 171 of 255

OB-GYN

Antenatal Counselling o

o

Ms XX has missed her period for 2 wks; she did a home preg test which was positive. This is her first experience. In the next 10 min, please talk to her and give her necessary advices about her pregnancy. A 38 yrs old pregnant lady came to you because she is concerned about problems during advanced-age pregnancy, counsel

Pregnancy

   

M G O S PMH

      

FH



Social Hx

    

Concerns

Was this planned pregnancy? What is your feeling about that? And your partner feeling? congratulations ☺ How do you describe your general health? Symptoms suggesting pregnancy: any nausea / vomiting? Breast engorgement / heaviness? Frequency? LMP, calculate due date Pap smear?



OSCE-guide-III.doc

HTN, Diabetes, Kidney disease, Heart disease, Epilepsy, exposure to chickenpox as a child, Rubella vaccine, HBV, HIV, Blood Group & Rh group Allergy Medications FH of complicated pregnancies, abortions, MR, genetic & congenital abnormalities Home, violence, support Environment: work Activity: exercise / any pet Diet SAD Address any problem you find while you’re taking history

Page 172 of 255

OB-GYN

Counselling: 







  



     

   

To ensure healthy outcome of the pregnancy I need to see you on scheduled visits, every 4 weeks till the 28th week, then every 2 weeks till the 36th week, and then every week thereafter and till delivery Today we’ll do physical examination including pelvic exam, Pap smear if more than 6 months, blood work including CBC, Lytes, INR/PTT, Urea, Creatinine, Blood Type, VDRL, Rubella antibody, Hepatitis, ± HIV, Urine dip and microscopy, ± ECG. Anatomy US at 20 weeks. Glucose challenge test at 24 weeks Risks of Down syndrome are: 1/400 at 30 yrs of age, 1/200 at 35 yrs of age, and 1/100 above 40 yrs of age  we try to anticipate it by US and integrated prenatal screening then confirm it by amniocentesis U/S for nuchal translucency: at 12 weeks IPS I: 11-14 wks /+/ IPS II: 15-18 wks (Maternal serum alpha-fetoprotein, β-hCG, uE3– Unconjugated estrogen) Amniocentesis (U/S-guided trans-abdominal extraction of amniotic fluid / for identification of genetic anomalies): at 15-16 wks, 0.5% risk of spontaneous abortion and risk of fetal limb injury You need a well balanced diet; Canada's Food Guide to Healthy Eating suggests 3-4 servings of milk products daily (greater if multiple gestation), a daily caloric increase of -100 cal/d in the 1st trimester, -300 cal/d in the second and third trimesters and -450 cal/d during lactation. If you do not consume an adequate diet, you can take daily multi-vitamins (avoid excess vitamin A) Important nutrients during pregnancy: folate; 0.4-5 mg per day / calcium; 1200-1500 mg per day / iron: 1 mg/d in T1, 4 mg/d in T2 and > 6 mg/d in T3 Pregnant ladies tend to have constipation, you can take Lactulose for this, avoid raw or processed meat Haemorrhoids, back pain, heartburn and increased vaginal discharge are common Will gain weight; 5-10 pounds in 1st half, 1 pound /week in 2nd half, total of 25-35 pounds in average Exercise is OK… walking, swimming, avoid strenuous activities Stay away from cats’ litter No medication without asking your doctor, no x-rays Smoking increase the risk of abortion, LBW, premature delivery No safe level of alcohol during pregnancy, better to avoid it totally Offer brochures, connect to support groups and classes for pregnant women

OSCE-guide-III.doc

Page 173 of 255

OB-GYN

Endometriosis You are covering for your colleague Dr. Smith. You are about to meet Mrs. XX to discuss the result of her laparoscopy & inform her that she has endometriosis. For the next 10 minutes, please talk to her& address all her concerns. -

Introduction: I would like to discuss the result of your laparoscopy but I need to get some information. o Why did you have laparoscopy done and what was your doctor’s concern? o You have endometriosis: ─ This means implantation of the interior lining of the uterus somewhere in other places outside the uterus including the ovaries, the supporting structures of the uterus or on the intestine (draw a picture of the uterus and ovaries for the patient). ─ During periods, this outside tissue also begins to bleed. This explains the painful periods. ─ This may cause infertility in some people. ─ Sometimes it runs in the family o I would like to ask about some symptoms (to fit everything together) ─ Pain: Analyze (OCD / PQRST). Relation to the period. Dysmenorrhea, dysuria, dyschezia, dyspareunia. Do you need pain killer? ─ Infertility: I understand your frustration. For how long have you been trying to conceive? Are you currently sexually active? How frequent? ─ Irregular vaginal bleeding  analyze ─ Frequency ─ Blood with stool, diarrhea

-

Menstrual history – brief Gynecological history Obstetric history – GTPAL Sexual history – brief

-

Past medical history: HTN, Diabetes, kidney disease, blood group & Rh. Allergies / medications / hospitalization / surgeries / blood transfusion Family history of abortion Social history: smoking, alcohol, drugs / work / home environments / support

-

16

Conclusion: endometriosis Plan: o Will do physical and obstetric examination o Give her treatment options ─ Medical:  NSAIDs – e.g. naproxen sodium (250 mg PO bid) 16  Pseudo-pregnancy: OCPs trial for 6-12 months (Ovral 1 tab PO od)  Pseudo-menopause (only short-term 7.5 mg/day for > 1-3 months) 5- Primary hyper-parathyroidism 678910-

Weight < 57 Kg Current smoker Premature ovarian failure (female on Tamoxifen for breast cancer / surgical menopause) Male on androgen-deprivation therapy for prostate cancer Heparin or anti-epileptic use or biologics (anti-cancer treatment)

Investigations: - BMD: Age group When to do BMD -

< 50 years If > 2 of the first (5) risk factors

50 – 65 years If > 2 of any from the list of risk factors

> 65 years Always do BMD, screen even there is no C/O

Blood work: o Serum calcium and phosphate levels o Alkaline phosphatase o Creatinine o SPEP (serum protein electrophoresis) o PTH (para-thyroid hormone) o Give vitamin D for 2-3 months, then assess the level, if > 0.75 nanogram  it is normal and do not repeat it again

Treatment: - Based on BMD, risk factors, age of patient  Fracture Risk Stratification  low, moderate, or high LOW MODERATE HIGH - Life style If fragility fracture (in thoraco-lumbar x-ray) OR prolonged - Life style modifications use of corticosteroids modifications - F/U DEXA Medical NO YES after 5 yrs treatment - Life style modifications - Life style modifications - F/U DEXA after 2 yrs - Medical treatment Life style modifications - Weight-bearing exercises (walking, jogging) - Ca 1200 mg/d (including the dietary intake, avoid ↑↑↑ Ca) - Vit D 1000 IU/d if < 50 yrs, and 2000 IU/d if > 50 yrs - Smoking cessation - ↓ alcohol and caffeine

OSCE-guide-III.doc

Medical treatment - Bisphosphonates - SERM (Raloxifene): agonistic effect on bone but antagonistic effect on breast and uterus - Parathyroid hormone (PTH) - Calcitonin (if back fragility fracture + pain) - HRT

Page 176 of 255

OB-GYN

Pediatrics

OSCE-guide-III.doc

Page 177 of 255

OB-GYN

History taking – Pediatrics History taking – Pediatrics  

Introduction: Chief complaint

1234567-

Analyze the CC Impact Rule out infection Differential diagnosis BINDE Past medical history Family history

1- Analyze the CC

- BINDE screening - Mother attitude! -

-

2- Impact

3- Red flags Rule out infection

-

4- Differential diagnosis 5- BINDE 6- Past medical history 7- Family history 0-6 years BINDE Pregnancy / Birth Immunization Nutrition Developmental Environmental

OSCE-guide-III.doc

In any pediatrics case: always CONSIDER child ABUSE / NEGLECT  report to CAS

TIME: Os Cf D: When did it start? How did it start? Sudden or gradual? - At that time, did your baby have any fever, flu-like symptoms? - Is it continuous or on and off? How often? Day and night? Character: - PQRST - If vomiting or diarrhea: COCA + BLOOD - Timing: is it related to feeding / meals? ↑↓ Factors: is it related to position? Meals? Is he drowsy? Floppy? Does he cry? Is it high pitched cry? Did you notice his suckling is weaker than before? Constitutional symptoms! R/O infection: Did you notice if your child has fever or skin rash? Cough / wheezes? Ear pulling or discharge? Runny nose? Foul smelling urine? Abdominal distension? Diarrhea? Rule out child neglect - BINDE screening - Mother attitude! Scan for risk factors for child abuse / neglect Hospitalizations / surgeries / blood transfusion Illnesses (cancer) / infections Medications / allergies. Travel Family history of similar disease in the family 6-14 years School performance

14-18 years HEAD SSS Home Education Activity Diet Suicide Sexual activity SAD (smoking / alcohol / drugs)

Page 178 of 255

OB-GYN

BINDE Pregnancy: - Was your pregnancy planned? If no  social issues - Were you having regular follow-up visits? How about U/S? Was it normal? - During your pregnancy, did you have any illness? How about any fever or skin rash? Have you ever been in contact with sick kids? Kids with skin rash or fever? Have you ever been in contacts with pets? - Did you take medications? Even OTC? Did you smoke? Drink alcohol? Have you ever tried recreational drugs? What about before pregnancy? - Were you screened for Hepatitis B virus? HIV? other diseases? There is screening test that we do a vaginal swab at 36 weeks called GBS, did u have it? - What is your blood group? What is your baby blood group? Birth: - Was your baby full term or not? - Was it a vaginal delivery or c-section? o If c-section  why? Was there any complication? Abnormality? o If vaginal  was it difficult labour? Prolonged labour? How many hours? Was there any early gush of water? How many hours? Did you need any help to make it easier; e.g. vacuum? - Did your baby cry immediately or not? Do you know what his Apgar score was? Did he need special attention? When did you leave the hospital? - Were there any bruises or swellings on your baby’s body? - Were you told that your baby had any special features? - After delivery, did you have any fever / discharge? Did you take any medications? Immunization: - Are your baby’s shots up-to-date? o Yes  when was the last shot? o No  any reason for that?  Our religion prohibits vaccination: ok, that is fine  We think vaccines cause autism: correct this info, vaccines are safe  We were busy  neglect concern  what is baby weight? Nutrition: - Weight: o o o o

Weight calculation: What is your baby’s weight today? What was his weight at birth? What was his highest weight? Do you have access to growth charts?  If below 3rd percentile: underweight  If (at any time) he crossed (down) two major lines: failure to thrive

At birth : x 5 months 1 year 2 years

Kg :2x :3x :4x

Kg Kg Kg

More than 2 years old: [(age X 2) + 8] Kg

Even low birth weight, catch up weight later, i.e. @ 1 year they must be around 10 Kg, not only 3 x

OSCE-guide-III.doc

Page 179 of 255

OB-GYN

-

Height: o

o -

To calculate height:  At birth X cm 50 cm  1 year 1 ½ X cm + 25 75 cm  2 years 1 ¾ X cm + 12.5 87.5 cm  3 years 1 7/8 X cm + 6.5 94 cm  4 years 2 X cm 100 cm For each year: the baby gains (½) of the previous year increase, so the baby gains ½ X by the first year, ¼ X by the second year, 1/8 X by the third year.

Diet: o

o

What do you feed your baby?  Everything we eat. No restrictions!  that is fine  Breast feed  if more than 4 months: • Any iron supplement? • Any Vit D supplement?  Formula: • Since when?  if since birth: any reason that you chose formula over breast feeding? • Which formula? Any recent change in formula? For any case of chronic diarrhea?  Do you give him solid food; biscuits / bread / cereal?  Which started first? The diarrhea or this new food?  How many diapers do you change per day? (normally 5 – 6)

Developmental: Now I would like to ask you some questions about the kind of activities that your child can do, and other questions to assess his development. Gross motor Fine motor Sit alone / roll over 6 months Draw line 15 months Crawling 9 months Draw cross 2 years Standing / cruising 1 year Draw circle 3 years Walking 15 months Draw square 4 years Go upstairs holding 18 months Draw triangle 5 years Go downstairs 2 feet 2 years Tricycle 3 years Social Social smile Stranger anxiety Separation anxiety Says “NO”

6 weeks 6 months 9 months 2 years

Speech / verbal Mama / papa 2 words beyond Ma, Pa 2-3 words phrases Short sentences Speaks fluently

9 months 1 year 2 years 3 years 5 years

N.B. (autism / Down syndrome / child abuse): there is no stranger or separation anxiety.

OSCE-guide-III.doc

Page 180 of 255

OB-GYN

Environment: - How do you feel being a new mom? How do you feel about your baby? o How is your mood? You look down for me, any chance you are being depressed? Did you have depression before? - With whom do you live? How is the relation between you? o How is the relation between you and the baby? o How is the relation between your partner and the baby? - How do you support yourself financially? - Do you live in home (basement: mold) or apartment? Is it an old building (lead)? - Any other children in the house? - Do you or any body in the home smoke? Drink? Use recreational drugs? - Is anyone of your family seeing a psychiatrist? Has mental illness? - In ABUSE cases: tell me more about your childhood … HEAD SSS Home: - With whom do you live? - How is the relation between you? Are they supportive? - Any siblings? Education: - Do you go to school? Do you like going to school? - Which grade? Which subjects do you study? - How about your marks, what marks do you get? What about in the past? Activity: - What kind of hobby do you have? - Have you travelled recently? - In EPILEPSY case: do you operate machines / drive / go hiking? Diet: - How about your diet? What do you eat? Do you follow special diet? - What is your weight? What was your weight before? Suicide: - How is your mood? - Any chance that you might hurt yourself? Sexual activity: - Are you dating? Are you in relationship? - Are you sexually active? When did you start? When was the last time? - How many partners do you have? Do you practice safe sex? Smoking / Alcohol / Drugs: - Now, I would like to ask you some personal questions, it is important to ask it, and it is confidential, do you Smoke? Drink Alcohol? Have you ever tried recreational drugs? - Sometimes people at your age might start to smoke, drink, or use recreational drugs. Do you know any of your friends doing this? How about you? Have you tried that? - For IV drugs: When was the last time? Did you share needles?

OSCE-guide-III.doc

Page 181 of 255

OB-GYN

Jaundice A new born 5 days old, with jaundice since day 2 Introduction Differential diagnosis of newborn jaundice - Physiologic (usually days 2-7)  CC unconjugated - Analyze the jaundice (OCD) - Breast milk jaundice - Impact / consequences - Breast feeding jaundice - Red flags / rule out infection Pathologic (anytime) - DD - Hemolysis (unconjugated) - BINDE - Infection  sepsis (conjugated or - Birth  pathological unconjugated) - Nutrition  physiological FH Introduction: Good morning Mrs …, I am Dr …, I am the physician in charge today, I understand that you are here because your son has jaundice (or is yellow). In the next few minutes I will be asking you some questions to help me figure out the condition, before I proceed, I would like to know the name of your child? … This is a nice name. 1- Analyze the CC: - When did it start? Early in the 2nd day (or before: pathological) or late (pathological or physiological)? - Who noticed it? You or someone else? When? Where did you notice it? How about his eyes? How about his feet? Is it spreading? Is it ↑ or ↓? - How about his urine, is it darker? And stools, is it pale? 2- Impact / Consequences: - Is he drowsy? Floppy? - Does he cry? Is it high pitched cry? - Did you notice his suckling is weaker than before? 3- Red flags / Rule out infection: - Did you notice if your child has fever or skin rash? Cough / wheezes? Discharge from his ears? Runny nose? Foul smelling urine? Abdominal distension? - Any night sweats / chills? Any lumps or bumps in his body? Tender points? 4- Differential diagnosis: Physiological Pathological How do you feed him? Breast milk? Formula? ─ Infection should be ruled out or ─ Breast feeding jaundice: (or “lack of confirmed by now breastfeeding” jaundice): Not enough milk ─ Hemolysis:  dehydration  What is your blood group? Your ─ Breast milk jaundice: is more of a baby blood group? Father blood gp? biochemical problem (inhibition of  Rh incompatibility  IUGR bilirubin conjugation leads to increased  Were you screened for infections levels of bilirubin in the blood). during pregnancy? Treatment: substitute with formula ─ Biliary atresia ─ Hepatitis: neonatal

OSCE-guide-III.doc

Page 182 of 255

OB-GYN

5- BINDE Birth: Nutritional history: - How do you feed him? Breast milk? Formula? - Breast: o How many times do you feed him? o Do you use 1 breast or both of them? How long each? o After feeding him, do you feel your breast engorged? - Formula: o Any reason to choose formula feeding? o Which type of formula? Do you know how to prepare it? Environment: - Any other children? Did any of them develop jaundice after birth before? 6- PMH?! 7- FH: - Jaundice - Liver disease - Blood disease - Disease called cystic fibrosis Diagnostic workup:

When to suspect pathological jaundice? Treatment

-

Hemolytic workup: CBC / blood gp (mother and baby) / peripheral blood smear / Coomb’s test / bilirubin (direct and indirect) Septic workup: CBC / differential / blood & urine cultures / TORCH screen TSH and G6PD screening Liver enzymes / bilirubin / and coagulation profile If in the first day (or early second day) of life Bilirubin rises > 85 µmol/L/day Bilirubin level > 220 µmol/L before 4 days of age Conjugated (direct) bilirubin > 35 µmol/L Persistent jaundice lasting beyond 1-2 weeks of age Ensure proper hydration and feeding If sepsis: treat the underlying infection Phototherapy: if total bilirubin is > 300 µmol/L, and only for unconjugated hyperbilirubinemia, it is contraindicated in direct hyperbilirubinemia Exchange transfusion: if total bilirubin is > 400 µmol/L

The mother has a concern: will he develop mental retardation? 1. This is a reasonable concern, I am glad you came here today to figure out 2. What made you think about that? 3. In order to be able to answer this, I first need to ask you some questions, examine him and we may need to do some blood tests. However I would like to inform you that jaundice in newborns is a common condition, and it is usually physiologic jaundice that does not lead to mental retardation Complications of Rh incompatibility: (1) kernicterus (brain damage  seizures), (2) Hydrops fetalis (generalized edema), (3) Hypoglycemia

OSCE-guide-III.doc

Page 183 of 255

OB-GYN

IUGR A newborn 3 hours old with IUGR, counsel the mother Introduction

News BINDE Obstetrical history

Mother PMH

Good morning … I understand that you just gave birth, my colleagues are taking care of your baby. And I would like to ask you some questions regarding your child health, but first tell me; - How do feel right now? - Have you seen the baby? - Did you pick a name? Your baby has just been diagnosed with a condition called “intra-uterine growth retardation” or “low birth weight” … For that reason; I would like to ask some questions about your pregnancy! -

GTPAL Were you pregnant before? How many times? Any abortions? Miscarriages? Any history of chemo therapy or exposure to radiation Any family history with congenital anomalies

+ The mother has a concern: the baby is green  This means he passed meconium (baby stool) which means the baby had a stressful delivery + The mother has a concern: is it my mistake?  This condition is related to multiple factors, some factors are related to pregnancy, and others are related to the baby or the environment. On the other hand, there are NO safe limits for smoking / drinking alcohol during pregnancy, so, in the future, if you decide to become pregnant, it is important NOT to smoke / drink alcohol / use recreational drugs during your pregnancy Possible causes: ─ Smoking / alcohol / cocaine during pregnancy, (cocaine during pregnancy  microcephaly, IUGR, MR) ─ TORCH infection, ─ Extreme of age, esp. advanced age pregnancy Risks for the next 48 hours: asphyxia / hypoglycemia

OSCE-guide-III.doc

Page 184 of 255

OB-GYN

Crying Baby Introduction CC Analysis of CC

OCD / all the time / day and night? Is he crying > 3 hrs/day for > 3 days/week for > 3 weeks ─ What initiates or increases the crying? o Any chance the baby is hungry? What do you feed him? o Any chance that he is too hot / too cold? Do you adjust the temperature? o Any chance that he is wet? How often do you change his diapers daily? Is there any skin or diapers rash? ─ What improves or decreases the crying? When he cries, what do you do? o Did you try to hug / hold / burp / sooth / play music / give him a walk? o Did you try to rock him? Shake him? What happened to him? ─ When he cries, does he pull his legs? Is he passing gases? Is his abdomen distended? Is it related to feeding? How are you coping with this? ─ How does this affect your life? And your partner life? Are you able to go to work? ─ Is he drowsy? Floppy? ─ Did you notice if your child has fever or skin rash? Cough / wheezes? Discharge from his ears? Runny nose? Foul smelling urine? Abdominal distension? Diarrhea? ─ Any infection (there will be other symptoms)  review of systems ─ Infantile colics (crying > 3 hrs/day for > 3 days/week for > 3 weeks), between the age of 3 weeks and 3 months, without another explanation  reassure ─ Child neglect ─ Feeding problems: overfeeding / hungry Scan for risk factors for child abuse Nutritional ─ How do you feed him? Breast milk? Formula? ─ What about his weight? Environmental ─ With whom do you live? How is the relation? ─ How do you support yourself financially? Do you get enough support? ─ Any other kids? Any repeated visits to the ER? ─ Mental problem ─ Parent SAD ─ ─

Impact

Red flags / R/O infection DD

BINDE

FH

Investigations (not including those for suspicious child abuse): CBC / urinalysis / stool analysis

OSCE-guide-III.doc

Page 185 of 255

OB-GYN

Chronic Cough – Asthma Child complains of cough for 6 weeks, post-pneumonia, father wants to renew antibiotics Here, we have two issues:  The productive cough & fever 6 weeks ago  pneumonia  The intermittent / dry cough that presented after CC HPI

Analysis of the CC Cough

Impact Red flags

Cough  Os Cf D /+/ COCA + B + Phlegm  Certain time of the day? Night? Acute phase Chronic phase Continuous / productive / Intermittent / dry cough / on and off / fever / loss of appetite no fever  Seen by a doctor? What  SOB, noisy breathing, wheezes, diagnosis? Treatment? chest tightness, nausea / vomiting  Anti-biotic history!  Does he cough to the extent of vomiting or LOC  Did you renew it? From the same doctor? Was  Pertussis vaccination? he examined? Any xrays were done? How did this affect his life? Daily activity? Constitutional symptoms Triggers of Asthma: any thing that ↑ this cough?  Chronic diarrhea  cystic fibrosis  Any allergy Brief Other allergic diseases: atopic dermatitis / allergic rhinitis Allergic diseases: asthma / skin allergies  

Differential diagnosis BINDE PMH FH Triggers

Infection  Medications  

Outdoor

Indoor

Stress

OSCE-guide-III.doc

          

Recent chest infection? Flu-like symptoms? Fever / chills? How do you use puffers? Stored properly? Not expired? Did you start new medication? β-blockers? Aspirin? Any recent ↑ in dose of these medications? Exercise Cold air Pollens (is it seasonal?) Dust: construction / smug (smoke/ fog/ exhaust) Do you smoke? Anybody around you? Do you have pets? People around you? Fabrics related: carpets floor? Any change in linen? Pillows? Blankets? Mattress? Curtains? Relation to any type of food? + Perfumes Do you live in a house (basement  mold)? Any construction renovation? Exposure to chemicals? Any new stressful situations?

Page 186 of 255

OB-GYN

Questions: Diagnosis: Investigations: Treatment:

hyper-reactive airways disease x-ray steroids puffer for 4 weeks

Counselling: ─ The most likely explanation for that is a condition called: hyper-reactive airways disease. It is a term used to describe asthma-like symptoms in infants (< 6 years old) that may later be confirmed to be asthma when they become old enough to participate in asthma tests (spirometry and bronchodilators). ─ This is a common problem, and is usually triggered by infection (acute bronchitis or pneumonia), it may last up to 10 weeks after infection. ─ It may be self limited; however, we need to start treatment with puffer (steroids puffer for 4 weeks). ─ When the child becomes older than 6 years, and if the condition is still persistent for more than 10 weeks, we send the child for investigations (spirometry and bronchodilators) to confirm the diagnosis of bronchial asthma. ─

If this condition happens in adults, we treat with puffer for 4 weeks, if no improvement; we send to investigate for asthma (spirometry and bronchodilators then metacholine challenge test).

Case: 9 weeks history of cough on Amoxil for 2 weeks ─ DD: asthma / bronchitis / cystic fibrosis / recurrent pneumonia ─ Investigations: CXR / CBC, differential, lytes / sweat chloride test

OSCE-guide-III.doc

Page 187 of 255

OB-GYN

Anemia 6-9 months, mother complains he is pale? 1- Analyze the CC - Clarify CC: What do you mean he is pale? Is he yellow? - Os Cf D - Who noticed it? You or someone else? Is there any chance that he had this pallor before and you were not aware of it? 2- Impact - Is he drowsy? Floppy? - Does he cry? Is it high pitched cry? - Did you notice his suckling is weaker than before? Signs of - Is he active / playful like before? What can he do? Is he crawling? anemia - If he is doing activity, did you notice any SOB? Fainting? - Is he gaining weight? 3- Red flags: rule out - Constitutional symptoms! infection - Did you notice if your child has fever or skin rash? Cough / wheezes? Ear pulling or discharge? Runny nose? Foul smelling urine? Abdominal distension? Diarrhea? 4- Diff diagnosis: - Rule out child neglect - Bleeding disorders: nose / gums / coughing / vomiting / bruises  Iron def. anemia on body / blood in urine / stools / joint swelling  Thalassemia Leukemia: Constitutional symptoms / Bone pain [if he walks,  Hemolytic disorders does he limp? if you carry him, does he complain of tender  Bleeding disorders points in his body] / cough / repeated infection  Chronic diseases Lead intoxication Leukemia 5- BINDE  

-

6- Past medical history

-

7- Family history

-

-

Scan for risk factors for child abuse / neglect N: What are you feeding him? Breast milk? From the beginning? Do you give him any iron supplements or iron fortified cereals? B: was he term or not? E: with whom do you live? How do you support yourself financially?  offer social support Where do you live, if old place, have you ever seen him eating the paint scales? Any heart / lung / kidney / liver disease? Hospitalizations / surgeries / illnesses (cancer) / infections Medications (Sulpha drugs – G6PD deficiency) / allergies Travel Family history of similar disease in the family Any bleeding disorder Any repeated surgeries? (cholecystectomy / splenectomy) Ethnicity: some blood diseases are more common in certain parts of the world, that is why I need to ask you about your background, what about your partner? Are you related by blood to your partner?

Investigations: lab works; CBC / differential / lytes / serum iron studies (ferritin, TIBC) / hemoglobin electrophoresis / KFTs / INR / PTT Treatment: iron supplement

OSCE-guide-III.doc

Page 188 of 255

OB-GYN

Vomiting The mother of (6 weeks – 3 months) old baby came to the clinic complaining of child’s repeated vomiting. Introduction Chief complaint 1- Analyze the CC

2- Impact

-

3- Red flags -

4- DD

When did it start? How did it start? Sudden or gradual? At that time, did your baby have any fever, flu-like symptoms? Is it continuous or on and off? How often? Day and night? COCA + BLOOD: what do you feed him? Is he vomiting the entire amount? Is it watery or curdy? Character: is the vomiting forcible? Projectile? Timing: is it related to feeding / meals? After vomiting, do you fell he is still hungry? General condition after vomiting ↑↓ Factors: is it related to position? When lying down? Other GIT symptoms: diarrhea Is he drowsy? Floppy? Does he cry? Is it high pitched cry? Did you notice his suckling is weaker than before? Dehydration: do you feel his lips dry / skin dry? Does he tear? How many diapers Failure to thrive: what about his weight, do you know his weight? What was his weight at birth? Do you have access to his growth charts? Constitutional symptoms! R/O infection: Did you notice if your child has fever or skin rash? Cough / wheezes? Ear pulling or discharge? Runny nose? Foul smelling urine? Abdominal distension? Diarrhea?

Differential diagnosis of 6 weeks vomiting Rule out child neglect - Pyloric stenosis -

-

Any infection / meningitis Brain tumour

-

-

GERD Wrong formula OR not preparing it well Overfeeding OR NOT burping

-

-

OSCE-guide-III.doc

BINDE screening Mother attitude! Family history of pyloric stenosis Appear at age of 2 – 4 weeks Projectile / non-bilious / baby still hungry after feeds Continuous Other symptoms / neurological: weakness / neck stiffness / seizures After feeds Not all feeds No ↓ in weight

Page 189 of 255

OB-GYN

5- BINDE

-

Scan for risk factors for child abuse / neglect Nutritional history: - How do you feed him? Breast milk? Formula? Which type? Do you know how to prepare it properly? Any solid food? - Which happened first, the vomiting or switching to the new formula? - Weight analysis! Environment: - How do you feel being a new mom? How do you feel about your baby? - How is your mood? You look down for me, any chance you are being depressed? Have you ever been depressed before? - With whom do you live? How is the relation? - How do you support yourself financially? - Do you get enough support?

6- PMH 7- FH Pyloric stenosis DD: duodenal atresia / tracheo-esophageal fistula Management plan: - Investigations: lab works (CBC, lytes, ABG) / US - If dehydrated: admission - If suspicious child neglect: contact CAS Potential risk factors for child abuse: - SAD parents (smoking / alcohol / drug use) - Pregnancy not planned - Preterm baby - Congenital anomalies - Baby who needed special attention after delivery - Separation from the child - Difficult child - Young couple - Parents with history of abuse - Stress or financial difficulties in the family Investigations for child neglect - Full blood work / CBC / albumin level - Fundoscopy - Skeletal survey

OSCE-guide-III.doc

CONTACT CAS

Page 190 of 255

OB-GYN

Diarrhea Diarrhea Failure to thrive – FTT  What about his/her appetite?  What other associated symptoms? (Respiratory / Gluten) Cystic fibrosis Celiac disease Milk protein HIV allergy From cow milk - Good appetite - Poor appetite - Respiratory - Gluten Should not be given < 1 year

-

NO FTT Toddler’s diarrhea Infections Lactase Deficiency (lactose intolerance)

A 50 years old father comes with 9 months child with 6 weeks of diarrhea (CHRONIC) 1- Analyze the - Os Cf D CC - COCA + BLOOD + others: - Watery / loose / bulky - Any undigested food - Difficult to wipe? - ↑↓ Factors: Juice (Excess fruit juice) - Identify FTT – weight: What is weight today? At birth? Last visit? The highest weight? Not gaining weight? - Other GIT symptoms: vomiting - APPETITE 2- Impact - Is he drowsy? Floppy? - Does he cry? Is it high pitched cry? - Did you notice his suckling is weaker than before? - Dehydration: do you feel his lips / skin dry? Does he tear? How many diapers - Failure to thrive: what about his weight, do you know his weight? What was his weight at birth? Do you have access to his growth charts? - Long period  malabsorption  anemia and rickets 3- Red flags: - Constitutional symptoms! (R/O infection) - Did you notice if your child has fever or skin rash? Cough / wheezes? Ear pulling or discharge? Runny nose? Foul smelling urine? Abdominal distension? Diarrhea? 4- Differential DD for ACUTE diarrhea: diagnosis - Use of antibiotics - Infectious: - Camping / travelling - Any body else at home with diarrhea? - Does he go to day care? DD for CHRONIC diarrhea without failure to thrive: - Toddler’s diarrhea: does he drink too much juice daily? - Infectious – parasitic / traveller’s diarrhea - Lactose intolerance: - Does he pass a lot gas? - Does he have any redness / skin rash at his buttocks?

OSCE-guide-III.doc

Page 191 of 255

OB-GYN

5- BINDE

DD for CHRONIC diarrhea WITH failure to thrive: - HIV (if susceptible): was he screened for HIV - Milk protein allergy: cow milk given before 1 year - Cystic fibrosis - Celiac disease Cystic fibrosis Celiac disease - Good appetite - Poor appetite - Repeated chest infections? Cough? - bulky foul-smelling stools - When did he poo the first time? - What do you feed your son? How - Did he have yellow discoloration many times? after birth? For how long? - Do you know how to prepare the - Does he have any bulging through formula? Was it changed his buttocks? recently? - Did you start to give him solid food? What type of food? How about bread, cereals, biscuits? Which started first, the diarrhea or the new food? - After you feed him, does he have abdominal distension? Gases? Pulling his legs? - Does he have flat buttocks? Thin legs and arms? Distended abdomen? - Scan for risk factors for child abuse / neglect - Nutritional history - Weight analysis

6- PMH 7- FH

Questions: - What is your differential diagnosis: o Cystic fibrosis o Celiac disease - If the biological mother called, want to know about her son, do you tell her or no? o In order to determine whether I should release any information or no, I would like first to know who has the legal custody (guardian) of this child. It might be the adopting father, a social worker (case manager) … Notes: - If the child was adopted, and you are speaking with one of the new parents: o Are you the biological mother/father? o Is this adoption or foster home? o When was the child adopted? At which age? From where? o What were the circumstances? o Do you have information about the biological parents? o Was he screened for HIV?

OSCE-guide-III.doc

Page 192 of 255

OB-GYN

Mother worried about her child weight - Not gaining weight - Not eating well 1- Analyze the CC

Weight analysis: - Weight today, birth, last visit, highest - When did you start to worry about that? Why? - Who noticed it? - Who is the primary care giver? For how long have you been with him? - Do you have his growth chart? If no: if you do not mind, I need to contact his family physician to take a look at his charts Height analysis: - Height today, birth, last visit

2- Impact

-

3- Rule out any serious condition: chronic inf/ dis / malignancy 4- Differential diagnosis

-

5- BINDE

-

6- PMH 7- FH

-

Is he drowsy? Floppy? Is he playful? Active like before? Any limitations? Does he turn blue with activity? Constitutional symptoms! Any congenital or long term disease? Review of systems: cardiac/ chest/ GIT/ urinary/ MSK/ skin/ allergy/ pale/ bleeding Rule out child neglect Difficulty swallowing (CP, Cleft Palate) Chronic loss: chronic Diarrhea (Celiac disease, CF, pancreatic insufficiency) / chronic vomiting (pyloric stenosis in a younger child) Diabetes mellitus (drinks too much water, pees a lot, tired) Chromosomal abnormalities / inborn error of metabolism Scan for risk factors of potential child abuse / neglect Apgar score at birth Diet in details: breast feeding/ formula/ cow milk? Any reason? For how long? Any supplements? Any solid food? Developmental milestones Environment: with whom do you live? Who takes care of the baby? Is he/she capable of doing this? SAD during pregnancy and now

Under weight: - Failure to thrive (FTT): weight decreases first then height will be affected later - Endocrine causes: fat and short - Congenital: everything is small / short, thin with small head Failure to thrive - Weight < 3rd percentile or falls across 2 majors percentiles - Most common cause is inadequate intake

OSCE-guide-III.doc

Page 193 of 255

OB-GYN

Case: A 2 years old boy does not want to eat. The father carries a bag! ─ History: o When you ask about the bag, he says it is for the boy lunch, it is full of candy and a coke. o Details about breakfast, lunch, dinner and snacks o Review of systems  will be negative ─



─ ─ ─

Plan; to improve the health and diet education of the father: o Educate about nutritious food and supplementation o Give brochures and Canada dietary guidelines o Refer to dietician specialist or nurse Recommend supplementation: o Iron o Multi-vitamins Examine the baby Blood works: CBC / lytes / serum iron studies (ferritin, TIBC) Differential diagnosis: o Stresses int the family o Child abuse / neglect o Failure to thrive

Case: A 6 years old developed severe allergy to peanut, child is now stabilized, counsel the father. ─ Is it first time to eat peanuts? Any similar reaction before? Any known food allergy? ─ Review of systems  will be negative ─ ─ ─

Past history of asthma, allergic rhinitis, allergic skin reaction? Family history of allergy? Asthma, allergic rhinitis, allergic skin reaction? Other siblings with allergic reactions?

Management: ─ Will send the boy for allergic testing ─ Strict avoidance of allergens ─ Epi-pen

OSCE-guide-III.doc

Page 194 of 255

OB-GYN

Fever Introduction CC 1- Analyze the CC

2- Impact

3- Red flags

SKIN RASH

4- Differential diagnosis: Review of systems

FEVER - Os Cf D - Any flu at that time? - Any diurnal variation? More at morning or night? - Any special pattern? More every 2nd or 3rd day? - Do you measure it? How many times daily? How do you measure it? - Did you try to give any medications to help? Did it help? - Is it the first time? - Other constitutional symptoms - Other persons at home with the same symptoms? - Is he drowsy? Floppy? - Does he cry? Is it high pitched cry? - Did you notice his suckling is weaker than before? - The fever and constitutional symptoms are already analyzed - Review of systems: DD - Is he tired? - Did you notice any skin rash?  OCD / distribution / color / do you feel it elevated?  Are his shots up-to-date? - Buttocks / abdomen  henoch schonlein purpura / Investigations: urinalysis Treatment: steroids - Trunk  vesiculo-papular  chickenpox - Face: measles / rubella - Cheek: fifth disease - Headache / drowsy / neck pain / rigidity / nausea / vomiting? Does he recognize you? Talk to you? - ENT - Chest: cough / phlegm / SOB / wheezes - Abdomen (pain, distension, diarrhea) / liver (yellow color, itching, dark urine, pale stools) / urinary (urine changes, crying while peeing, loin pain) - Joints: pain / swelling / mouth ulcers - Scan for risk factors for child abuse / neglect

5- BINDE 6- Past medical history 7- Family history

OSCE-guide-III.doc

Page 195 of 255

OB-GYN

Runny Nose / Flu / URTI 8-15 years child is coming to see you with his mom, c/o: runny nose / flu / URTI? Introduction - To BOTH the mother and the child - During the encounter, distribute the questions and interaction between both the mother and the child 1- Analyze the CC - Os Cf D - COCA - What ↑ or ↓ - Is this the first time? Or did it happen before? 2- Impact - Is he playful? Active like before? Any limitations? 3- Red flags - Constitutional symptoms Review of systems: - Rule out infection: Any recent flu-like symptoms? Do you feel tired/ fatigue? History of sinusitis / Pain in your face? Any sneezing? Red eyes? Pain/discharge in ears? Any sore throat/ oral ulcers/ tooth pain? Pale / bleeding - R/O meningitis: Neck stiffness / pain? Headache? N/V? - Cardiac / chest / GIT / urinary / MSK / allergy - Skin rash 4- DD

5- BINDE

6- PMH 7- FH

Physical exam

OSCE-guide-III.doc

-

Allergic rhinitis: runny nose related to seasons, recurrent, no fever Viral flu: respiratory symptoms / joints & muscles ache Viral common cold Scan for the risk factors of potential abuse Immunization School performance Any congenital or long term disease? Other members in the family with symptoms? School contacts?

Mouth ENT LNs Chest exam

Page 196 of 255

OB-GYN

Rash Clinical Presentation Fever, headache, parotitis (bilateral; pushes earlobes up and out), myalgia, malaise Measles Appearance: erythematous maculo-papular rash; Koplik spots Timing: 10-14 days incubation, rash 3 days after start of symptoms Distribution: starts at hairline, spreading downwards; palms and soles typically not involved Rubella Appearance: pink, maculo-papular rash. Timing: 14-21 day incubation; rash 1-5 days after start of symptoms. Distribution: starts on face spreading to neck and trunk. Chickenpox Appearance: macules  papules  vesicles  crusting; all stages apparent (varicella) at once (polymorphous rash)  very pruritic Timing:  10-21 days incubation;  1-3 days prodrome: (fever and respiratory symptoms),  Then rash Distribution: face, trunk, extremities, mucosa, palms and soles. Erythema Appearance: uniform, erythematous maculo-papular rash Infectiosum Timing: 4-14 days incubation, rash 10-17 days after symptoms (fifth disease) Distribution: bilateral cheeks with circum-oral sparing, can affect trunk Roseola Appearance: pink maculo-papular rash (faint). Timing: 5-15 days incubation; rash 3-5 days after symptoms. Distribution: starts at neck and trunk spreading to face and extremities Management: rest / anti-pyretics / fluids / good nutrition Mumps

When does the kid go back to school? Infectivity: 7 days pre-parotitis to 7 days post-parotitis No rash, no fever (infectivity: 4 days prerash) Infectivity: 7 days pre-rash to 5 days postrash Infectivity: 1-2 days pre-rash until vesicles have crusted over

Infectivity: prior to onset of rash

Reye Syndrome:  Acute hepatic encephalopathy and non-inflammatory fatty infiltration of liver and kidney  Mitochondrial injury of unknown etiology results in reduction of hepatic mitochondrial enzymes, diagnosis by liver biopsy  Associated with aspirin ingestion by children with varicella or influenza infection.  40% mortality

OSCE-guide-III.doc

Page 197 of 255

Pediatrics

Delayed Speech Introduction Verbal assessment Rule out any serious condition

BINDE PMH FH

Clarification: is it not gaining words, losing words OR not speaking at all? -

Hearing loss Autism Anatomical: tie tongue / cleft palate

- History of meningitis / jaundice - Family history of deafness or hearing loss

VERBAL ASSESSMENT - Would you please tell me more about that! - When did you start to have concerns? Did you seek medical attention before? - Is the child able to speak at all? How many words is your child capable of using? When did he start to say it? Can he use many words in one sentence? - Was he able to use more words (talk better) and lost them? - How can he communicate with you? What does he do if he wants something? I would like to ask you some questions in order to reach to the cause of this condition: HEARING: - How do you describe his hearing? Does he have hearing difficulties? - If you call him, would he respond and reply? What if you are behind him? What if you are in another room? - Did you notice that he keep increasing the volume of the TV? - Did he get repeated ear infections? Fluids in the ears? Discharge? - Did he take any medications? Any antibiotics (aminoglycosides)? - Was he ever screened for hearing test, when he was born? AUTISM: - Does he maintain eye contact? Does he show emotions? - Is he aggressive? Does he play with other kids? - Does he do repeated movements like rocking, or head banging? - Does he have a favourite toy? How does he play with it? (train / spinning wheels) - Any family history of autism? BINDE: - Start with the development: to rule out MR Developmental (mile stones): - What can he do? When did he start to sit? Crawl? Stand? Walk? Climb stairs? - As a child, did strangers make him nervous? - Does he control his urine / bowel movements? Environment: - Screen for neglect: how many hours you spend with him? Is he a difficult child? - Family factor: how many languages do parents and other family speak at home? Pregnancy / Birth: - Did you have skin rash during pregnancy? TORCH infection? SAD during preg? - Was it complicated labour? Apgar score? - Did he have any special features? Congenital malformations? Cleft palate?

OSCE-guide-III.doc

Page 198 of 255

Pediatrics

Seizing child counselling Refer to the seizing child phone call case in the emergency medicine section for analysis of the event History

Analyze the event Fever Rule out BINDE PMH FH

Counselling

During / before / after Analyze the fever Rule out meningitis / pneumonia

Febrile seizure / epilepsy Introduction Febrile seizures

Introduction: ─ Based on what you have told me, the most likely explanation of your child seizures is a medical condition we call “benign febrile seizure” ─ What do you know about “febrile seizures”? Do you want me to clarify some information about it? In details? Febrile seizures: ─ This condition usually affects kids 6 months to 6 years, it is not uncommon, and a lot of children (around 3%) might have attacks. ─ We do not know exactly the reason for it, but it is related to fever and may be because the children brain is not fully developed at that age, and can not tolerate high fevers. ─ Usually it is self-limited, benign, typical attack is less than 15 minutes, and will not recur in 24 hours. Most children will outgrow their condition after the age of 6 years. ─ Another attack(s): ○ From the studies we know it might happen again; for each 100 child who got 1 febrile seizure attack: o 65 children will not have it again o 30 children will have another attack o 3 children will have many other attacks even without fever o 2 children will develop seizure disorder ○ The best treatment for it is the prevention that is why it is important to make sure that whenever he gets a fever, to seek medical attention and to decrease the fever ASAP using Tylenol or cold foments. Then find the source of fever and treat. ○ In case it happens again: ○ Turn the child on his side / protect him from hitting any nearby object / do not force objects into his mouth ○ Bring to ER if seizure does not stop within 15 minutes ○ Diazepam 5 mg PR suppository ○ If repeated attacks, we may consider prophylactic anti-convulsion therapy ○ Will do CT, EEG ─ I will give you some brochures and web sites in case you want more information. ─ Any other questions or concerns.

OSCE-guide-III.doc

Page 199 of 255

Pediatrics

ADHD counselling The father comes to you saying that his son was diagnosed with ADHD two days ago and he has concerns about ADHD and Ritalin. Counsel for 10 minutes. Introduction To diagnose ADHD: - 2 settings (school / home) Address concerns - > 6 months duration  Diagnosis (symptoms of ADHD) - < 7 years old child  Impact Differential diagnosis:  Differential diagnosis - ODD /+/ Conduct disorder BINDE - Specific learning disability PMH - Seizures (petit-mal epilepsy) FH  ADHD / MR / autism / depression - Depression Conclusion Introduction: - Who diagnosed it? Usually teachers recognize it first (pick it), but to make a diagnosis a psychiatrist, paediatrician, or a specialized nurse assessment is needed Concern – do you give Ritalin (which is amphetamine) to children? - Actually yes. A lot of children use Ritalin, it is the first line of treatment for ADHD, and it is effective and has been used for long time. - It is not exactly amphetamine, it is the same family, it is called “methylphenidate” and it is approved for this indication. - In children, it helps them to focus as increases their concentration and channels their energy, this is crucial for children, as it allows them to do better in schools. Even though we might not be able to cure all children with ADHD, by we try to help them with education, so that they can have career and live independently in the future, without problems with the law. - It is generally a safe medication in children. It is not addictive, and we can stop it at any time. However, like any other medication, it has its side effects, that include: insomnia (that is why we give it early), abdominal pain, and not all children improve on it. Before talking further about ADHD and Ritalin, let me first ask you some questions to see if your child meets the criteria of ADHD or any other developmental challenge: Diagnosis (hyperactive / inattentive / impulsive): - Did the teachers complain that your child is full of energy? Spinning all the time? Refuse to stand still? Talk all the time? Answers even if he is not asked? Does he stand in-line or does he break the queues? - Can he focus on one subject for > 30 minutes? Can he finish his tasks (e.g. the homework)? Does he jump from one activity to another without finishing it? Does he lose his stuff? Does he forget his belongings? - Is this only at school or also at home? - Did you notice that yourself? - How much time do you spend with him? How about the mother, is she involved? - How about before? Did anyone mention that or no? IMPACT: - Impact on functioning, school performance, relationship with peers

OSCE-guide-III.doc

Page 200 of 255

Pediatrics

Differential diagnosis: ODD Conduct disorder Learning disability Petit-mal epilepsy Depression Autism MR -

Does he like not to follow the instructions? Does he like to challenge his teachers and other family members? Is he aggressive? Does he fight a lot with other children? Does he have a pet? How does he treat his pet / or other pets? Did you notice that he takes others’ belongings without telling them? Does he tell the truth all the time? Does he like to set fires? Does he like to go to school? Does he have specific difficulty in reading / writing / mathematics? Does he have a history of seizures? LOC? Abnormal movements? Was he stressed recently? Any loss of a beloved one? Is he sad? Crying? Nightmares? Losing weight?

BINDE: detailed developmental history Conclusion: - I am really sorry for this loss; it must be difficult for children in his age to go through all that. How is he/she coping with that? He has NO symptoms OR does not fulfill the criteria: - Based on what you have told me, and the fact that it happened > 8 years old, it is less likely that he has ADHD. - On the other hand, his symptoms and behavioural changes could be related to the loss of his mother, which might lead to depression. And in his age group, depression may manifest as behavioural changes like what he is experiencing right now. - We can refer your child to a child psychiatrist who can help him deal with that. He has symptoms: - Based on what you have told me, your child symptoms meet the criteria for diagnosis with ADHD. However, this is not uncommon condition, and there is medical treatment for it, in which the first line is Ritalin. - Counsel on Ritalin. Notes: - Whenever you hear that one of the parents has passed away  show empathy. I am sorry to hear that, it must be difficult for children in his age to go through all that. How is he/she coping with that? How are you coping?

OSCE-guide-III.doc

Page 201 of 255

Pediatrics

Vaccination counselling New comer to Canada, comes to you as she has some concerns about vaccinations Introduction / welcome her / how do you feel? Speak with enthusiasm (to encourage) with three Identify the language barrier counselling sessions: Identify concerns - Pap smear - Deal with concerns one by one - Breast feeding - Pose frequently and ask if she has any questions - Vaccination Candidacy for vaccination Mother vaccination Otherwise, speak neutrally ○ What are vaccines? ○ How do we vaccine? ○ Side effects of vaccines Introduction / welcome her / how do you feel? - Good evening Mrs …vich, my name is Dr … I understand that you are a new comer to Canada, and you came to the clinic because you have some concerns about vaccinations. We will discuss all you concerns. First of all, welcome to Canada, for how long have you been here? How do you feel being here? Identify the language barrier - Before we proceed, am I clear, or do I need to talk slower? We can arrange for an interpreter or a family member, if you would like to. Identify concerns - Now, can you tell me more about your concerns? - Do you need general information, or do you have specific concerns? o I heard that vaccines cause autism! o I think we do not have these diseases in Canada, why should we give the vaccines for diseases not common here? Thanks for coming here to discuss your concerns with me. Vaccines cause autism! - What gave you this feeling? Concerns? - There is misinformation among the general population that there is a connection between vaccines and autism. And the origin of this misinformation is a study done in England many years ago, the study found there is a connection between autism and 1 type of vaccines; namely the MMR. - And because we take vaccines very seriously, a much larger study was done, in large number of countries, including very large number of children. Now we found for sure that there is no connection between vaccines and autism. The only relation is a coincidence between the age in which parents start to notice autism symptoms and the age we start to give MMR. - When we tried to figure out why the first study found the connection, the explanation of that was a bias in the selection by the author and the study was conducted to favour this outcome. Another theory to explain the connection was the preservative used in the vaccine (Thiomersal) and it contains mercury. However Canadian vaccines do not contain it. - I can assure you that there is no connection between vaccines and autism. Any questions till now? Why to vaccine against diseases not preset in Canada? - Even though we do not have these diseases commonly in Canada, and this is because we have a successful vaccination program. But because the world is a becoming more and more a small village, and people travel easily from a place to another, we do not want your child to be vulnerable to preventable diseases if there is any outbreak anywhere. And I will give you some examples: o In England, people stopped to vaccinate their children for MMR, they started to have measles infections again, and some children ended up losing their lives. o Diphtheria is also coming back because of lack of vaccinations, even in many European and developed countries.

OSCE-guide-III.doc

Page 202 of 255

Pediatrics

Tetanus is universal; it is in the soil everywhere, if a child is injured, the injury is vulnerable to be contaminated with tetanus, which has serious fatal consequences. Do you have any questions or any more concerns? o

-

Candidacy for vaccination / history – BINDE: Before I proceed further, I would like to make sure that your child is a good candidate for vaccination, and we usually start vaccination at 8 weeks. - Does your child have any illness, sickness, fever, flu-like symptoms? If high fever, we wait. Did he get recurrent infections before? (if not very young) - Was he diagnosed with any neurological diseases? - Was he scanned for HIV? Were you scanned for HIV? Is there any risk that you might have HIV? - Does he have any allergy? Based on what you told me, your child is a candidate for vaccination. - Is it ok till now? Mother vaccination: - As a child, were you vaccinated? How do you feel about that? - If it is ok with you, we can set up a follow-up meeting to discuss in details your vaccination status and find what vaccination(s) you might need to take. What are vaccines? - Vaccines are one of the most significant preventive interventions in medicine. It helped to save the lives of large number of children all over the world. As children are vulnerable to be infected with many infections, even thought it might affect adults, but usually with no serious consequences as compared to children! - We have large number of diseases and infections, caused by large number of bugs; some of them cause serious illnesses. We try to protect against many of it. We take the different bugs (bacteria and viruses), and we either kill it or we make it so weak so it will not cause any harm, then we inject it into our bodies by a needle. Usually the body reacts by forming “anti-bodies” which are protective chemicals against these bugs. Later in life, if the child will get exposed to the real bugs, these anti-bodies will protect him. - Was I clear? Any questions? How do we vaccine? - Because we have large number of diseases, and each one needs more than one shot, we try to decrease the number of shots to be given. - For example, there is a shot called “pentacel” that stimulates the body to produce protective antibodies against 5 diseases, it protects against diphtheria, tetanus, polio, pertussis and hemophilus influenza B. It is to be given at 2, 4, 6, 12 and 18 months and another booster dose at around 5 years. - You do not need to worry about memorizing them; I will give you a schedule of the required immunizations to follow with his family doctor. And these vaccines are covered by OHIP. - Any questions or concerns do you have? Side effects of vaccines - Vaccines are generally safe, millions and millions of children were and are vaccinated and it saved the lives of large number of them. - However, like any other medical intervention, vaccines have some side effects. - It may cause minor issues, like pain, redness and swelling, at the site of injection, it may cause fever, which we deal with Tylenol for pediatrics. - There are some few rare side effects that include prolonged crying, seizure, and some children become floppy, again, this is rare, but if this happens, you need to seek medical attention immediately. - Lastly, very rarely, children experience a serious allergic reaction, and that is why we keep the children under medical supervision for 20 minutes after vaccination. I hope that I answered you concerns, and now you have a better idea about vaccinations. Any other ideas or concerns you need to talk about? Thanks for coming and see you next visit for the first vaccination shot.

OSCE-guide-III.doc

Page 203 of 255

Pediatrics

Child with DM counselling Father of 6 years old boy comes to you as he has some concerns about his diabetic son History

-

Counselling

-

Complications Management

When was the child diagnosed? How? What type of insulin? Is DM controlled? Regular measure of blood glucose? Hb A1c! Symptoms of DM Symptoms of DKA Symptoms of complications: nephropathy / retinopathy / neuropathy (usually develops more than 5 years after diagnosis) Do you have special concerns? What do you know about diabetes? Did you read? Hear? Anything? Type 1 DM Insulin DKA /+/ Hypoglycemia /+/ Prevention of complications Macro and micro vascular complications Meal plan, education, exercise, psychological support Insulin injections with BS monitoring Children are more prone to brain damage with hypoglycaemia, therefore high target range in younger and tight control in older kids

Diabetic ketoacidosis (DKA): - Diabetic ketoacidosis (DKA) is an acute, major, life-threatening complication of diabetes. DKA mainly occurs in patients with type 1 diabetes, but it is not uncommon in some patients with type 2 diabetes. - DKA is a state of absolute or relative insulin deficiency aggravated by ensuing hyperglycemia, dehydration, and acidosis The most common causes are underlying infection, disruption of insulin treatment, and new onset of diabetes. - DKA is defined clinically as an acute state of severe uncontrolled diabetes associated with ketoacidosis that requires emergency treatment with insulin and intravenous fluids. - Symptoms of hyperglycemia associated with diabetic ketoacidosis may include thirst, polyuria, polydipsia, and nocturia. - Signs of acidosis may include shallow rapid breathing or air hunger (Kussmaul or sighing respiration), abdominal tenderness, and disturbance of consciousness. - Signs of dehydration include a weak and rapid pulse, dry tongue and skin, hypotension, and increased capillary refill time. Hypoglycemia: - Hypoglycemia is a syndrome characterized by a reduction in either plasma glucose concentration or its tissue utilization to a level that may induce symptoms or signs such as altered mental status and/or sympathetic nervous system stimulation. - Symptoms of hypoglycemia may be categorized as neurogenic (adrenergic) or neuroglycopenic. Sympathoadrenal activation symptoms include sweating, shakiness, tachycardia, anxiety, and a sensation of hunger. Neuroglycopenic symptoms include weakness, tiredness, or dizziness; inappropriate behavior (sometimes mistaken for inebriation), difficulty with concentration; confusion; blurred vision; and, in extreme cases, coma and death

OSCE-guide-III.doc

Page 204 of 255

Pediatrics

Bed wetting counselling / Nocturnal Enuresis Introduction 1- Analyze the CC

-

3- Rule out infection 2- Impact

4- DD

-

-

5- BINDE

-

6- PMH 7- FH

OSCE-guide-III.doc

-

OS CF D: - When did it start? How did it start? Sudden or gradual? - Frequency - Primary or secondary (dry period(s) of time)? - Is it continuous or on and off? How often? Day and night? Every day? Every night? ↑↓ Factors: stress / drinking too much fluids before bedtime How does Mom feel about it? How does the child feel about it (impact of this on child)? Constitutional symptoms! Did you notice if your child has fever or skin rash? Odd smell or colour of urine? Pulls his penis? Cries while peeing? Rule out child neglect / abuse Medical conditions

-

BINDE screening Parent attitude! DM (drinking too much water / going more often to pee / feeling tired / losing weight) - Diabetes insipidus (history of meningitis / brain infection / head trauma) - UTI (detailed in No 3) - Neurological: trauma or surgery to back / bowel dysfunction / leg weakness or numbness - Seizure disorder Stressors - New sibling - Home / school change - School performance Very briefly because the child is more than 6 years old - Scan for risk factors for child abuse / neglect How is his school performance? Who is the primary care giver, who else does live with them at home, is he the only child, any sisters or brothers? Kidney disease Kidney disease Bed wetting DM Seizure disorder

Page 205 of 255

Pediatrics

Counselling

-

Management

-

-

OSCE-guide-III.doc

The condition is common; - By the age of 4 years, 25% of children have this problem - By the age of 7 years, 5-10% of children have this problem - More common in boys than girls May be it is due to regression of his development because of the current stresses in his life Condition is usually self limiting & you need to give the child some time & he will adapt very well to the changes Try behavioural modification for 3 months: - Avoid drinking late before sleep, - Try to wake him up to go to the washroom, - Document dry nights and use reward system. Alarm therapy Pharmacological measures: - Desmopressin (DDAVP): 1 hour before bedtime. Be careful to water intake, the only serious side effect is seizures due to water intoxication - Oxybutynin: at bed time, it ↓ bladder contraction therefore it leads to ↑ bladder capacity

Page 206 of 255

Pediatrics

Breast feeding counselling  

 

A lot of enthusiasm Congratulation for the news, how do you feel being a (prospective) mother? It is good to hear that you plan to breast feed. Do you have any concerns? I need to ask some questions ─ If still pregnant: ○ How the pregnancy is going so far? ○ Pregnancy questions of BINDE ○ Ante-natal care follow-up (from OB-GYN) ─ If already delivered: ○ Birth questions of BINDE PMH: medications (lithium, antibiotics, anti-convulsions) & allergies Social history: smoking, alcohol, drugs / work and home environments

1- Rule out contra-indications for breast feeding: - The mother has active herpes simplex on breast - The mother has untreated, active T.B. - The mother has been infected with HIV / is taking antiretroviral medications - The mother is using or is dependent upon an illicit drug - The mother is taking prescribed cancer chemotherapy agents, such as antimetabolites that interfere with DNA replication and cell division OR is undergoing radiotherapy; however, such nuclear medicine therapies require only a temporary interruption in breastfeeding - An infant diagnosed with galactosemia, a rare genetic metabolic disorder - N.B.: Hepatitis B is NOT a contraindication to breastfeeding 2- Counselling: Advantages of breast feeding - Highly nutritional, providing all elements baby needs (especially colostrum), breast milk contains: more vitamin C, easily absorbable iron, less protein load on the baby - Contains antibodies to help your child fight infections - Ready, worm, clean, economic, sterile - Less allergic - Secures bonding between mom and baby, emotional satisfaction for the mother and creates sense of security for the baby - Help mom reduces weight, a method of contraception





Problems with breast feeding that might happen to the mother: - Sore nipple (clean & apply soothing lotion) - Engorgement (pump) - Mastitis (treat & do not stop feeding)

OSCE-guide-III.doc

Page 207 of 255

Pediatrics

3- Advice: - Mother should get enough nutrition, fluids, vitamins and rest. - Give supplementations of: o Vitamin D from day 1 o Iron from 4 – 6 months o Start solid food from 4 – 6 months, I will give you a table with the recommended time and types to start solid food - Mother can use OCP but it will reduce amount of milk OR use an IUD - Avoid using any medication without asking your Doctor - Avoid smoking & alcohol - Care of the breast: frequent cleaning with water and proper hygiene, warning signs: engorgement, tenderness, redness, hotness - I will give brochures & information about BF classes - I will give you the immunization schedule so that you remember to bring him for followup and for vaccination - Do you have any questions or concerns? 4- Frequently asked questions about breast feeding: How often you should feed your child? - On demand at the beginning - Then the child will adapt to a schedule of every 3 – 4 hours





How long should the baby stay on each breast? (10 minutes)



How do you know that your baby is feeding well? - Gaining weight - Sleeping well - Wetting 5 – 6 diapers daily What do you do if you do not have enough milk? (You can supplement with formula)

 

How can you breast feed & work at the same time? - Use pump & keep the milk in a bottle for 3 – 6 hours outside and 24 hours in a fridge, you can keep it in the freezer

OSCE-guide-III.doc

Page 208 of 255

Pediatrics

Psychiatry

OSCE-guide-III.doc

Page 209 of 255

Pediatrics

Mental status exam – the psychiatry interview ABS MAP TCIJ ALWAYS clarify! What do you mean by that? Appearance Dressed Dishevelled: dressed and groomed poorly Groomed Given age matches his chronological age Dressing matches the weather Behaviour Psychomotor retardation vs. Psychomotor agitation Eye contact * Cooperative * Hostile * Abnormal movements / jerks / tics / lip smacking (anti-psychotics) Speech Volume (low / normal / loud) (process of Tone (parkinsonism: monotonous) talking) Articulation Fluent Pressured speech Mood How is your mood? How do you feel? Write it in patient “own words” Affect Quality (by pt observation) Elevated Euthymic Depressed Anxious Congruency Appropriateness Others Stable / range / labile / flat Perception Normal perception Illusions Hallucinations Thought Processing (how does the patient connect Goal directed ideas); e.g. how did you come here today? Thought block Flight of ideas Loose association Tangentiality Circumstantiality Contents Obsessions Delusions Suicidal / homicidal ideation Cognition Mini mental exam Insight Judgement Brief comment: (1) The patient is well dressed, well groomed; and his appearance matches his chronological age. (2) He has (good / poor) eye contact, cooperative (not), with psychomotor … (retardation / agitation) (3) His speech is of normal volume, tone, fluent, not slurred, and not pressured. (4) His mood is … (5) His thoughts are organized (or disorganized). (6) There are no delusions or hallucinations. (7) There is no suicidal ideation or homicidal thoughts. (8) Judgement (good / poor), insight (intact / lost).

OSCE-guide-III.doc

Page 210 of 255

Pediatrics

Perception Hallucinations: - Visual: o Usually organic (tumour / epilepsy / cocaine and amphetamine) o Brain tumour /+/ alcohol intoxication / DT /+/ cocaine / hallucinogens o Do you see objects / things that others do not see? o Can you describe what do you see? o Do they give you any messages? o Are these messages asking you to harm yourself or anyone else? - Auditory: o Usually schizophrenia o Do you hear voices / things that other people do not hear? When you are alone, do you hear voices coming from your head? o How many voices o Are they familiar or not? o Are they talking to you or about you? What are they telling you? o Did they ever ask you to harm yourself or somebody else? What is preventing you from doing this? o How do you feel about these voices? - Tactile: o Cocaine chronic use (most probably) OR delirium tremens o Do you feel ants / insects crawl on your body / skin? - Smell: usually epilepsy Though Processing: o

How did you come here today?

Content: + Obsessions: - Repeated intrusive thoughts that the patient knows it is wrong, and he can not resist, if he resists  ↑ anxiety  take actions to try to ↓ anxiety (compulsions) - Mostly regarding: cleanliness, contamination / order / checking / … o Do you have any repeated thoughts or images that you find difficult to resist? About what? What do you do? + Suicidal / homicidal ideation: o Do you have any thoughts or ideas of harming yourself? o Or harming other people? o Any access to weapons? - If the patient is suicidal / homicidal / can not take care of himself  admit, if he/she refuses  form 1 (for involuntarily admission – for 3 days – for psychiatric assessment – by another physician). Form 1 has to be filled within 1 week from seeing the patient. - I want to file form 1 for the patient and call the hospital security to bring the patient back. I have concerns about safety of the patient and other people.

OSCE-guide-III.doc

Page 211 of 255

Pediatrics

+ Delusions: - False fixed believes, that do not match with the patient cultural and religious background - You can not convince the patient it is wrong, even with proof - The ideas o Believable (could be) – non bizarre o Unbelievable (could never be) – bizarre o Do you believe that other people would like to harm you? OR conspire against you? o Do you think that others would like to control you? o Read your mind? Thought broadcasting o Put thoughts into your head? Thought insertion o Steal thoughts from your head? Thought withdrawal o If you are watching the TV or reading the newspaper, do you believe that they are talking about you? Delusion of reference o Do you believe that you are a special person? With a special talents? Or special power? Do you believe that you have a special mission to do in life? Do you think you deserve to be treated specially? Grandiosity o Do you feel other people are falling in love with you? Eromantic o Do you believe any part of your body is rotten? Cognition: - Are you becoming forgetful? Are you losing your staff? - Assess abstract vs. concrete thinking! Insight: - Do you think that you are doing well? Or do you need help? Judgement: - If there is a fire in the building, what are you going to do? - If you find a stamped and addressed envelop on the ground, near the mail box, what would you do? General screening: - Depression: o What is your mood? How do you feel? o Did you lose interest in things that were interesting to you before (e.g. certain hobby, playing something)? - Anxiety: o Are you the kind of person who worries too much? o Do you have excessive fears or worries? - Psychosis: o Do you hear voices or see things that others do not? o Do you think that someone else would like to hurt you?

OSCE-guide-III.doc

Page 212 of 255

Pediatrics

DSM-IV-TR Diagnostic and Statistical Manual of Mental Disorders 4th Ed/2000 – Text Revision Multi-axial system (5 axes) The DSM-IV organizes each psychiatric diagnosis into five dimensions (axes) relating to different aspects of disorder or disability: - Axis I: Clinical disorders, including major mental / psychiatric disorders, and learning disorders, Substance Use Disorders - Axis II: Personality disorders and intellectual disabilities (although developmental disorders, such as Autism, were coded on Axis II in the previous edition, these disorders are now included on Axis I) - Axis III: Acute medical conditions and physical disorders - Axis IV: Recent stressors, i.e. psychosocial and environmental factors contributing to the disorder - Axis V: Global Assessment of Functioning or Children's Global Assessment Scale for children and teens under the age of 18 (a questionnaire) Example of a full proper psychiatric diagnosis: - Bipolar I / Anti-social personality / DM+HTN / Divorce / global assessment was not done because the patient was not cooperative Diagnosis of diseases based on DSM-IV-TR is based on CRITERIA and TIME. - Depression: o MI PASS ECG or MIS GE CAPS o You need to find at least 5 of the 9 for > 2 weeks, including at least one of the Mode or Interest. o If not fulfilling these criteria: non-specified mood disorder o In teenagers: we do not need M or I, we can replace it with agitation OR drop in school performance + other 4 criteria. - Schizophrenia: o 4 positive symptoms: hallucinations, delusions, disorganized speech, disorganized behaviour. o 1 other category; negative symptoms: mood, catatonia … o At least 1 month of active symptoms (2 of 5) + 6 months of deterioration in functioning. o 1 active symptom (not 2) is accepted in the following cases:  If the hallucinations are > 2 voices (commanding or commenting)  The delusions are bizarre - Anxiety: 1- Panic attack vs. panic disorder: a. In panic disorder, there is at least ONE panic attack with at least ONE month of worries and fears of having it again b. Panic attack might be one or more attacks c. If patient is avoiding going outside  with agoraphobia 2- Phobias specific to certain objects 3- OCD 4- GAD: excessive unrealistic fears for more than 6 months PLUS other manifestations 5- PTSD (acute or chronic): Have you ever encountered a situation in which your personal or mental safety and wellbeing were endangered? When? Do you have flashbacks or nightmares?

OSCE-guide-III.doc

Page 213 of 255

Pediatrics

History taking – Psychiatry MOAPS: mood / organic / anxiety / psychosis / serious conditions (self care, suicide, homicide, support) / HEADSSS Major psychiatric illness Suicide Minor psychiatric illness Personality disorder / Drinking / addiction / Eating / sleeping S disorders / Somatic disorders / A Mood Anxiety Psychosis Cognitive (delirium / D dementia) ... P Criteria  Low: MI PASS ECG 1- Panic attack vs. panic Criteria (1 month of 2-5 active E  High: DIG FAST + disorder symptoms + 6 month of R MI PASS ECG 2- Specific phobias function deterioration) S 3- OCD - 1st time or did you have it - 1st time or did you have it O 4- GAD before? before? N 5- PTSD - What about the opposite? S Dx: one of the mood disorders Past psychiatric history Past psychiatric history MOAPS Organic: 1- SAD if IV drug use: check for liver (hepatitis) / constitutional symptoms (HIV) 2- PMH, including constitutional symptoms 3- Rule out medical conditions as DD, e.g. medications and specific diseases Thyroid disease Mitral valve prolapse Brain tumour / HIV Anxiety / psychosis Mood / psychosis Mood / anxiety Serious conditions (red flags): - Self care (are you eating / sleeping well?) - Suicidal / homicidal ideation Social history: - How do you support yourself financially? - With whom do you live? Family support? For teenagers, add: HEADSSS Family history of psychiatric illness: suicide / depression / SAD / seen by psychiatrist

OSCE-guide-III.doc

Page 214 of 255

Psychiatry

Mood disorders:

OSCE-guide-III.doc

Page 215 of 255

Psychiatry

Psychosis Pt comes to the clinic complaining of strange feelings in his right hand Clarify the CC OSCD

Criteria :  Hallucinations  Delusions 1st time or did you have it before? Past psychiatric history Organic

Mood / Anxiety Serious conditions Social history Family history

123456 

Can you tell me what is going on! What do you mean? Is it pain, numbness, tingling? OS: When did it start? What were you doing? C: Is it all the time, or on and off? Any specific setting? In you opinion, why do you have this? In addition to …, do you have any other strange feelings? Any hallucinations: What? For how long? Any (tactile or visual) hallucinations  will be mostly organic  cocaine (substance abuse) until proven otherwise

Have you seen a psychiatrist before? Cover the following:  Head injury / trauma  Brain infection (fever, vomiting) / Brain tumour (N/V)  HIV  Thyrotoxicosis  Medications: thyroxin, anti-parkinsonism (L-dopa)  SAD (cocaine, marijuana, amphetamines) Constitutional symptoms / self care / suicide

Differential diagnosis: - Schizophrenia (a mental disorder that impairs the way you perceive reality. It could be very disabling) - Brief psychotic disorder - Post-partum psychosis - Drug-induced - Brain tumour - HIV - Delirium / dementia - Mood disorder Investigations: - CBC / toxicology screen - HIV / syphilis test - Septic workup - CT / MRI brain Pay attention to patient cues - Poor hygiene - Looking at wall or ceiling - Paranoid - Talking to some body.

OSCE-guide-III.doc

Page 216 of 255

Psychiatry

Management: ─ Will examine and do some tests o Because you have stopped your medications, it looks like your schizophrenia is relapsing, that is why we need to admit you and refer you to psychiatrist to reassess your condition, how do you feel about that? ─ Will start medication which is helpful in reducing the symptoms (Risperidone). Explain about side effects: weight gain / ↑ blood glucose level / ↑ cholesterol / drowsiness ─ Arrange follow up visit ─ Information e.g.: support groups / brochures Notes: ─



Whenever you suspect substance abuse: after you ask “have you ever tried recreational drugs?” ask “what about crack cocaine? Do you sniff? Do you inject? Did you share needles” o If shared needles  scan for hepatitis (liver symptoms), HIV (repeated infections / repeated diarrhea) If the patient came because his parents or roommate have concerns, you can ask the patient: what kind of concerns does … have?

Difficult situations: ─

─ ─







If the patient with hallucinations tells you that he sees a radiation and gives you a photo and asks: do you see it doctor?  For me it does not look like radiation, but I can understand that you see this as radiation At any time the patient starts to agitate and worries about special hallucinations! o You are safe here, no body will harm/hurt you If the patient is away: o Do not chase him/her around the room, stand by your chair o I would like to assure you that you are safe here, no one will harm you I do not like “Egyptian people”, by the way, are you Egyptian doctor? o Why are you concerned about that? o Whether I am Egyptian or not will make no difference in this situation I do not like “gays”, by the way, did you see a gay patient today doctor? o Why are you concerned about that? o As a physician, I deal with all patients, regardless their race, religion, sex, sexual orientation or anything else! Do you think I am crazy doctor? o There is no medical term called “crazy”. However sometimes some people have difficulties in the way they handle their thoughts and the way they interact with and perceive reality, we call that schizophrenia. It is a mental illness like any other illness that can affect the body, that we can treat with medications

Case: A young man can not move his neck, DD acute dystonia: ─ Trauma ─ Meningitis ─ Subarachnoid hemorrhage ─ Cervical disc ─ Muscle spasm ─ Anti-psychotic medication (e.g. haloperidol), treatment: lorazepam Case: Patient is in the balcony, wants to fly, mother is calling you?! What do you tell her? Talk to him to attract his attention ─ The doctor should get the phone number and address and ask the nurse to call 911 ─ Ask her if you can speak with the patient  psychosis patient ─

OSCE-guide-III.doc

Page 217 of 255

Psychiatry

Schizotypal personality disorder -

Delusions Magical believes Limited number of friends that share the same believes

Ethical challenges: - Will you hook me to the cleaning machine that cleans the blood? I am glad you came here today, I think you need help, but not with the machine. - Will you admit me doctor? We need further psychiatrist assessment then we may need to admit you.

OSCE-guide-III.doc

Page 218 of 255

Psychiatry

Panic attack Patient comes to the clinic complaining of dizziness Clarify the CC

Analysis of CC

HPI

Criteria during attack (AS)

Between attacks Anxiety

MOAPS

When you say dizziness, do you feel: - light headed - Spinning Os Cf D Analyze the attack: How did it end? How many attacks? Are they similar? What were you doing? When was your last one?  ↑ cardiac: heart racing17 / chest pain / tightness / excessive sweating  ↑ respiratory: SOB / wheezing  ↑ neurology: dizziness / numbness / tingling / weakness / shaky / buzzing sounds / headache / vision changes / difficulty balance  ↑ GIT: nausea / vomiting / difficulty swallowing  Depersonalization: you feel that you are outside of your body  Derealisation: feel things around you are strange / not real  Excessive fears of: losing control / going crazy / dying  Do you have fears of having other attacks?  How does it affect you? Do you avoid going out? (Relation to agoraphobia?)  Are you the kind of person who worries a lot? Excessive fear  Are you under any stress in your life? How can you cope with this?  Any special fears? High altitudes? Closed places? Talking in public? Pets?  Have you ever encountered a situation in which your personal or mental safety and wellbeing were endangered? When? Do you have flashbacks or nightmares?      

Past psychiatric history Serious conditions

Hypoglycemia Thyroid disease Pheochromocytoma SAD (cocaine / amphetamine / alcohol withdrawal) / Caffeine Arrhythmias / MVP Anemia (fatigue / light headedness / heavy menses / PMH anemia)

Constitutional symptoms Self care  Suicide AMPLE Heart diseases / thyroid / abdominal tumours  

PMH Family history Social history

17

Any heart racing, ask the patient “can you tap it for me”, then comment to the examiner: “it looks regular / irregular for me”

OSCE-guide-III.doc

Page 219 of 255

Psychiatry

COUNSELLING -

-

-

-

-

With what I heard from you today, the most likely diagnosis to your symptoms is a medical condition that we call “panic attack”. We still need to do physical examination, some investigations like blood works, urine analysis, electrical tracing of your heart (ECG), to exclude other medical conditions and to confirm our diagnosis. o Now Mr … what do you know about “panic attacks”? o Do you want me to explain this in details over the next few minutes? Inform the patient: o Explain the pathophysiology: panic attack or panic disorder is a kind of severe anxiety, it happens suddenly, in attacks. Usually it is related to stress. o It is due sympathetic over-activity, imagine you are crossing the road, and a speedy car is approaching you, normally, our body reacts to this by enhancing the sympathetic nervous system, which leads to some changes: increase in the heart rate, rise in blood pressure, and you feel alert. This is normal and useful reaction. o The same reaction might happen suddenly without any external trigger, and this would be stressful, and this is what we call a “panic attack”. o Consequences: this might happen again / may cause significant limitations Preventive measure: o Life style modification (↓caffeine and alcohol / better sleep hygiene) o Relaxation techniques (e.g. breathing techniques / meditation) Treatment: o Like many other conditions, it could be treated. o Treatment varieties include:  Talk therapy  Medications: 2 types • Anti-anxiety: Lorazepam 0.5 mg qhs x 2 weeks (it is important to use it on schedule, not irregularly) • SSRIs: Paroxetine 10 mg od x 4 weeks – similar to what we usually use with depression. Like any other medication, they have their side effects; GIT disturbances, headache, some sexual dysfunction. And this improves by time. • Follow-up 2-3 weeks Offer more information: brochures / web sites Whenever you suspect social problems  involve the social workers

Generalized Anxiety Disorder (GAD)  Excessive anxiety and worry (apprehensive expectation), occurring more days than not for at least 6 months, about a number of events or activities (e.g. work / school)  The person finds it difficult to control the worry  Treatment: o Lifestyle: caffeine and alcohol avoidance, sleep hygiene o Psychological: psychotherapy, relaxation, mindfulness, and CBT o Pharmacological:  Benzodiazepines (short term, low dose, regular schedule, long half-life, no PRN) o Buspirone (tid dosing) o Others: SSRIs/SNRI (paroxetine), TCAs (nortriptyline), beta-blockers o Avoid Bupropion due to stimulating effects

OSCE-guide-III.doc

Page 220 of 255

Psychiatry

Tiredness OR weight loss Introduction CC Clarify the CC

Os Cf D

Ask about sleep

Tiredness - Is it weakness? Can not do? - Lack of energy? Tiredness? - Limitation of activity? How many blocks are you able to walk? - Not being refreshed after sleep? Do you have any special concerns? Timing: - Morning or all day: ?depression - End of the day: organic - How many hours? And before? - Find difficulty falling asleep? - Do you wake up during night? - When you wake up, do you feel refreshed? Do you need naps?

↑ sleep /or/ the same ? organic - Constitutional symptoms - Tender points in your body All systems review (head to toe): - Cardiovascular - Lung - GIT / liver - Urinary - MSK / skin / rheumatology / autoimmune - Anemia / bleeding ± LMP / mens. hx / pregnancy Cancer colon for males - Endocrine (thyroid / DM)

↓ sleep / insomnia ? depression Criteria : MI PASS ECG 1st time or did you have it before? What about the opposite? Past psychiatric history Organic …/…/… Anxiety / psychosis Serious conditions Social history Family history

Counselling on depression

MI – mood / interest PMH of cancer Social history – SAD Family history Diabetes Mellitus: - Hx of DM Fluctuations (acute)

- Symptoms: Complications (chronic) vascular MICRO MACRO - Eat more - Blurred vision DKA - Nephropathy - CAD - Drink more - Tired Hypoglycemia - Neuropathy - CVS screen - Pee more - Weight loss - Retinopathy - PAD / impotence N.B. β-blockers are contraindicated in DM: it causes hyperglycemia / and it masks hypoglycemia

OSCE-guide-III.doc

-

Blood sugar measured Emergencies

Page 221 of 255

Psychiatry

Sleep / fatigue notes 1- Sleep hygiene questionnaire: - How many hours? How about before? - How does this affect your life? Do you work night shifts? empathy Before - When do you go to bed? - Find difficulty falling asleep? How long does it take you? - Do you sleep in dark room? Lit room? - Do you eat before sleeping? Heavy meals? Late meals? - Do you drink before sleep? Alcohols? Coffee? - Do you exercise before sleep? - Do you read in bed? Watch TV? During - Do you wake up during night? Any reason? Can you sleep again? - Do you sleep alone? Or do you have sleep partner? o Does he notice you are snoring? Do jerky movements? o Does he snore? Does he do jerky movements? - Do you have dreams? Nightmares? After - Do you wake up early? - When you wake up, do you feel non-refreshed? Do you need naps? - Do you work on shifts? 2- Depression:  Psychomotor question: do you think things take more time to do now? Compared to before?  Pancreatic cancer  depression  Whenever you find alcoholic patient  check for complications: i. Cancer pancreas ii. Liver damage (↑ liver enzymes) / hepatitis / cirrhosis / carcinoma iii. GIT: upper GIT bleeding / peptic ulcer perforation iv. Depression (alcohol / depression / suicide) is common combination  Treatment for depression (or most of the psychiatric diseases): i. Life style modification ii. Talk therapy iii. Medications Usually in combination 3- Domestic abuse presentations - Headache - Abdominal pain - Insomnia / sleeping pills - Vaginal bleeding - Fatigue

OSCE-guide-III.doc

4- Fatigue - Depression / PTSD - Domestic abuse - Hypothyroidism - Fibromyalgia - Anemia: - Old person: think cancer & occult blood - Young female: think menorrhagia - Diabetes Mellitus, polyuria - Anorexia nervosa

Page 222 of 255

Psychiatry

Insomnia A lady complaining of insomnia Common presentation to: domestic abuse / depression / anxiety Introduction CC Clarify the CC

Insomnia / Tiredness - Difficulty falling sleep - Waking up Analysis CC: Os Cf D - More at certain time of the week? - Did you try anything to help? Did it work? Ask about sleep Sleep hygiene questionnaire Anxiety - Do you have too many worries? - What comes in your mind before falling asleep? - Any changes / stresses in your life? - Do you wake up with nightmares? Depression - Screen with MI; if positive  screen MI PASS ECG PMH Social - With whom do you live? Support?  Screen for domestic violence or spouse abuse - Children? - Financial support? Notes  Did you ever think to hurt yourself? NO, my kids need me, o What about if they are not around? Maybe! o This means: implicit yes to suicidal ideation 

Lady looking for renewal of benzodiazepines: o Renew it and tell her that she needs to use it properly o Tell her that she is in grief, she needs SSRIs, start SSRIs, taper benzodiazepines

OSCE-guide-III.doc

Page 223 of 255

Psychiatry

Domestic Violence – Spouse Abuse Screen for domestic violence or spouse abuse: -

-

-

ASSURE confidentiality: I would like to assure you that our conversation is completely confidential, whatever you will tell me here, I will not release any information, unless otherwise required by the law! With whom do you live? How do you describe this relationship? Supportive? o How long have you been in this relation? o Do you feel safe at home? In this relationship? Do you or your partner go through stressful times? o Do you sometimes have conflicts? Arguments? Is there any chance that you partner drinks or uses drugs? How often? When he drinks, does he become angry? Lose control? When was the last time?

Verbal / emotional: - Does he start to shout at you? Swear at you? - Does he call you names? How does this affect your self-esteem? Physical: - Did he ever get angry to the extent that he became physical? - Did he try to put you down? Does he try to control you? How did this affect you? - Did he try to push you? Hit you? How many times? - Any visits to the ER? When was the last time? Financial: - Who is controlling the spending at home? - Do you have access to financials? Do you take permission? - Did he ever to try to take you money against your wishes? Sexual: - Did he ever force you to do sexual activity against your will? How do you feel? -

Duration of abuse? Severity? Hospitalized! How does this affect you? Are you pregnant now? Do you work now? How do you support yourself financially?

Children involvement: - Did he ever mistreat / abuse you in front of the children? - Did he ever mistreat / abuse the children? Fatality: - Do you have access to weapons at home? - Did you ever have thoughts to put an end to this all by ending your life or his life? - Did you ever talk to anyone about this? OUTCOME: - The patient decides to end the relationship and leave  you must provide support and shelter - The patient decides to continue: either with OR without police involvement

OSCE-guide-III.doc

Page 224 of 255

Psychiatry

Wrap-up: - Based on what you have told me, what you are experiencing (or have gone through) is called domestic violence or spouse abuse, and it is common. It is an illegal crime, and it is against the law. - It is not your mistake, and you should not feel guilty about that. It is unacceptable, and nobody deserves to be treated in this way. - We know from studies that the situation will not improve, on the contrary, it will deteriorate, and you do not need to accept this. The studies show that the longer you stay in this relationship, the higher the chance of abuse. - Consequence to the children (if any): psychological trauma - It is important that you consider reporting the situation to the police for your safety. It is difficult decision to leave or stay. - The husband needs help, check willingness to get counsel. -

-

-

-

If you like to end the relationship and leave: o You can call the police, they will come and arrest him, then they will investigate the case, and may put charges against him, and you do not need to worry that he might hurt you, the police can give a restraining order o I will connect you with the social services and support groups, you do not need to go through all of this by yourself, they will be able to help with housing, financial support for both of you and the kids However, if you want to continue the relationship, you can still involve the police. We know from the studies that if you involve the police, situation will improve, they will come, speak with him, do some investigations, and then, by law, your husband will be obliged to attend special training courses: o Anger control o Stress management and relaxation techniques o Drinking problem rehabilitation o Marital counselling My concern is that if you go back without taking any measures, things might deteriorate and get out of control and one of you might end up losing her/his life, at least: o You need to prepare an escape plan:  Where to go when you don’t feel safe!  A bag with essential belongings (e.g. IDs, bank documents, …)  Easily accessible (you can pick it in second and leave) o We will schedule a follow-up visit within few days Is it ok with you if I document this? Support: o Police is not the only option, in the city, there are community recourses: shelter, hotline, legal aid, they are free, and confidential. o Give social support group numbers.

Clues for domestic abuse: - She is avoiding eye contact - Not answering directly for questions about the relationship with partner - I hate drinking - He is a great father, likes his daughters, and works very hard. - I wish I can be a better mom / I am a failure / do you think too I am clumsy.

OSCE-guide-III.doc

Page 225 of 255

Psychiatry

Child Abuse18 The child came to the ER with femur fracture, the skeletal survey showed multiple healing fractures, counsel Introduction

Analyze the event

Is it the first time?

BINDE

Other children PMH of the child

… I assure you that he is ok, and after we finish I would accompany you to see him, is it ok with you. Before this I would like to ask you some questions to know more about his condition / fracture - Can you describe what happened? What he was doing? - Who witnessed it? Anybody else? - When did this happen? When did you come to the ER? - Other injuries before or visits to ER? - Analyze each event - Did you take him to the same hospital?  Are you the biological mother?  Is your current partner the biological father? - B: screen for the risk factors for child abuse:  Was this pregnancy planned? Regular f/u visits?  Was he a term baby? Did he need special attention?  Has he had congenital anomalies?  Do you think he is a difficult baby? Fussy baby?  SAD for both partners! - I: Are his shots up-to-date? If no, any reason? - N: What is his weight? Do you know about his growth charts? Regular f/u visits? - D: Is he hyperactive baby? Challenges you most of time? - E:  How do you support yourself financially? Any support from the biological father?  Anybody at home seeing a psychiatrist? Illness?  Tell me more about your childhood - Do you have other children? - Repeated visits to ER? Chronic illness / bone or metabolic diseases

Screen for domestic violence or spouse abuse

18

Good TWO screening questions: immunization (not up-to-date) / weight (FTT or under nutrition)

OSCE-guide-III.doc

Page 226 of 255

Psychiatry

Wrap-up: -

-

-

How do you feel your child has so many fractures? … I know that you are concerned about your son, sometimes it is challenging to look after a child, especially if you do not have enough support. In the same time, children at that age have flexible bones, and it is difficult to explain the nature of his fracture(s) only by jumping from a couch. In these situations we usually involve the children aid society (CAS), this is a kind of social services devoted to the safety and well being of children o Please do not do this? Why? o They will take my son! Why r u saying so? Any experience with them? o Not necessarily that they take your child, this is not their first priority Their first concern is the safety of your child, what will happen is that they will: o Ask you some questions about what happened! o Come to visit you at home o Talk with your partner Then they will take their next step based on the results of these meetings I am sure you are sharing my concerns about … (the child name) safety!

If there is spouse abuse / domestic violence: - Also, based on what you have told me, what you are experiencing (or have gone through) is called domestic violence or spouse abuse, and it is common. It is an illegal crime, and it is against the law. - It is not your mistake, and you should not feel guilty about that. It is unacceptable, and nobody deserves to be treated in this way. - We know from studies that the situation will not improve, on the contrary, it will deteriorate, and you do not need to accept this. The studies show that the longer you stay in this relationship, the higher the chance of abuse. - It is important that you consider reporting the situation to the police for your safety.

OSCE-guide-III.doc

Page 227 of 255

Psychiatry

Domestic abuser You are bout to see a 55/60 years old gentleman, whose wife is recovering in the ER, she has bruises, and he asked to speak with you. In the next 10 minutes counsel him Introduction Analysis SH / Safety Counsel



Domestic violence





Anger control Stress management and relaxation techniques Drinking problem rehabilitation Marital counselling



Offer social support if there is a need

 

Introduction:  If the patient asked to see you: I understand that you are here because you are accompanying your wife, she has bruises and my colleagues are taking care of her right now. How can I help you today?  If the patient is inquiring about her status: I can assure you that she is stable and in safe hands now.  If the patient asks to see her: After we will finish, I will ask her, if that is ok with her, I can take you there. Analysis:  Do you have any idea how did she end up having all these bruises?  Was there any argument / disagreement / shouting? Did you lose control? Did it end up that you physically hurt her?  Is this the first time or happened before? Any repeated visits to the ER before? Social history: How long have you been together? What is the nature of your relationship? Stable? Was there and significant conflicts before?  Was there any recent change or stressor in your life? How do you support yourselves financially? Do you have enough resources?  Do you have anybody else at home? Any family support? Do you have children? How is the relation with them?  SAD 

Safety:  Criminal record / access to weapons at home  If you go home now and face the same situation, how would you react?  Any chance that you might hurt yourself or any other one?

OSCE-guide-III.doc

Page 228 of 255

Psychiatry

Counselling:   





I can see that you are going through stressful period of time. It must be difficult for you and your wife. Sometimes this stress might present by changes in behaviour and/or personality. If you do not have enough support at home, things might get out of control. What happened is what we call “domestic violence”; it is a kind of “physical abuse”. It is not acceptable, and it is considered illegal crime. However, this is your wife decision. If she chooses to report you, that is her right, and nobody can prevent her. She can press charges against you, and they will take you to the court, in this case you might need legal help, this might have serious consequences. On the other hand, if she decides not to take any measure, may be you should try to improve the situation by taking steps to decrease the stress in your life, and you can consider reducing your alcohol drinking. Drinking alcohol might leads to what we call “disinhibition” in which one might lose control on his reactions and usually this leads to violent and serious consequences. I can help you by referring you to attend: o Alcohol rehabilitation programs o Stress management and anger control programs



I recommend also that you consider attending family marital counselling; they have good experience in dealing with couples going through difficult times.



Finally, I can help you to contact the social services. They might be able to help; you can speak with them and see what they might be able to do! Is that ok with you?

OSCE-guide-III.doc

Page 229 of 255

Psychiatry

Depression Screen: MI PASS ECG Organic: Illness: hypothyroid, anemia / pernicious anemia, M.S, cancer / cancer pancreas  Medication B Blockers, Anti-parkinsonian  SAD 

Depression management / counselling 7. Inform the patient a. Based on what you have told me, the most likely explanation for your condition is a medical condition called “depression”, what do you know about depression? Did you read anything about it? It is the most common mood disorder, in which you feel low, upset and lack of energy. It is a common problem, and it is treatable. b. Explain the pathophysiology: it is related to imbalance of the chemicals in our brain, most likely related to decreased serotonin c. Consequences / complications of the condition: it affects functionality, leads to decreased concentration and ability to work, and in severe cases in susceptible persons, it might lead them to suicide 8. Treatment (outpatient): a. Talk therapy (psychologist / psychiatrist) b. Medications: SSRIs (which are very effective medications) take effect after few weeks  Cipralex 10 mg PO od x 3 weeks (side effects include: 1. weight gain, 2. GIT symptoms, 3. sexual dysfunction). These side effects improve by time and to reduce it, we start increasing the dose gradually (start low, go slow).  Because depression has serious (fatal) consequences, if you do not take this medication, you will be compromising your safety.  DO NOT stop it on your own; we can start to gradually decrease the dose after the proper period of treatment. You need to continue on it for at least 6 months after symptoms improve. c. Follow up visit after 2-3 weeks d. Contract: sometimes when the anti-depressant starts to work, the energy level improves while the mood is still low, that is why sometimes there is increase in suicidal ideation. Usually happens 2-3 weeks, you need to promise me that if this happens with you, call 911 or call me immediately and come to see me 9. Offer more info: brochures / web sites 10. Break every 30-60 seconds and ask the patient: does that make sense? Is this acceptable? Reasonable? Is it clear? 11. Treatment (hospitalization): for suicidal patients a. From what you have told me, you are meeting the criteria of what we call “…” and I have concerns about your safety, because you have more than THREE risk factors for suicide as per the screening test. Do you mind to stay with us in the hospital for few days, so we can do the required investigations and start the medications, until you feel ok, what do you think about that? ─ No doctor, I am not staying in the hospital!!! ─ Actually, Mr … as I told you, I have concerns about your safety, we can not compromise your safety. And by allowing you to leave today, we will be compromising your safety.

OSCE-guide-III.doc

Page 230 of 255

Psychiatry

Dysthymia COMMON CASE IN THE EXAM 

Depression presentations: o Sad (low mode), weight loss, insomnia, tired  Scale the sadness 0 – 10 o Indecisiveness: difficulty making decisions o Low self esteem  how do you feel about yourself? o If good days: ask for periods (check for gaps ≤ 2 months)



How does it affect your life? o Then assess functionality; what do you do?!



Screen MI: o If positive  MI PASS ECG  If positive  assess SAD PERSONS



Any relation to menstrual periods  pre-menstrual dysphoric disorder



Counselling: similar to depression, but mention that Dysthymia is a milder form of depression, with longer duration, and does not interfere with life functionality.

OSCE-guide-III.doc

Page 231 of 255

Psychiatry

Premenstrual Dysphoric Disorder (PMDD) DSM-IV-TR Diagnostic Criteria for Premenstrual Dysphoric Disorder A. In most menstrual cycles during the past year, five (or more) of the following symptoms were present for most of the time during the last week of the luteal phase, began to remit within a few days after the onset of the follicular phase, and were absent in the week post-menses, with at least one of the symptoms being one of the first four listed 1. Markedly depressed mood. Feelings of hopelessness, or self-deprecating thoughts 2. Marked anxiety, tension, feeling of being "keyed up" or "on edge» 3. Marked affective lability 4. Persistent and marked anger, irritability, or increased interpersonal conflicts 5. Decreased interest in usual activities 6. Difficulty concentrating 7. Lethargy, easily fatigued, lack of energy 8. Change in appetite – overeating or specific food cravings 9. Hypersomnia or insomnia 10. A sense of being overwhelmed or out of control 11. Physical symptoms – breast tenderness or swelling, headaches, joint or muscle pain, sensation of bloating or weight gain B. The disturbance markedly interferes with work, school, social activities or relationships with others C. The disturbance is not merely an exacerbation of the symptoms of another disorder such as Major Depressive Disorder, Panic Disorder, Dysthymic Disorder or Personality Disorder D. Criteria A, B and C must be confirmed by prospective daily recordings and/or ratings during at least two consecutive symptomatic cycles (how to diagnose) Treatment  1st line: SSRIs highly effective in treating PMDD o Fluoxetine (20 mg od) and sertraline (50 mg od) most studied o Can be used intermittently in luteal phase (mid cycle  onset of menstruation – pre-menstrual) for 14 days  2nd line o Alpraxolam (Xanax) for anxiety symptoms  3rd line o ± OCP containing progesterone drospirenone (e.g. Yasmin) o GnRH agonists (e.g. leuprolide) o If GnRH agonist completely relieves symptoms, may consider definitive surgery (i.e. Total abdominal hysterectomy+ bilateral salpingo-oophorectomy)

OSCE-guide-III.doc

Page 232 of 255

Psychiatry

Abdominal Pain / Headache Abdominal pain for few weeks, and was seen by a surgeon last week, comes to your office (± to have MRI OR to renew medication). Headache for 7 months, young man, comes to renew Tylenol 3 Chief complain ± a request (investigations OR medication renewal) HPI

Analyze the CC

   

AS

  

Impact Red flags

  

 

Os Cf D / PQRST / ↑↓ / 1st time When did the headache (pain) start? Did you seek medical attention? What was the diagnosis? Did you take any medication? When did you start Tylenol 3? Why? Analyze previous visits: is the pain different from before? How? Other pains / headache GIT / liver Genito / urinary How does this affect your life? How are you coping? Constitutional symptoms Screen red flags for headache:  Trauma  Worse at night  Nausea / vomiting  Bothered by light /+/ Neck pain / stiffness  Weakness / numbness / tingling in body / seizures Are you under stress? Support systems

DD Medical problem PMH FH SH

Somatisation MOAPS screening (screen for depression) PMH: HEAD SSS FH of psychiatric disease SH Counselling

Physical examination Notes: - Somatisation disorder: (4 pains / 2 GIT / 1 neuro / 1 sexual) complains - If the pain is only during the day, and not nights  mostly non-organic

+ If requesting MRI  NO MRI + Actions for Tylenol 3: - If using it for few weeks  stop it / do not worry about withdrawal symptoms - If using it for long time  counsel / renew / promise to cut down gradually - If using it for depression  start SSRIs / taper Tylenol 3 (decrease gradually) / then brief counselling for BOTH Tylenol 3 & depression - If drug seeker  DO NOT give any narcotics / rehabilitation + Always renew the medication, except for drug seeker (very anxious to renew the narcotic / will not accept another alternative / making stories to rash you to prescribe it). + If not drug seeker: reassure the patient that you will prescribe her pain medication before the end of the session, but in order to prescribe the proper medication, you need to ask some questions, can you bear with me for few minutes?

OSCE-guide-III.doc

Page 233 of 255

Psychiatry

Somatoform disorders DD General Characteristics:  Physical signs and symptoms lacking a known medical basis in the presence of psychological factors  Cause significant distress or impairment in functioning  Symptoms are produced unconsciously  Symptoms are not the result of malingering or factitious disorder which are under conscious control  Primary gain: somatic symptom represents a symbolic resolution of an unconscious psychological conflict; serves to reduce anxiety and conflict; no external incentive  Secondary gain: the sick role; external benefits obtained or unpleasant duties avoided (e.g. work) Management of Somatoform Disorders:  Brief frequent visits  Limit number of physicians involved in care  Focus on psychosocial not physical symptoms  Minimize medical investigations; co-ordinate necessary investigations  Biofeedback  Psychotherapy: conflict resolution  Minimize psychotropic drugs: anxiolytics in short term only, antidepressants for depressive symptoms

Somatization disorder



Conversion disorder





Pain disorder



─ ─

Hypochondriasis



Fibromyalgia

─ ─ ─ ─



Lots of symptoms: ≥ 8 physical symptoms that have no organic pathology: 4 pain + 2 GIT + 1 sexual + 1 pseudo-neurology One or more symptoms or deficits affecting voluntary motor or sensory function that mimic a neurological or general medical condition (e.g. impaired co-ordination, local paralysis, double vision, seizures or convulsions La belle indifference Pain is primary symptom and is of sufficient severity to warrant medical attention Post-traumatic / post-surgical Exacerbated by psychic factors Preoccupation with fear of having, or the idea that one has, a serious disease (e.g. brain tumour) based on a misinterpretation of one or more bodily signs or symptoms Onset often after car accident Wake from sleep feeling un-refreshed Wide spread pain, above and below waist, both sides of the body Characteristic reproducible tender points: occiput, low cervical C5C7, lateral border of the sternum, post neck, lateral epicondyle 2cm below that point, lat thigh, med knee Treatment:     

Chronic fatigue syndrome Factitious disorder / malingering

OSCE-guide-III.doc

─ ─

Patient education Exercise program (walking, aquatic exercises), physical therapy (good posture, stretching, muscle strengthening, massage) Stress reduction, CBT Amitriptyline 10 – 25 mg qhs Gabapentin 300 mg tid

Similar to fibromyalgia but FATIGUE is the predominant feature Associated with sleep apnea / irritable bowel syndrome

Page 234 of 255

Psychiatry

Counselling for somatisation disorder: - I understand that you are here because of …, and to (renew medication / do MRI / …) and we will discuss that, but before discussing this, I would like to explain the findings in your case. - Based on the symptoms (± and the surgeries you had) the most likely explanation to your pain (headache) is a medical condition called “somatisation disorder”. - What do you know about “somatisation disorder”? Would like me to explain? - It is not uncommon condition, and we do not know the exact explanation for it, but we believe that because some patients are more sensitive to pain than others, or may be due to patients difficulty in handling stresses in their lives, these stresses may manifest as painful experiences (symptoms). - Do you have a family doctor?  YES o I will explain some points for you now, and then you can arrange a meeting with your family physician and discuss the follow up with him, in these situations, it is important to have only one physician dealing with all the investigations so that he can get better understanding of the whole situation.  NO, I do not have a family physician! o I can be your family physician, if you would like to. That means we will set a follow-up visits every 4 weeks, during which we will review underlying symptoms, to make sure we are not missing any serious condition. o We will review the stress in your life and see how we can help you with it:  I can refer you to psychiatrist to help you deal better with any stress / conflict in your life  And we can consider some medications (Amitriptyline 25 mg PO qhs), it belongs to a family of medications called TCA (tri-cyclic antidepressants) but we use it for pain control Counselling if the patient is depressed: - I understand that you are here because of …, and to renew medication and we will discuss that, but before that, I would like to explain the findings in your case. - Based on what you have told me, the most likely explanation to your symptoms is a medical condition called “depression”. - We need to treat the depression with [talk therapy, behavioural modifications, and medications (SSRIs, TCA)]  depression counselling - For Tylenol 3 we will not stop it suddenly, I will renew it for you, but we will agree that you will gradually cut it down, over the next few days, till the other medication (SSRIs) kicks in.

-

If there was a suicidal attempt: However, because of the suicidal attempt 2 days ago, we would like you to stay with us in the hospital for few days so that we can start the treatment

OSCE-guide-III.doc

Page 235 of 255

Psychiatry

Drug seeker If you find a man searching in the drawers of the hospital, firmly ask him to stop, tell him this is private property and he is not allowed to go through this medical stuff I wish it could be that simple, but I need more information and physical exam before I can write any prescriptions to you, as I am a little bit concerned about the amount you have been taking, which might have been harmful to you

Introduction HPI Analyze the CC

    

AS

  

Impact

    

Red flags

 

 

Analyze Tylenol 3 Other medications

Why are you taking it? What was the diagnosis? Os Cf D / PQRST / ↑↓ / 1st time When did the headache (pain) start? Did you seek medical attention? What was the diagnosis? Did you take any medication? When did you start Tylenol 3? Why? Analyze previous visits: is the pain different from before? How? Other pains / headache GIT / liver Genito / urinary How does this headache affect your life? How are you coping? Have the medications been impacting your life? Relationship with family Education, Employment Legal problems, police involvement? Constitutional symptoms Screen red flags for headache:  Trauma  Worse at night  Nausea / vomiting  Bothered by light /+/ Neck pain / stiffness  Weakness / numbness / tingling in body / seizures Are you under stress? Support systems

In addition to Tylenol 3, do you take any other meds? Sleeping pills? MOAPS screening PMH: HEAD SSS FH of psychiatric disease SH

Counselling Analyze Tylenol 3 -

So you told me you are taking it for … Who prescribed it to you? Who renewed it to you? Why? When was the last renewal? Can you show me your last bottle? How many tablets do you use now? And before? When did you start to ↑ the use? When you take it, beside for the headache relief, how do you feel? What if you do not take it, how do you feel? Shaking? Heart racing? You feel you are on the edge? Do you renew it from the same doctor or different doctors? Why you did not go to him this time? Is it ok that I contact him? Do you renew it from the same pharmacy or different pharmacies? Is it ok that I contact the pharmacy? Did you ever obtain the medication from the street?

OSCE-guide-III.doc

Page 236 of 255

Psychiatry

Given the benign history with no suspicion of ↑ ICP or focal deficits, and description of headache consistent with the common tension headache, full neurological examination is not indicated, I would like to perform a brief neuro screening exam  move on. Counselling: - I understand that you are here to renew your Tylenol 3; we will discuss that, but before that let me ask you: what is your understanding of Tylenol 3? - Tylenol 3 is a good medication when it is used for particular indication. Do you know what does it contain? It contains 2 medications: o One of them is the regular Tylenol as you buy it from the pharmacy o The other one is codeine - Tylenol itself is a safe and effective drug, and can be used for long time, however, if there is no strong indication to use it, it is better to ↓ it as it might cause liver and kidney injury. - On the other hand, the other medication “codeine” it is a drug belongs to the family we call “narcotics” which is similar to morphine. It is an excellent pain killer if used for short term, but, if it is used for long term, this is concerning for us, do you know why? o First of all, people need to keep increasing the dose in order to obtain the same effect; we call that “tolerance”. o Also, if you stop using it suddenly, you will have “withdrawal” symptoms, similar to that you have now; running nose, tearing, N/V, diarrhoea, drowsiness, muscle aches, sweats, shaking, and heart racing. - For these reasons, people get easily hooked on Tylenol 3, and can not stop it. Not only that, they will need to keep increasing the dose. We call that “a habit forming medication”. - If I renew your medication, I will not be helping you, it will be like a vicious circle, and the more I renew your medication; the more dependent you will be on it; the more you will need it. For that reason it is not the right step to renew it. o Can you give me just few pills; I have a very important interview? o Even if I give you few pills, this is not the solution, this will be temporarily, and the problem will keep increasing. We must stop the drug o I can help you with “sick note” o I can give you another non-narcotic medication that can help you with your pain - I appreciate your trust to give me all the information, but based on what you described, you are having “dependence” on narcotics. - It sounds like you have been going through a lot of stress in your life. I am wondering that if you would be interested in talking to one of our social works here, who is expert to find out the community resources for you. There are also some numbers you can call; they are professionals to help people deal with medications or drugs. Or if you like, I can refer you to a detoxification center, where they will help you to quit. The seeker may be seeking “Fiorinal” -

Fiorinal is a combination preparation of (barbiturate / caffeine / ASA) properly used only for the relief of occasional tension headaches. It is a “habit-forming” medication; that can precipitate withdrawal symptoms: agitation, delirium and seizures. The fact that patient consumes a lot suggests overuse due to dependence. The patient may also develop “analgesic headache” syndrome, in which inappropriately used analgesics actually cause headache. Suggest a “drug holiday”, with weaning from caffeine and alcohol, proper sleep hygiene, diet control, exercise, and stress management.

OSCE-guide-III.doc

Page 237 of 255

Psychiatry

Lithium discontinuity Introduction

   

Mania

History

Have been diagnosed with bipolar 3 years ago, and would like to discontinue your medication. What is the medication you want to stop? Why would you like to stop your medication? I am glad you came here to discuss it, any other concerns

When were you diagnosed? How? Any serious consequences? Were you hospitalized? For how long?  Were you seen by a psychiatrist? Regular f/u? Do you feel: DIG FAST (distractibility, impulsiveness, grandiosity, flight of ideas, activity, sleep, talkative) MI PASS ECG  

Today

Lithium

Scan for depression History

Do you renew your medications on regular basis? How much lithium do you take? From the beginning?  Are you taking it regularly?  Do you measure lithium level? On regular basis? When was the last time? What was it? What is your target?  Are you still taking it? Did you stop?  How do you feel about lithium?  Hypothyroidism: do you have your thyroid hormone measured? Do you feel cold? Dry skin? Constipation?  give thyroxin  Diabetes insipidus: do you feel thirsty? Drink more? Pee more? Got your urine checked? ttt: thiazides  Abdominal pain? Nausea / vomiting?  Neuro – shaking/tremors: β-blockers  Neuro – ataxia/balance/seizure: stop it I know that you have been asked all these questions before, let me ask it for another time!  

Side effects

MOAPS

Do you feel: DIG FAST (distractibility, impulsiveness (with painful consequences), grandiosity, flight of ideas, activity, sleep, talkative) D Do you have a lot of projects? Were you able to finish it to the end? Can you focus on multiple projects?  Are you spending more money than before? Are you borrowing money that you can not I pay back? Are you over-using your credit cards?  With whom do you live? Many sexual partners?  SAD: what started 1st; feeling high or talking drugs?  Have you had problems with the law? Fighting? Arrest? Speeding tickets? G Do you feel very special? Have special mission? Do you feel a lot of thoughts? Ideas? F A How much time do you spend on your projects? How many hours do you sleep? Any changes? S T Did anybody mention that you are talking fast?

OSCE-guide-III.doc

Page 238 of 255

Psychiatry

Side effects of any medication: - Liver toxicity - Nausea, vomiting, diarrhea - Headache - Insomnia, irritability - Agranulocytosis (Carbamazepine): check CBC every week, then every 2 weeks, then every month Counselling: - I understand you are here because you would like to discontinue the lithium, however before we discuss that; I would like to know your understanding about mania and mood disorders! - Mood disorders are common, and the most common of them is depression where people feel low and do not concentrate and its treatment include the talk therapy and medications that could be used for 6-12 months and could be stopped if the condition improved and in some times we need to give the treatment for longer periods of time. - This is not the case for mania/bipolar. We can treat and control it, but we can not cure it, may be one day in the future we will be able to do this. - Your chance of relapse if you stop it is 60% and after the second time this goes up to 80%, and after the third time it goes higher to 90%. You can see it is increasing. - Based on your lithium level, which is within therapeutic target (0.5 – 1.2), we can measure it today and we can try to decrease it gradually to check if you are feeling good and closely monitor you. But you have to promise me that at anytime you feel high mood, start to spend too much, talk fast or start not to sleep well, you have to come to see me or go to the nearest ER and inform them. - Regarding your inability to write, this is not related to lithium, thought block is not a side effect of lithium. You may try some relaxation techniques to help you concentrate more.

OSCE-guide-III.doc

Page 239 of 255

Psychiatry

Manic patient If the patient is psychotic: First step is to detect early what is his mood? - If high mood: manic attack, with psychotic feature - If no high mood: brief psychotic disorder / schizophreniform Usually patient brought by police or family member or asked to come by family members  Patient is talking fast and a lot, laughing, moving around  Ask whether the patient has been on medication before or not, e.g.: Lithium  Ask about any side effects of lithium medication N/V / Diarrhea / tremors / polyuria  Obtain history in the usual format Introduction Ask about the Mood Assure the patient Red flags HPI

MOAPS

PMH / FH SH

Assure the patient: you are safe here, you are in the hospital and no one will hurt you Fever / headache / nausea & vomiting / head injury  OCD  Mania (DIG FAST)  Depression (MI PASS ECG)  Suicide (SAD PERSONS)  If you leave what will happen? What would you like to do? Screen  SAD: alcohol / substance abuse / amphetamine  Medical conditions; hyperthyroidism: history of thyroid problems, symptoms (heart racing, sweating, heat intolerance, neck swelling, visual field changes) Psychiatric disease

N.B. if any patient has mood disorder; go through DIG FAST and MI PASS ECG Management:  Explain that the patient has recurrence of his mania or bipolar. This is because he stopped taking the Lithium.  Will examine and do some tests.  Will start medication. If Lithium is causing some troubles, we can start another medication.  Usually you need to admit the patient to control the symptoms of mania (from what you have told me, you are meeting the criteria of what we call “manic episode” and I have concerns about your safety).

OSCE-guide-III.doc

Page 240 of 255

Psychiatry

Suicidal attempt Introduction

-

Analyze the event

-

Before

After

Psychiatric assessment Risk MOAPS

-

-

LOTS OF EMPATHY … And to see what should be the next step, first, I would like to know how you feel about being saved. o If happy, I am glad for that o No! Assure confidentiality Can you tell me more about what happened? What is the name of the medication? How many tablets? Any alcohol with it? Why did you do that? Is it the first time? Who saw you and brought you to the hospital? Assess the plan here, was it organized? Or it was an impulse? Did you leave a note? Recently, have you been giving your belongings away? What is going in your mind now? If you leave the hospital, what are your plans? Where do you want to go? What do you want to do? If another crisis may happen, are you going to hurt yourself? Were you seen by a psychiatrist? Were you given a diagnosis? Do you see your psychiatrist? Take meds? Assess the risk factors: Analyze SAD PERSONS Screen for anxiety Screen for psychosis Screen for suicidal / homicidal ideation / self care Past medical history / allergy / medications / …

Decision Conclusion / Counselling SAD PERSONS S A D Sex Ag Depressio Mal e n e > 65

P E Previou Ethano s l attempts SAD

3-4 >5 E R N

S

R Rationa l thinkin g lost

S Suicid e in the family

O Organize d plan

N NO suppor t

S Seriou s illness

HEAD PMH SSS

Release if enough support Hospitalize -

SAD What did you think will achieve by ending your life? Sometimes people hear voices asking them to end their life, did you hear this? HEAD SSS H: With whom do you live? … Anybody else? … Anybody else? If there is a stepparent in the image, ask about the relations with him and with other parents. Do you feel safe at home? Then ask gradually, if there is a chance that this parent might get angry when he drinks? May shout, may swear at, may push, and may hurt? - Past medical history

OSCE-guide-III.doc

Page 241 of 255

Psychiatry

Decision: - If still depressed and/or SAD PERSONS (>3-5)  admit - It she is ok, regrets the accidents, no SAD PERSONS  release Conclusion / Counselling: HOSPITALIZE - Based on our interview, I have concerns about your safety, because you have more than THREE risk factors for suicide as per the screening test. Do you mind to stay with us in the hospital for few days, so we can do the required investigations and start the medications, until you feel ok, what do you think about that? RELEASE - Based on our interview, it is ok if you would like to leave, but you have to arrange a follow up meeting with your family doctor within 3 days. - However, I would like you to know that life sometimes could be challenging, and you may face challenges in the future. It is important that you learn how to deal with challenges. If you feel over whelmed, talk to somebody, and ask for help - I can arrange a meeting with a social worker, a psychiatrist! - I would also like you to promise me that if at any time you want to harm yourself or end your life, you will seek medical help immediately; you can come to my office or call 911. If multiple suicidal attempts  borderline personality disorder  do NOT admit Notes: - If no eye contact, wasting time, no pt interaction  assure confidentiality - Whenever you hear “car accident”  show empathy / did you hurt yourself / ask about who was in the car / was any one injured? - If the person driving was < 18 and was driving alone  be curious  this must be an important meeting / person that you really did not want to miss! - The girl asks you to tell her mother that she crashed mother’s new car! She does not want to directly (herself) inform the mother! o I can not do this. o Why do you think this would help? “She will not be angry” I see, however, life is full of challenges, it is better that you try to learn how to deal with challenges yourself. o We can help you to tell your mother by yourself, we can arrange a meeting with your mother, I can be present, or we can ask a nurse or a social worker to be there. - The girl does not want to inform her parents that she did attempt suicide! o You assess her and if she is to be released, e.g. she regrets what happened, she is happy to be saved, no SAD PERSONS risk factors  she is competent  respect her wishes.

OSCE-guide-III.doc

Page 242 of 255

Psychiatry

Eating disorder Young female, her parents brought her because they have concerns about her weight Anorexia nervosa Restrictive

Bulimia nervosa Binge-purge Under weight Distorted self image Amenorrhea

Disturbed perception  loss of insight  incompetence  inform parents and admit involuntarily Introduction

Average low weight

Binge-purge (> 3 times / week) > 3 months Lose control  over-eat  react (purge)

Your parents brought you …. How do you feel about that? I am glad that you came: - To figure this out (if she is ok) - To assure your parents (if she is not ok)

Weight analysis Diet Exercise Extra measures Impact MOAPS - Mood: scan for depression - Organic: DM / hyperthyroidism / constitutional symptoms (cancer) - AP: screen for anxiety / psychosis - S: HEAD SSS FH Eating disorder / psychiatric illness / suicide Weight analysis: - What is your weight today? - When did you start to lose weight? What was your weight at that time? How much did you lose? What was your highest weight? What is your target weight? - Why are you losing weight? - Are you losing weight alone? Or someone else is encouraging you? - When do you look at yourself in the mirror, how do you perceive yourself? How do you perceive your weight? - Do you like to dress in baggie? - It looks like you lost a lot of weight in short period of time; I would like to know how did you achieve that? Diet: Let us talk about your diet; - How many meals do you eat per day? How about snacks? o What do you eat in breakfast? How about the amount? - Do you calculate calories? How much calories do you eat per day? - Do you eat alone or with other people? - Do you like to collect recipes? To cook?

OSCE-guide-III.doc

Page 243 of 255

Psychiatry

Exercise - How about exercise? Do you exercise? - How many times a week? - Do you dance? Practice any sports? Extra measures: - Do you take anything else to help you to lose weight? - Do you take stool softeners? Do you take water pills? - Did you try before to induce vomiting? - Do you sometimes exceed the amount of food you intended to eat? How many times a week? - How do you feel after that? How do you compensate? Impact / consequences: Because you have lost a lot of weight, I would like to know the impact of this on you! - Do you have amenorrhea? When was the LMP? Regular? - Do you feel cold / tired / swelling in your legs? - Pigmentation on your skin? Fine hair growth? Skin changes? - Any bony pains? Fractures? - Muscle cramps? Calf pain? - Heart racing? Light headedness, dizziness, fainting? Conclusion: - I am concerned that you have a condition called “Anorexia Nervosa” (explain) - It is affecting your body, without treatment it could be fatal - The treatment is to start eating and to gain weight. It is a tough task but I will refer you to a multi disciplinary team to start treatment - Would you like to discuss that with your parents Management of anorexia nervosa: - Anorexia patient is to be admitted to hospital if: o
View more...

Comments

Copyright ©2017 KUPDF Inc.
SUPPORT KUPDF